🔙 Quay lại trang tải sách pdf ebook Tạp Chí Epsilon Số 14 Ebooks Nhóm Zalo No 14 Không phải thực tế mà chính là phép toán là thứ sinh ra các con số và cho chúng tính chất!” “ SỐ VÀ LỊCH SỬ PHÁT TRIỂN LOÀI NGƯỜI – Nguyễn Lê Anh “ “Xin nhấn mạnh: tôi xem thống kê học là một cách suy nghĩ BẢY TRỤ CỘT THÔNG THÁI CỦA THỐNG KÊ HỌC Nguyễn Văn Tuấn VÀ CÁC CHUYÊN MỤC KHÁC 13. 12. 2018 MỘT SỐ BÀI TOÁN TRÊN TÂM ĐƯỜNG TRÒN EULER Trần Quang Hùng TOÁN HỌC VÀ ẢO THUẬT Nguyễn Hùng Sơn NHỮNG CƠ HỘI MỞ RA SAU NGÀY HỘI TOÁN HỌC MỞ Phạm Hy Hưng NO 14 Dec 2018 CHỦ BIÊN: TRẦN NAM DŨNG BIÊN TẬP VIÊN: LÊ VIẾT ÂN LÊ PHÚC LỮ NGUYỄN TẤT THU VÕ QUỐC BÁ CẨN TRẦN QUANG HÙNG NGUYỄN VĂN HUYỆN ĐẶNG NGUYỄN ĐỨC TIẾN LỜI NGỎ Bạn có tin vào chuyện cổ tích? Cách đây 4 năm, chúng tôi, cùng với những độc giả đầu tiên đã bắt đầu câu chuyện cổ tích của mình bằng việc cho ra đời số đầu tiên tạp chí Epsilon, một tạp chí với mục tiêu vô tư, phát hành online miễn phí, và phục vụ những người yêu thích toán học. Một tạp chí được xây dựng bởi cộng đồng và cho cộng đồng. Câu chuyện cổ tích thử nghiệm ấy đã được lan truyền đi, được đông đảo bạn đọc đón nhận, chia sẻ, và cùng chung sức xây dựng. Câu chuyện của chúng tôi cũng đã có một kết thúc rất đẹp: chúng tôi dừng lại ở con số 13, khi Epsilon đang ở độ chín. Chúng tôi đã dừng lại trong tự hào, để từ đó mở đường cho một người anh em khác, tạp chí Pi. Chúng tôi dừng lại, nhưng một phần nào đó trong mỗi cá nhân chúng tôi vẫn luyến tiếc về những ngày tháng đầy thơ mộng của Epsilon. Rồi thời gian dần trôi, Pi ngày càng vững mạnh, giờ đây đã là một tờ báo quen thuộc với những người yêu toán. Dẫu vậy, trong lòng các bạn yêu toán vẫn nhớ về Epsilon và họ luôn hỏi chúng tôi liệu Epsilon có ngày trở lại? Vào một ngày đẹp trời mùa hè 2018, được thôi thúc bởi những giấc mơ đẹp, tổng biên tập Trần Nam Dũng quyết định để người em Epsilon tái xuất, cùng đồng hành với anh Pi của mình. Vậy là tất cả chúng tôi tái hợp. Ban biên tập Epsilon những ngày đầu tiên phần lớn vốn là những cậu trai trẻ, độc thân và có rất nhiều thời gian, giờ đây đều đã có gia đình riêng, đã biết thế nào là những vất vả mưu sinh của cuộc sống. Nhuệ khí của tuổi trẻ đang bắt đầu chuyển mình thành kinh nghiệm của những người đàn ông từng trải. Mặc dù vậy, khi nghe lời hiệu triệu của tổng biên tập, tất cả chúng tôi đều không chút đắn đo, cùng trở lại. Chúng tôi lại cố gắng liên lạc với từng đồng nghiệp để có được những bài viết tốt nhất, lại làm việc thâu đêm để có được những bản biên tập đúng hạn và đúng chất lượng. Tất cả có được bởi chúng tôi tin vào nhau, và tin vào những điều kỳ diệu của những câu chuyện cổ tích. Epsilon trở lại và vẫn giữ vững triết lý của mình: phục vụ cho cộng đồng yêu thích toán, trong đó đặc biệt đề cao tính "khả đọc" của từng bài viết. Epsilon sẽ luôn luôn miễn phí, luôn luôn phát hành online và tương lai chúng tôi sẽ từ từ chuyển hoàn toàn sang 100% online: hiện nay chúng tôi vẫn xuất bản theo kiểu tập tin định dạng pdf, trong tương lai chúng tôi sẽ sử dụng sang những định dạng khác, giúp cho tính khả đọc không chỉ đúng với nội dung mà còn với hình thức trình bày. Epsilon sẽ cố gắng mở rộng hơn đối tượng tác giả, sẽ sử dụng hệ thống bình duyệt bởi những người có chuyên môn. Chúng tôi sẽ tập trung cao hơn về chất lượng, do vậy, chúng tôi sẽ chỉ duy trì xuất bản 2 số mỗi năm, vào ngày 13 tháng 6 và 13 tháng 12. Epsilon 14 vẫn giữ bản sắc của mình các với chuyên mục quen thuộc như bài toán hay - lời giải đẹp, điểm sách, toán học giải trí, lời giải và bình luận các cuộc thi, ... và tất nhiên, không thể thiếu những câu chuyện toán học đẹp như cổ tích. Và bây giờ, mời các bạn cùng đọc Epsilon 14. MỤC LỤC Nguyễn Lê Anh Số và lịch sử phát triển loài người .............................. 6 Vaselin Dimitrov Định lý không điểm tổ hợp - Combinatorial Nullstenllensatz ............... 16 Nguyễn Hùng Sơn Toán học và Ảo thuật .................................... 24 Ban Biên tập Epsilon Thế nào là tư duy Logic - Hành trình tìm kiếm máy bay mất tích MiG-21U ........ 29 Võ Nhật Vinh Giới thiệu bài toán tối ưu hai lớp .............................. 49 Nguyễn Song Minh Tính chất Phi Archimedean của Định giá P-Adic ...................... 55 Võ Quốc Bá Cẩn Phương pháp thêm biến trong giải phương trình hàm .................... 66 Ngô Văn Thái Sáng tạo - Làm chặt ..................................... 79 Nguyễn Văn Tuấn Bảy trụ cột thông thái của thống kê học ........................... 93 Trần Quang Hùng Một số bài toán trên tâm đường tròn Euler ......................... 98 Nguyễn Trường Sơn Điểm Humpty-Dumpty trong tam giác và ứng dụng .................... 109 Lê Viết Ân Một số bổ đề hữu dụng tiếp cận lời giải trong các bài toán hình học ............ 134 Nguyễn Duy Liên Bài toán hay - Lời giải đẹp ................................. 164 Lê Phúc Lữ Hướng tới kỳ thi VMO 2018 - 2019 ............................ 168 Nguyễn Trần Hữu Thịnh, Phạm Quốc Thắng, Nguyễn Trường Hải, Võ Thành Đạt, Trần Bá Đạt 4 Tạp chí Epsilon, Số 14, 12/2018 Trường Đông Toán học miền Nam năm 2018 và những bài toán hay ............ 214 Phạm Hy Hưng Những cơ hội mở ra sau ngày hội toán học mở ....................... 233 Trần Nam Dũng Epsilon, MOD, BM2E và tinh thần tình nguyện ...................... 239 5 Tạp chí Epsilon, Số 14, 12/2018 SỐ VÀ LỊCH SỬ PHÁT TRIỂN LOÀI NGƯỜI Nguyễn Lê Anh Tôi thường bắt đầu bài giảng đầu tiên cho sinh viên năm thứ nhất bằng việc thông báo tên của tôi và một số quy định. Những quy định đầu tiên là sinh viên không cần phải xin phép để được vào ra khỏi lớp. Và điều này thì có hàm ý các sinh viên không nên chỉ vì phải đến đúng giờ mà phân tâm khi đi đường và vô tình để bị xảy ra tai nạn giao thông. Tôi còn nói nhiều điều nữa, rồi tôi hỏi - Có đúng là các em đã thi vào trường đại học một cách trung thực không ? Gần như ngay lập tức các em đều khẳng định - Thưa thầy, đúng ạ. Tôi cũng nhận ra vẻ kiêu hãnh tự hào trên khuôn mặt của các bạn trẻ mới bước những bước đầu tiên vào giảng đường Đại học. Tôi im lặng và nói. - Các em có cho phép tôi được kiểm tra không ? - Vâng. - Có ai trong số các em cho tôi biết !3 là gì không ? Thế rồi cả lớp ồn ào, rồi chìm vào sự tĩnh lặng. Tôi gọi một vài sinh viên để họ đứng lên trả lời. Các vị đang dõi theo đọc bài viết này chắc cũng đang tự tìm lấy câu trả lời. Thôi thì đủ các kiểu trả lời. Tôi chờ cho tới khi sinh viên tự nhận thấy là tất cả các câu trả lời của các bạn ấy đều không đúng và tự hiểu ra là không có câu trả lời, rồi tôi nói: - Nếu chắc chắn các em đã thi vào đại học một cách trung thực thì hoặc là bộ Giáo Dục đã không trung thực khi đứng ra làm người cầm cân nảy mực - họ đã không có đủ kiến thức để làm việc ấy - hoặc là trường đại học mà các em đang học không trung thực khi tổ chức thi. Chúng ta sẽ cùng với các em làm cho rõ hơn chuyện này. Thế rồi tôi bắt đầu giảng bài. Tôi nói rằng các con vật không phải là không biết đến các con số, có điều chúng gắn liền con số với sự vật. Khi đưa cho một đứa trẻ nhỏ 5 cái kẹo thì chúng sẽ hình dung ra 5 cái trước mắt chúng. Khi ăn cái kẹo nào là cái kẹo ấy sẽ biến mất đi khỏi sự tưởng tượng. Vậy để kiểm tra một đứa trẻ có khả năng toán học không thì chỉ cần đưa cho nó kẹo, bảo nó cho vào túi, rồi bảo nó ăn, rồi bảo nó cho bạn nó. Rồi bất ngờ hỏi số lượng kẹo còn lại trong túi khi số lượng ấy chỉ vài ba cái. Nó sẽ trả lời đúng cho dù đó là phép toán với các số khá lớn. Những con số như vậy được gọi là số tự nhiên. Nó được dùng để thông báo số lượng các vật thể là kết quả của quá trình đếm. Quá trình đếm bắt đầu là 1, rồi đến 2,... không thể có 6 Tạp chí Epsilon, Số 14, 12/2018 số đếm mà không có gì. Phải mãi tới 20 nghìn năm trước CN, do nhu cầu mà, xuất hiện số 0. Số 0 là số không có gì, tức không phải là số. Đó là sản phẩm nhân tạo phi tự nhiên đầu tiên được loài người tạo ra. Sự xuất hiện số 0 đồng nhất với thời điểm xuất hiện văn hóa. Trong việc sử dụng số đếm thì “1 con bò là hợp lý, nhưng 1=2 con bò là vô nghĩa”. Việc sử dụng các con số như 1=2 cốc nước, 2=3 đoạn đường ... được hiểu là khái niệm số đã thay đổi. Các con số đã bị bỏ đi đặc tính “nguyên khi đếm” và chuyển sang loại con số thông báo lượng. Người ta hình dung các con số như tỷ lệ độ dài của các đoạn thẳng với đoạn được coi là 1 - đoạn đơn vị. Để thông báo một con số nào đó người ta thông báo ra phép dựng hình để tạo ra nó. Và thế là hình học Ơ-cơ-lit ra đời. Theo Ơ-cơ-lit thì số được đồng nhất với đoạn thẳng có thể dựng ra được và chỉ những đoạn thẳng nào dựng ra được mới được coi là con số. Như vậy đường chéo của hình vuông có cạnh bằng 1 là sốp2 (căn bậc hai của 2). Nếu giải thích p3 là số mà bình phương lên thì bằng 3 là vô nghĩa, bởi có biết nó là số đâu mà bình phương lên !! Câu giải thích “sốp3 được là đoạn thẳng sinh ra từ hai điểm cắt nhau của hai vòng tròn bán kính bằng nhau, vòng tròn này đi qua tâm của vòng vòng kia” thì đúng. A p3 1 B Và tất nhiên từ định lý Pitago chúng ta có thể chứng minh được bình phương của nó chính là bằng 3: Câu hỏi “Có thể dựng ra được đoạn thẳng có độ dài bằng với ! là diện tích của hình tròn bán kính đơn vị hay không?” là một bài toán rất khó và phải mãi tới thế kỷ 19 mới có câu trả lời. Câu trả lời là “Không thể dựng ra được đoạn thẳng có độ dài bằng với diện tích hình tròn bán kính đơn vị”. Điều này đồng nghĩa với việc số ! diện tích hình tròn bán kính đơn vị không phải là một con số theo quan niệm của Ơ-cơ-lít. Bằng phép dựng hình chúng ta có thể chia đoạn thẳng đơn vị ra làm 10 phần. Phần thứ nhất được gọi là 0, phần tiếp theo được gọi là 1,.. cứ như thế cho tới 9. Mỗi phần lại có thể chia ra 10 phần và cách gọi tên của đoạn nhỏ là tên của nó kèm theo tên của đoạn mà nó nằm bên trong. Đó là cách gọi theo hệ thập phân mà ngày nay mà chúng ta vẫn dùng để gọi tên các con số. Nếu chúng ta không chia ra làm 10 phần mà chỉ chia thành 2 phần thì chúng ta có cách gọi nhị phân của các con số. Như thế số thập phân 0:99999 : : : tuần hoàn mãi mãi số 9, là ký hiệu đoạn cuối cùng của quá trình chia 10 phần mãi mãi. Nếu coi quá trình trên là việc ăn 1 cái bánh, cứ mỗi lần ăn hết 9=10 chỉ còn lại 1=10, rồi lại ăn hết 9=10 phần còn lại ấy... thì hầu như ai cũng cho rằng “sẽ chẳng bao giờ ăn hết được cái bánh” - vì “Lúc nào cũng còn !”. Tuy nhiên nếu coi đấy là một đoạn 7 Tạp chí Epsilon, Số 14, 12/2018 đường đi từ A đến B, và khi đã đi đến được B rồi thì tất nhiên phải có lúc đi qua vị trí là điểm 9=10 đoạn đường, rồi tiếp theo là đi qua vị trí là điểm 9=10 đoạn còn lại,... cứ như thế. Sự tự mâu thuẫn này không phải là lý do chúng ta không thể đi hết đoạn đường từ A đến B, mà là do chúng ta “ép buộc, lý giải quá trình đi này bằng một thứ ngôn ngữ phi thực tế - ngôn ngữ toán học”. Điều này cũng sẽ xảy ra khi chúng ta nghiên cứu về thế giới. Chúng ta sẽ làm quen với khái niệm một vật vừa là hạt lại vừa là sóng... và còn nhiều điều nghe như phi lý nữa. Những người cho rằng quá trình đi từ A tới B sẽ đi đến được B thì sẽ công nhận dãy 0:9I 0:99I 0:999I ::: có giới hạn và bằng 1; họ là môn đệ của lý thuyết toán học có giới hạn. Dãy “có giới hạn” kiểu như dãy 0:9I 0:99I 0:999I ::: như ở trên, hoặc là dãy sinh ra khi Iasin đuổi con Rùa. Iasin là người vô địch về chạy ở thế vận hội cổ đại. Tài năng chạy của ông ta bị mang ra ví với tài năng chạy của một con Rùa. Khi Iasin đuổi tới chỗ con Rùa hiện tại thì con Rùa đã lại đi được một tí, cứ như thế Iasin lại phải đuổi tới chỗ con Rùa đang ở, rồi cứ như thế mà tạo ra dãy vô hạn có tính chất “có giới hạn”. Vậy những người theo trường phái Acsimet thì cho rằng Iasin sẽ đuổi được kịp con Rùa, và những người phản bác Acsimet thì không. Nói về Acsimet thường mọi người hay nhớ về câu chuyện ông ta cở truồng chạy ra phố reo Ơrêca khi tìm được phương pháp đo thể tích của chiếc ngai vàng mà không phải nấu chảy nó ra. Mọi người ít biết về việc Acsimet là người đầu tiên đưa ra quan niệm về cách đo diện tích của hình tròn bán kính đơn vị. Ông ta dựng ra các hình đa giác đều nội và ngoại tiếp hình tròn, và nhờ việc có thể tính được diện tích của các đa giác nội ngoại tiếp mà Acsimet có được dãy các số để đánh giá cận trên và cận dưới cho diện tích hình tròn đơn vị. Trên thực tế Acsimet đã sử dụng phép dựng hình để dựng ra được hai đoạn thẳng 3 10 71 và 3 10 3 10 71 < ! < 3 10 70 , và chứng minh được 70 . Ông ấy đã làm việc này từ hơn 2250 năm về trước, khi ấy chưa có cách ghi bằng các con số 0; 1; 2; 3; : : : Các con số 3 10 71 hay 3 10 70 chỉ là thứ mà ngày nay chúng ta diễn đạt lại cách dựng hình của Acsimet. Nếu cứ làm tiếp như Acsimet chúng ta sẽ nhận được dãy số đánh giá cận trên và cận dưới diện tích hình tròn bán kính đơn vị. Biên độ chính xác của đánh giá càng sau càng tốt hơn, tiến dần tới 0. Và, như chúng ta vẫn hiểu dãy các cận trên và cận dưới này là “tiến tới !”, và rằng cận trên và cận dưới thì không bao giờ bằng nhau. Vậy là Acsimet đã coi như “tồn tại” thêm một thứ số chỉ có thể dựng ra được dãy cận trên và cận dưới, càng sau càng chính xác hơn, mà có thể không dựng ra được đúng đoạn thẳng nào có độ dài là nó theo kiểu Ơ-cơ-lit. Nhờ vào Acsimet mà chúng ta có thêm các con số, chúng ta gọi chúng chúng là số thực. Chúng được sinh ra nhờ quá trình dựng hình theo Ơ-cơ-lít và lấy giới hạn theo kiểu Acsimet. Nhờ có công cụ giới hạn với số thực của Ascimet mà chúng ta định nghĩa ra tích phân đạo hàm,..thế rồi tìm ra các phương trình vật lý, ra Bigbang,... ra điện ra nền văn minh của chúng ta ngày nay. Lại nói về quá trình Iasin đuổi Rùa, hay là đi từ điểm A tới điểm B. Những ai cho rằng Iasin sẽ đuổi được Rùa là họ theo Acsimet tạo ra trường phái toán học liên tục với công cụ giới hạn. Họ tạo ra vật lý học, rồi điện... Vậy còn những ai không công nhận quá trình đi được từ A tới B, tức về mặt logich Iasin không có cách nào đuổi được con Rùa, họ là môn đệ của trường phái toán học có cấu trúc - đó là tự động hóa, đó là công nghệ thông tin, mạng máy tính, trí tuệ nhân tạo, ... tức là ít nhất cũng bao gồm toàn bộ cái thế giới ảo ngày nay. Một khi nào các bạn về vùng ven biển, các bạn có thể thấy người dân khâu lưới đánh bắt cá. Người ta cần phải chuẩn bị công cụ cho những chuyến đánh bắt. Lịch sử các con số, tức lịch sử của toán học, đó là lịch sử hoàn thiện tấm lưới đó. Chúng ta quăng lưới để đánh bắt các con “cá” - là các quy luật, từ cái biển là thế giới tự nhiên bao la. Đó là việc phát hiện ra các hạt như hạt 8 Tạp chí Epsilon, Số 14, 12/2018 ánh sáng photon, hạt điện tử, các loại phản vật chất,... và các hạt như hạt Quack còn nhỏ hơn cả hạt nhân nguyên tử đến nhiều tỷ lần ... Ơ-cơ-lit đưa ra cách để khai sinh ra các con số, Acsimet đã thêm vào đó các số thực. Số âm ra đời để làm cho phương trình a C x D b luôn có nghiệm. Người ta sử dụng !3 để chỉ nghiệm của phương trình 5 C x D 2. Và điều quan trong là nằm ở chỗ : các nhà toán học đã chứng minh được “Khi thêm số âm vào hệ thống các con số thì chúng ta không tạo ra bất kể một mâu thuẫn nào như khi không có chúng”. Vậy sử dụng số âm hay không sử dụng số âm là quyền của mỗi người. Sử dụng số âm sẽ làm cho quá trình tính toán trở nên đơn giản hơn. Điều này cũng tương tự như việc sử dụng thế giới Âm để giáo dục văn hóa làm người một cách đơn giản hơn. Người Việt quan niệm cõi Âm có tác động quyết định lên cái duyên khiến một sự việc xảy ra. Tín ngưỡng của chúng ta còn cho rằng cõi Âm là sự tiếp tục của sự sống, và mỗi con người vẫn phải chịu trách nhiệm về hành vi khi còn đang sống trên cõi Dương Thế. Tôi cũng luôn thấy mình vẫn là mình vẫn được chở che khi nghĩ đến cõi Âm nơi mà Ba và Mẹ tôi đã đến cùng với ông bà tổ tiên. Nếu sự chấp nhận cõi Âm không bị lợi dụng và không gây ra sự tự mâu thuẫn của các quyết định của “Dương Thế”, thì sự chấp nhận cõi Âm là thuộc phạm trù văn hóa. Nó làm cho cuộc sống có thêm những điều không thể lý giải logich được, làm cho cuộc sống của chúng ta là của những con người. Một số bạn thường vấn vương câu hỏi “Vì sao trừ của trừ lại bằng cộng - tức là !.!3/ lại bằng 3 ?” Câu trả lời nằm ở chỗ các bạn ấy đã cố tình hình dung “trừ” như là nợ, và để rồi nợ của nợ là thứ cực kỳ khó hiểu. Có ai mang tấm lưới đánh bắt cá ra kho mà ăn bao giờ ! Số !3 chỉ là công cụ để thực hiện phép toán, nó không có trong tự nhiên, nó là cái lưới do chúng ta tạo ra, nó không phải là cá. Vậy chúng ta không nên cố gắng hình dung ra !3 phải là gì. Điều mà chúng ta có thể làm là vá lưới, tức mong muốn cái công cụ số của chúng ta phải có được những tính chất sau. 1 - Có 2 phép toán C và " (cộng và nhân), và mọi hai số đều có thể cộng với nhau cũng như nhân với nhau để cho kết quả là số. 2 - Thứ nhất nó có số 1 (tức là cái đoạn thẳng đơn vị thừa hưởng từ thời cố nội 7 nghìn đời ông Ơ cơ lít). 3 - Tiếp theo là số 0 (là mốc đẻ ra nền văn hóa và văn minh mà chúng ta đang tự hãnh diện khi đứng trước con chó). Tiếp theo là các tính chất của phép cộng như 4 - phép giao hoán a C b D b C a: 5 - phép kết hợp .a C b/ C c D a C .b C c/: 6 - phép cộng với 0 thì a C 0 D a: 7 - mọi số a đều có số, được ký hiệu là !a, để cho a C .!a/ D 0: Bây giờ là đến lúc chúng ta trả lời câu hỏi về !.!3/ thì bằng 3: Theo tiên đề 7 thì !.!3/ là một số mà .!3/ C .!.!3// D 0: 9 Thế rồi lại theo tiên đề 7 chúng ta có 3 C .!3/ D 0: Tổng 3 C .!3/ C .!.!3// có 2 cách tính. Tạp chí Epsilon, Số 14, 12/2018 Cách thứ nhất Œ3 C .!3/" C .!.!3// D 0 C .!.!3// D !.!3/: Cách thứ hai 3 C Œ.!3/ C .!.!3//" D 3 C 0 D 3: Do tiên đề 5 về kết hợp và tiên đề 4 về giao hóan, tiên đề 6 về cộng với 0 chúng ta khẳng định !.!3/ D 3: Như vậy không phải “nợ của nợ là được”. Đặc tính !.!3/ D 3 là hệ lụy của việc chúng ta muốn có số âm để tiện trong tính toán mà lại muốn nó có đủ các tính chất thông thường của phép toán. Đối với phép nhân chúng ta mong muốn nó có được các tính chất sau 8 - phép giao hoán a " b D b " a: 9 - phép kết hợp .a " b/ " c D a " .b " c/: 10 - quy tắc hỗn hợp .a C b/ " c D a " c C b " c: 11 - phép nhân với 1 thì a " 1 D a: 12 - mọi số a ¤ 0 thì có một số, ký hiệu là 1=a, mà a " 1=a D 1: Bây giờ là lúc chúng ta trả lời câu hỏi “vì sao bình phương của một số âm lại là một số dương ? Ví dụ .!3/ " .!3/ D 9”. Theo định nghĩa !3 là 3 C .!3/ D 0. Do đó 3 " 3 C 3 " .!3/ D 3 " .3 C .!3// D 3 " 0 D 0; hay 9 C 3 " .!3/ D 0. Mà theo tiên đề 8 thì .!3/ " 3 D 3 " .!3/, nên 9 C .!3/ " 3 D 0: Mặt khác, ta cũng có .!3/ " 3 C .!3/ " .!3/ D .!3/ " Œ3 C .!3/" D .!3/ " 0 D 0: Nhự vậy, .!3/ " 3 C .!3/ " .!3/ D 0 và .!3/ " 3 C 9 D 0, suy ra .!3/ " .!3/ D 9: Như vậy “Bình phương của một số âm là một số dương” không phải do Trời sinh ra nó như thế. Số âm không có, chúng ta chấp nhận cho số âm vào trong hàng ngũ các con số thì nó có những tính chất như vậy. Nó là hệ lụy của việc chúng ta muốn nó tuân thủ các tính chất của phép toán. Vậy không phải thực tế mà chính là phép toán là thứ đẻ ra các con số và cho chúng tính chất! Số âm ra đời là để cho phương trình a C x D b luôn có nghiệm. Số hữu tỷ ra đời là để cho phương trình a " x D b (a ¤ 0) luôn có nghiệm. Và số thực là đời là để cho mọi dãy hội tụ thì “tiến đến giới hạn” và cái giới hạn ấy là số. Tuy nhiên để cho phương trình bậc hai ax2 C bx C c D 0 luôn có nghiệm chúng ta phải cho thêm vào hàng ngũ các con số một loại số nữa. Chúng được gọi là số phức. Số phức được ký hiệu là a C ib và chúng có thể thực hiện các phép toán cộng và nhân như sau : .aCib/C.cCid/ D .aCc/Ci.bCd /; .aCib/".cCid/ D .a"c!b"d /!i.a"d Cb"c/: 10 Tạp chí Epsilon, Số 14, 12/2018 Với định nghĩa như vậy thì số phức có đủ các tính chất từ 1 tới 12 như nêu trên. Và với việc sử dụng chúng thì phương trình ax2 C bx C c D 0 luôn có hai nghiệm là x1 D !b C pb2 ! 4ac 2a ; x1 D !b ! pb2 ! 4ac 2a : Người ta đã chứng minh được việc thêm các số phức vào chỉ có tiện hơn cho tính toán mà không làm nảy sinh ra mâu thuẫn gì như trước khi thêm chúng vào. Vậy là chúng ta có thể sử dụng hay không sử dụng số phức. Việc sử dụng số phức sẽ làm cho việc nghiên cứu dễ dàng hơn, bởi vì đối với số phức các hàm số mũ và hàm lượng giác có cùng bản chất cos x C i # sin x D eix, các hàm logarithm và các hàm lượng giác ngược cũng biểu diễn đơn giản qua nhau. Cũng như việc biểu diễn số thực trên đường thẳng, việc sử dụng số phức ít nhất có lợi là chúng ta có thể coi mỗi điểm của mặt phẳng như là biểu diễn của một số phức. Ở đó phép nhân với số phức cos ˛ C i # sin ˛ D ei˛ chính là phép quay mặt phẳng đi một góc là ˛. Như vậy số phức xuất hiện trong vật lý ở những chỗ nào có dao động, bởi các hàm lượng giác là các hàm số theo góc, và quay đi một góc thì sẽ đơn giản là nhân với một số số nếu sử dụng số phức. Cơ học lượng tử khẳng định chuyển động của một hạt rất nhỏ sẽ có tính chất sóng. Dao động này không phải là dao động của không gian, mà tại mỗi điểm của không gian 3 chiều của chúng ta, khi hạt chuyển động đến sẽ có thêm những chiều nữa nơi chúng sẽ dao động. Vậy là tại mỗi điểm trong không gian, hàm biểu diễn trạng thái của hạt sẽ là hàm số phức theo thời gian. Theo dõi lịch sử phát triển của con số chúng ta thấy các con số có xuất phát điểm là từ các nhu cầu thực tế nhưng về sau nó xuất hiện là do sự hoàn mỹ của công cụ toán học. Thật đáng tiếc mọi đa thức đều có nghiệm trên trường số phức. Như vậy việc sử dụng các phép toán cộng và nhân để mở rộng trường số với đủ 12 tính chất như trên đã không còn có thể. Muốn có thêm các số nữa chúng ta phải giảm thiểu các tính chất mà chúng phải thỏa mãn. Mọi học sinh đều còn nhớ hằng đẳng thức a2 ! b2 D .a C b/.a ! b/. Hẳn không ít bạn nhỏ đã thử sức và không thể khai triển được hệ thức a2 C b2. Vào năm 1878, Clifford đưa ra ý niệm về việc khai triển a2 C b2 thành một bình phương, kiểu như hằng đẳng thức đáng nhớ. Ông ấy muốn có khai triển a2 C b2 D .˛ # a C ˇ # b/2với mọi a và b. Điều này chỉ xảy ra nếu trong khai triển .˛ # a C ˇ # b/2 D .˛ # a C ˇ # b/.˛ # a C ˇ # b/ D ˛2 # a2 C ˇ2 # b2 C .˛ˇ C ˇ˛/ab; thì ˛2 D 1; ˇ2 D 1 và ˛ˇ C ˇ˛ D 0. Lẽ dĩ nhiên các ˛; ˇ không thể là số được, bởi không thể có số mà ˛ˇ D !ˇ˛. Không phải là số thì cũng chả sao, chúng ta sẽ mở rộng chúng để sử dụng với một điều phải được chứng minh là “các ông bạn mới này không gây ra sự phiền toái nào như trước khi chúng ta công nhận chúng!”. Chúng ta gọi chúng là các Clifford. Dirack đã hình dung phương trình vi phân đạo hàm riêng bậc hai @2' @t 2 ! @2' @x2 ! @2' @y2 ! @2' @z2 D 0; ở dạng bình phương phương trình bậc nhất ! @2' @t 2 ! @2' @x2 ! @2' @y2 ! @2' @z2 D ˛ #@'@t C ˇ #@'@x C # #@'@y C $ #@'@z "2: 11 Tạp chí Epsilon, Số 14, 12/2018 Tức là tìm ra các “số” ˛; ˇ; #; $ về thực chất là các ma trận 4 chiều, thỏa mãn A2 ! B2 ! C2 ! D2 D .˛ # A C ˇ # B C # # C C $ # D/2: Khi ấy phương trình có dạng ˛ #@'@t C ˇ #@'@x C # #@'@y C $ #@'@z D 0: Do các số Clifford có thể được hình dung như các ma trận. Với cách hình dung như thế chúng ta mở rộng khái niệm số ra ma trận. Các ma trận được coi là các con số. Chúng ta có thể cộng và nhân ma trận. Chúng ta có thể có được các đa thức với các hệ số là các ma trận, hơn thế nữa chúng ta có thể định nghĩa được hàm số với biến số mà các ma trận. Và do ˛; ˇ; #; $ là các ma trận nên các chiều thêm vào (phần ảo) được Dirack gọi là phản vật chất. Như thế phản vật chất được sinh ra trong quá trình hình dung số dưới dạng các Clifford. Vậy là chúng ta đã có được công cụ số, Chúng ta ký hiệu số thực là R và số phức là C. Bỏ đi điều kiện giao hoán của phép nhân thì chúng ta có thể mở rộng trường số để nó có thêm các số nữa và đó là quanternion. Số quanternion có dạng a C ib C jc C kd trong đó i 2 D j 2 D k2 D !1; k D i # j và i # j D !j # i; i # k D !k # i; j # k D !k # j: Trong trường số này chỉ trừ tính chất giao hoán của phép nhân còn 11 tính chất tích còn lại được thỏa mãn. Như thế tích của các số vẫn có tính kết hợp và mọi số khác 0 thì đều có ngược. Cũng giống như số phức với mặt phẳng, mỗi phép quay trong không gian 3 chiều tương ứng với một số quanternion. Như thế để biểu diễn các trạng thái vật lý mà mỗi điểm liên quan tới sự chuyển đổi của 3 đại lương vật lý thì hàm trạng thái tại một điểm trong không gian sẽ là hàm quanternion theo thời gian. Số là toàn bộ tài sản mà con người có được để nhận biết thế giới. Từ các đường thẳng số chúng ta dựng ra các không gian nhiều chiều hơn, mỗi điểm là bộ các con số được gọi là tọa độ. Những không gian này là trong sự tưởng tượng của chúng ta, không có trong tự nhiên. Nó là công cụ để chúng ta nhận thức. Chúng ta luôn có mong muốn nhận biết tự nhiên. Chúng ta nhìn lên bầu trời và tự hỏi “vạn vật từ đâu mà ra, nó sẽ đi đến đâu?” Ngày nay chúng ta quen với việc tìm đường đi bằng bản đồ trên điện thoại. Chiếc màn hình điện thoại thì không phải là thế gới thực, nó chỉ tương ứng thế giới thực vào với nhận thức. Vậy, chúng ta cũng sẽ hình dung chuyển động một vật trên bầu trời hay trong hạt nhân nguyên tử như một điểm, và bằng một cách nào đó, nó tương ứng “qua lại” với các tọa độ trên màn hình điện thoại 3 chiều của chúng ta, tức vào không gian nhận thức được tạo ra từ các trục số. Tạm thời chưa nói tới việc làm thế nào để có được tương ứng “qua lại” chúng ta bắt đầu bằng câu hỏi cái điểm trên sẽ chuyển động như thế nào trên mô phỏng của chiếc điện thoại 3 chiều, tức trong hệ tọa độ mà chúng ta hình dung ? Do tốc độ viết ra chậm hơn tốc đô tư duy nên hành văn hơi bị lủng củng. Điều này sẽ được chỉnh sửa dần dần. Dành cho các bạn muốn đọc thêm Để hiểu được suy nghĩ của Archimedes (287-212) chúng ta sử dụng bn và an để ký hiệu diện tích của hình đa giác đều 3 # 2n đỉnh nội tiếp và ngoại tiếp hình tròn đơn vị. Đối với ! – diện tích của hình tròn bán kính đơn vị thì bn ! ! ! an. Rõ ràng là bn < bnC1 < ! < anC1 < an: 12 Tạp chí Epsilon, Số 14, 12/2018 Sau đây chúng ta trình bày một phần tính toán của Archimedes (287-212). Các con số mà chúng ta nhìn thấy trong các tính toán dưới đây đã có hơn 2250 năm tuổi. Ký hiệu số thì khác nhiều bởi vì khi ấy người ta chưa biết sử dụng ký hiệu số theo hệ thập phân như của chúng ta ngày nay. Archimedes cho rằng diện tích hình tròn thì nhỏ hơn diện tích đa giác ngoại tiếp và chu vi vòng tròn thì lớn hơn chu vi của đa giác nội tiếp. Archimedes đã tính ra được các yếu tố cần thiết phục vụ cho việc tính chu vi và diện tích của các đa giác đều nội và ngoại tiếp hình tròn dựa vào định-lý Pitago và định-lý “đường phân giác trong thì chia cạnh đối thành hai phần tỷ lệ với độ dài của hai cạnh bên”. Trong tính toán của mình Archimedes đã sử dụng sốp3 nhưng lại không nói tới nó một cách 153< p3 <1351 trực tiếp mà dựa vào ước lượng265 780 : C D I F E A O B AC D p3 >265 Trong tam giác vuông 4AOC có ∠AOC D 30ı, vậyAO Phân giác OD chia cạnh AC theo tỷ lệAD DC D AO 153: D 153 571AO: CO , suy ra AD D AO AC C AO CO AC 780 3013 34: Chu vi vòng tròn ! # AB thì lớn hơn chu vi đa giác nội tiếp 12 # AI , vậy ! >780 " 12 3013 34: Quá trình tính toán tiếp theo cũng tương tự như vậy nhưng dựa trên việc liên tiếp chia đôi góc ∠AOD. Sau 3 lần chia thì Archimedes đã có được các đa giác đều 12; 24; 48; 96 đỉnh với các ước lượng sau. 13 Tạp chí Epsilon, Số 14, 12/2018 780 " 12 3013 34<240 " 24 1838 911<66 " 48 1009 16<66 " 96 2017 14< ! <153 " 96 4673 12<153 " 48 2334 14<153 " 24 1162 18<153 " 12 571 : Do 31071<66 " 96 2017 14<153 " 96 4673 12< 317nên 31071< ! < 317: Sử dụng hàm số lượng giác chúng ta có thể “diễn đạt” lại thuật toán trên như sau. tan ˛ sin ˛ ˛˛ ˛ D ! 3 # 2n!1 Thay x D ! 3 # 2n!1 vào hằng đẳng thức 8ˆˆ< tan xC1 1 ˆˆ: sin x D 1 tan x2 ta được8ˆˆ< 1 3 # 2n!1 # tan ! tanx2 # sin x D 2 # sin2 x2 C1 3 # 2n!1 # sin ! D 2 3 # 2n # tan ! 3"2n!1 ˆˆ: 3 # 2n # tan! 3 # 2n # 3 # 2n!1 # sin! 3"2n!1 # 3"2n 3 # 2n # sin2 x2$2 3 # 2n!1 D Thay an D 3 # 2n!1 # tan! 3 # 2n!1 ; bn D 3 # 2n!1 # sin! 3 # 2n!1 vào công thức trên, ta có 8< : anC1bnD 2 1 anC1 anC1 # bn D b2nC1 dùng để tính giá trị an; bn với a1 D 3p3; b1 D 3p3 2 : Theo dõi các tính toán trên, chúng ta nhận thấy là Archimedes đã ước lượng cận trên của số ! nhờ diện-tích, trong khi ước lượng cận dưới của ! dựa vào chu vi. Hóa ra là, sử dụng cùng một 14 Tạp chí Epsilon, Số 14, 12/2018 đa giác đều nội tiếp, việc tính toán để ước lượng cận dưới của ! thông qua chu-vi sẽ cho kết quả chính xác hơn là thông qua diện tích ! Chi tiết này cho chúng ta hiểu thêm về Archimedes, nhất là vào thời mà việc cộng-trừ-nhân-chia hai số với nhau là việc làm mất nhiều thời gian và rất vất vả. Việc chứng minh ! không phải là số hữu tỷ là do J.H. Lambert tìm ra vào năm 1768, tức là khoảng 2000 năm sau khi Archimedes “tìm được giá trị của số !”. Chứng minh sau đây là của Ivan Niven1. Ivan Niven sử dụng phép phản chứng. Giả sử ! D ab , với a và b là các số nguyên dương. Khi đó, ta đặt nŠ D Xn f .x/ D xn.a ! bx/n kD0 an!k.!b/k xnCk kŠ.n ! k/Š: Khi 0 ! x ! ! thì f .x/ < !nan nŠnên 0 < Z ! 0 f .x/sin xdx < 2!nan nŠ : Do limn!12!nan nŠ D 0 nên 0 < Z ! 0 f .x/sin xdx < 1 với n đủ lớn. ."/ Mặt khác đạo hàm nhiều lần đa thức f .x/, ta thấy ı Khi 0 ! j ! n thì f .j /.0/ D 0: ı Khi n ! j D n C k ! 2n thì f .nCk/.0/ D .n C k/Š nŠ.n ! k/Šan!k.!b/k là các số nguyên. Z ! Ta tính tích phân 0 f .x/sin xdx nhờ công thức tích phân từng phần và kết hợp với nhận xét đạo hàm nhiều hơn 2n lần đa thức f .x/ có bậc 2n thì nó sẽ bằng 0: Z ! 0 f .x/sin xdx D Xn jD0 .!1/j f .2j /.x/ cos xj!0 D Xn jD0 .!1/jC1hf .2j /.!/ C f .2j /.0/i: Do f .! ! x/ D f .x/ nên f .2j /.!/ D f .2j /.0/: Như vậy Z ! 0 f .x/sin xdx D 2Xn jD0 .!1/jC1f .2j /.0/ là số nguyên với mọi n: .""/ Khẳng định ."/ và .""/ mâu thuẫn với nhau, vậy ! không thể là một số hữu tỷ. 1Bull.Amer.Math.Soc. 53(1947), 509. 15 Tạp chí Epsilon, Số 14, 12/2018 ĐỊNH LÝ KHÔNG ĐIỂM TỔ HỢP - COMBINATORIAL NULLSTENLLENSATZ Vaselin Dimitrov (Trần Nam Dũng dịch và giới thiệu) GIỚI THIỆU Từ người dịch bản tiếng Nga. Thường thì hiếm khi một kết quả mới và quan trọng của toán học lại có thể trình bày trên ngôn ngữ mà một học sinh (cho dù là học sinh chuyên) có thể hiểu được. Phương pháp Combinatorial Nullstenllensatz, được tìm ra 10 năm trước (bài viết này được đăng năm 2005 – người dịch bản tiếng Việt) bởi nhà toán học người Do Thái Noga Alon [Nora Alon, Combinatorial Nullstellensatz, Combinatorics, Probability and Computing, 8 (1-2), 7-29.], là một ngoại lệ may mắn. Chúng tôi đăng bài viết của học sinh người Bulgaria Vaselin Dimitrov về các ứng dụng của kỹ thuật đơn giản nhưng rất mạnh này vào các bài toán tổ hợp và lý thuyết số. Ta đưa vào một số ký hiệu cần thiết. Số phần tử của tập hợp S được ký hiệu là |S|. Ta ký hiệu Zn là nhóm các lớp thặng dư theo mô-đun n theo phép cộng; Ký hiệu Fp là trường các lớp thặng dư theo mô-đun nguyên tố p(các lớp thặng dư này thực sự tạo thành một trường, chúng có thể cộng, nhân và chia). Nếu F là một trường thì F[x1, x2, ..., xn] ký hiệu tập hợp các đa thức biến x1, x2, ..., xn trên trường F. Trong một số trường hợp, ta sẽ thực hiệu việc tính toán trong cấu trúc tương ứng (nhóm, trường, vành đa thức) mà không cần phải nói rõ cho từng trường hợp. Ví dụ hệ số nhị thức Ckn vốn là số nguyên dương sẽ được ta xét như phần tử của trường F mà đa thức của chúng ta lấy hệ số. Ví dụ trong F3 ta có (1 + x)3 =1+3x + 3x2 + x3 =1+ x3 bởi vì 3=0. Fedor Petrov Dẫn nhập Mục tiêu của bài báo này là qua ví dụ các bài toán olympiad giới thiệu với bạn đọc kỹ thuật đại số Combinatorial Nullstenllensatz, được đề xuất bởi Noga Alon trong bài viết [1]. Ta đều biết trên một trường bất kỳ đa thức một biến bậc d có không quá d nghiệm. Sự kiện này được sử dụng rộng rãi trong các bài toán olympiad, và các nguyên lý nội suy dựa trên sự kiện này. Combinatorial Nullstenllensatz mở rộng sự kiện này lên đa thức nhiều biến. Nói một cách nôm na thì bản chất của định lý như sau: nếu một đa thức của n biến số bằng 0 trên một hình hộp chữ nhật n chiều đặc biệt nào đó thì đa thức này đồng nhất 0. Từ đây mới xuất phát tên 16 Tạp chí Epsilon, Số 14, 12/2018 gọi Nullstenllensatz, được dịch là định lý không điểm. Một kết quả đơn giản lại là một công cụ mạnh trong nhiều bài toán tổ hợp. Định lý 1 (Combinatorial Nullstenllensatz). Giả sử F - trường và f ∈ F[x1, x2, ..., xn] - là đa thức không đồng nhất 0 có tổng bậc !ni=1 mi, trong đó hệ số của xm1 1 ...xmn n khác 0. Khi đó với mọi các tập hợp S1, S2, ..., Sn ⊆ F với |Si| > mi, 1 ≤ i ≤ n, tồn tại ci ∈ Si sao cho f(c1, ..., cn) ̸= 0. Để chứng minh định lý này ta cần đến hai bổ đề Bổ đề 1. Giả sử rằng f, như một đa thức theo biến xi, có bậc ti, 1 ≤ i ≤ n, và giả sử rằng Si ⊆ F, |Si| > ti. Khi đó nếu f(x1, ..., xn) với mọi bộ (x1, ..., xn) ∈ (S1 × S2 × ... × Sn), thì f ≡ 0. Chứng minh. Dễ dàng chứng minh quy nạp theo n. Bổ đề 2. Giả sử gi = "s∈Si(xi − s), 1 ≤ i ≤ n là đa thức theo biến xi. Nếu đa thức f bằng 0 tại mọi điểm mà tọa độ là nghiệm của các đa thức g1, ..., gn (tức là f(c1, ..., cn)=0 với ci ∈ Si), thì tồn tại các đa thức h1, ..., hn ∈ F[x1, ..., xn] sao cho f = # 1≤i≤n trong đó deg hi ≤ deg f − deg gi với mọi i. Chứng minh. Theo điều kiện của bổ đề higi f(x1, ..., xn)=0 với mọi (x1, ..., xn) ∈ (S1 × S2 × ... × Sn). Đặt ti = |Si| − 1. Với mọi i ta khai triển đa thức gi(xi) theo bậc của xi: gijxj. Khi đó với xi ∈ Si gi(xi) = $ s∈Si i − # (xi − s) = xti+1 1≤j≤ti i = # xti+1 1≤j≤ti gijxj. (1) Gọi f là đa thức thu được từ f bằng cách sau: ta viết f như tổng của các đơn thức và lần lượt thay các xmi i , trong đó mi > ti, bằng các tổ hợp tuyến tính của các bậc nhỏ hơn của xi, sử dụng hệ thức (1). Với mọi i ∈ {1, 2, .., n} đa thức thu được có bậc theo biến xi không vượt quá ti. Chú ý là đa thức f thu được từ f bằng cách trừ đi các đa thức có dạng gihi, trong đó deg hi ≤ deg f − deg gi. Và hơn nữa thì f(x1, .., xn) = f(x1, ..., xn) với mọi (x1, ..., xn) ∈ (S1 × S2 × ... × Sn). Như vậy f(x1, ..., xn)=0 với mọi (x1, ..., xn) ∈ (S1 × S2 × ... × Sn). Theo bổ đề 1, f ≡ 0 . Suy ra f = !ni=1 higi, là điều phải chứng minh. 17 Tạp chí Epsilon, Số 14, 12/2018 Chứng minh định lý 1. Không mất tính tổng quát, giả sử rằng |Si| = ti +1 với mọi i. Giả sử rằng kết luận của định lý không đúng. Khi đó đa thức f bằng 0 tại mọi điểm mà tọa độ là nghiệm của các đa thức g1, ..., gn. Theo bổ đề 2 tồn tại các đa thức h1, ..., hn sao cho f = # 1≤i≤n higi. Theo điều kiện định lý, hệ số của"ni=1 xtii khác 0. Nhưng deg hi ≤ deg f − deg gi theo bổ đề 2 nên bậc của đa thức higi = hi"s∈Si(xi − s) không vượt quá deg f, trong đó tất cả các đa thức i . Như vậy, hệ số của"ni=1 xtii trong mỗi số hạng bậc deg f, có trong higi, đều chia hết cho xti+1 higi, và có nghĩa là trong cả tổng f = !1≤i≤n higi bằng 0. Mâu thuẫn. Các bài toán Bài toán 1 (Định lý Cauchy – Davenport). Với các tập hợp A, B trong một trường số nào đó ta định nghĩa A + B = {a + b|a ∈ A, b ∈ B}. Khi đó với mọi số nguyên tố p và với mọi tập hợp A, B ⊆ Zp ta có |A + B| ≥ min {p, |A| + |B| − 1} . (2) Lời giải. Nếu như |A + B| > p thì khẳng định của bài toán là hiển nhiên: trong trường hợp này A + B = Zp. Giả sử |A| + |B| ≤ p và giả sử ngược lại rằng |A + B| ≤ |A| + |B| − 2. Khi đó trong Zp tồn tại tập hợp C chứa A + B có |C| = |A| + |B| − 2 phần tử. Xét đa thức f(x, y) = $ c∈C (x + y − c). trong Fp[x, y]. Bậc của nó bằng |A|+|B|−2, và hệ số của x|A|−1y|B|−1 bằng C|A|−1 |A|+|B|−2 mod p. Biểu thức này không bằng 0, bởi vì |A| + |B| ≤ p. Theo định lý không điểm tổ hợp, tồn tại a ∈ A và b ∈ B sao cho f(a, b) ̸= 0. Điều này mâu thuẫn với định nghĩa của f. Vậy điều giả sử là sai, nghĩa là |A + B| ≥ |A| + |B| − 1. Định lý Cauchy-Davenport có nhiều ứng dụng. Và ứng dụng nổi tiếng nhất là kết quả kinh điển sau: Bài toán 2 (Định lý Erods-Ginzburg-Ziv). Cho số nguyên dương n. Khi đó từ 2n − 1 số luôn chọn được n số có tổng chia hết cho n. Lời giải. Bằng quy nạp ta có thể đưa bài toán về trường hợp n = p là số nguyên tố (Nếu n = ab và khẳng định đã được chứng minh với a và b thì từ 2n − 1 số, ta có thể lần lượt lấy được 2b − 1 bộ a số (các bộ không giao nhau) sao cho tổng các số của mỗi tổng chia hết cho a. Chia các tổng này cho a rồi áp dụng cho các thương thu được khẳng định đối với b.) Giả sử a1, a2, ..., an là các số nguyên đã cho. Không mất tính tổng quát có thể giả sử 0 ≤ a1 ≤ ... ≤ a2p−1 ≤ p − 1. Ta xét hai trường hợp. 18 Tạp chí Epsilon, Số 14, 12/2018 1. ai+p−1 = ai với i nào đó thuộc {1, ..., p − 1}. Khi đó khẳng định là hiển nhiên vì ta có thể chọn các số ai = ai+1 = ... = ai+p−1. 2. ai+p−1 > ai với mọi i ∈ {1, ..., p − 1}. Trong trường hợp này ta xét các tập hợp Ai = {ai, ai+p−1} , 1 ≤ i ≤ n. Lần lượt áp dụng bài toán 1, ta có |A1 + A2 + ... + Ap−1| ≥ p, tức là A1 + A2 + ... + Ap−1 = Zp. Nói riêng −a2p−1 ∈ A1 + A2 + ... + Ap−1. Như vậy tồn tại các số ci ∈ {ai, ai+p−1}, sao cho c1 + ... + cp−1 + a2p−1 = 0 (trong Zp). Lưu ý rằng định lý Erdos-Ginzburg-Ziv có thể được chứng minh một cách độc lập mà không cần dùng đến định lý không điểm tổ hợp. Chúng tôi dành việc tìm một chứng minh như vậy cho bạn đọc như một bài tập. Bài toán 3. Cho d, n ∈ N và p là số nguyên tố. Khi đó tồn tại các số x1, ..., xd ∈ Z, sao cho xd1 + xd2 + ... + xdd ≡ n( mod p). Lời giải. Ta có thể giả sử rằng d (p − 1)(deg f1 + ... + deg fk). (3) 19 Khi đó hệ Tạp chí Epsilon, Số 14, 12/2018 fi(x1, ..., xn)=0, 1 ≤ i ≤ k, (4) Có nghiệm (x1, ..., xn) ∈ S1 × ... × Sn, khác (0, ..., 0). Chứng minh. Xét đa thức F(x1, ..., xn) = $k i=1 i (x1, ..., xn)) − δ$n (1 − f p−1 j=1 $ s∈Sj\{0} (xj − s), trong đó δ = (−1)|S1|+...+|Sn|−n$n j=1 $ s∈Sj\{0} s−1 ̸= 0 được chọn sao cho F(0, ..., 0) = 0 (ở đây với mỗi s ∈ Fp \ {0} ta ký hiệu s−1 là nghịch đảo của s trong Fp, tức là phần tử s′ duy nhất trong Fp sao cho ss′ = 1). Theo (3), F là đa thức bậc!1≤j≤n(|Sj | − 1), trong đó hệ số của x|S1|−1 1 x|S2|−1 2 ...x|Sn|−1 n bằng −δ ̸= 0. Từ định lý không điểm tổ hợp suy ra tồn tại (c1, ..., cn) ∈ S1 × ... × Sn, sao cho F(c1, ..., cn) ̸= 0. Vì F(0, ..., 0) = 0 nên không phải tất cả các ci đều bằng 0. Nhưng khi đó $n (cj − s)=0, j=1 Từ đó f p−1 $ s∈Sj\{0} i (x1, ..., xn) ̸= 1 với 1 ≤ i ≤ k. Nhưng theo định lý nhỏ Fermat ap−1 = 1 trong trường Fp nếu a ̸= 0 . Suy ra fi(c1, ..., cn)=0 với 1 ≤ i ≤ k. Tức là hệ (4) có nghiệm khác 0 là (c1, ..., cn) ∈ S1 × ... × Sn. Định lý được chứng minh. Khi S1 = ... = Sn = Fp ta được định lý Chevalley-Warning. Và sau đây là hai hệ quả trực tiếp nữa. Bài toán 4 (Troi-Zannier, 1997). Cho k ∈ N và p là số nguyên tố. Giả sử rằng S1, ..., Sn ⊆ {0, 1, ..., p − 1} là các tập hợp có chứa 0 và sao cho !1≤j≤n(|Sj | − 1) ≥ 1 + k(p − 1). Giả sử aji, 1 ≤ j ≤ k, 1 ≤ i ≤ n là các số nguyên bất kỳ. Khi đó tồn tại các phần tử xi ∈ Sin, 1 ≤ i ≤ n, không phải tất cả đều bằng 0, sao cho aj1x1 + aj2x2 + ... + ajnxn ≡ ( mod p) với mọi j ∈ {1, ..., k}. Lời giải. Đây là hệ quả của định lý 2 với fj (x1, ..., xn) = aj1x1 + aj2x2 + ... + ajnxn. 20 Tạp chí Epsilon, Số 14, 12/2018 Với S1 = ... = Sn = {0, 1} ta thu được kết quả kinh điển của Olson, đã từng được sử dụng trong các chủ đề olympic toán (ví dụ trong IMO Shortlist năm 2003, định lý Olson được đề cập như một bài toán số học). Hệ quả (Olson, 1969). Cho k, n ∈ N và p là số nguyên tố, hơn nữa n ≥ 1 + k(p − 1). Giả sử aji là các số nguyên bất kỳ, 1 ≤ j ≤ k, 1 ≤ j ≤ n. Khi đó tồn tại tập con khác rỗng các chỉ số I ⊂ {1, ..., n}, sao cho# i∈I với mọi j = 1, 2, ..., k. aji ≡ 0( mod p) Bài toán 5 (Alon). Cho p là số nguyên tố và G là đồ thị vô hướng, trong đó bậc trung bình của các đỉnh (tức là trung bình cộng của bậc các đỉnh) không nhỏ hơn 2p − 2 và bậc cao nhất của một đỉnh không lớn hơn 2p − 1. Khi đó G có chứa đồ thị con p-đều (tức là có thể bỏ đi một số cạnh và một số đỉnh của đồ thị để bậc của mỗi đỉnh còn lại bằng p). Lời giải. Gọi V và E là tập các đỉnh và các cạnh của G tương ứng. Với mỗi đỉnh v và cạnh e ta đặt av,e = %1, nếu v ∈ e 0, nếu v ̸∈ e. Mỗi một cạnh e ta cho tương ứng với biết xe ∈ Fp. Với mỗi đỉnh v ∈ V ta xét đa thức tuyến tính fv = # e∈E av,exe. Giả thiết bậc trung bình các đỉnh của G không nhỏ hơn 2p − 2 tương đương với bất đẳng thức |E| > (p − 1)|V |, do đó số các biến số xe (bằng |E|) thỏa mãn đánh giá |E| > (p − 1)|V | = (p − 1)×[số các đa thức tuyến tính fv]. Từ đó ta có thể áp dụng định lý 2 cho n = |E| và S1 = ... = Sn = {0, 1}. Thu được điều sau đây: mỗi biến số xe có thể cho tương ứng 0 hoặc 1 sao cho không phải tất cả các phần tử đều bằng 0 và với mỗi một đỉnh, số số 1, được cho tương ứng với cách cạnh có đầu mút là đỉnh đó, là bội của k. Vì mỗi bậc không vượt quá 2p − 1, các cạnh được cho tương ứng với 1 sẽ tạo thành đồ thị con có bậc các đỉnh đều bằng p. Chúng ta sẽ kết thúc câu chuyện bằng một ứng dụng tuyệt vời sau: một chứng minh ngắn và sơ cấp cho giả thuyết Erdos-Heilbronn, vốn là vấn đề mở suốt 30 năm. Năm 1994, đã tìm được một chứng minh rất phức tạp và sau hai năm là chứng minh được trình bày dưới đây. Xin chú ý với độc giả về sự giống nhau của giả thuyết này với định lý Cauchy-Davenport. Bài toán 6 (giả thuyết Erdos-Heilbronn). Cho p là số nguyên tố. Với A, B ⊆ Zp, ta định nghĩa A ⊙ B là tập hợp tất cả các số dư khi chia cho p của các tổng dạng a + b với a ∈ A, b ∈ B và ba ̸=. Chứng minh rằng |A ⊙ B| ≥ min {p, 2|A| − 3} . 21 Tạp chí Epsilon, Số 14, 12/2018 Lời giải. Giả sử |A| = k và m = 2k˘4. Ta chọn phần tử a bất kỳ thuộc A và đặt B = A \ {a}. Dễ dàng kiểm tra được rằng A ⊙ A = A ⊙ A và m = |A| + |B| − 3. Ta cần chứng minh rằng |A ⊙ B| ≥ min {p, m + 1}. Trường hợp 1. m + 1 ≥ p (tức là k ≥ (p + 3)/2). Ta chứng minh trong trường hợp này A ⊙ A = Zp. Xét một phần tử m bất kỳ thuộc Zp. Chia các phần tử của Zp ra thành các cặp có tổng trong Zp bằng m. Ta thu được (p−1)/2 cặp, và trong đó có một phần tử sẽ đi cặp với chính nó. Theo nguyên lý Dirichlet tập hợp A chứa hai phần tử của cùng một cặp, từ đó m ∈ A ⊙ A, là điều phải chứng minh. Trường hợp 2. m + 1 < p. Ta chứng minh rằng trong trường hợp này |A ⊙ A| ≥ m + 1. Giả sử ngược lại. Khi đó tồn tại tập con C của Zp sao cho |C| = m và A ⊙ B ⊆ C. Xét đa thức f(x, y)=(x − y)$ c∈C (x + y − c) trên Fp[x, y]. Đa thức này có bậc m +1 = (k − 1) + (k − 2), trong đó bậc của xk−1yk−2 bằng Ck−2 2k−4Ck−1 2k−4 ̸= 0 trong Zp (hãy chứng minh điều này!), bởi vì 2k − 4 = mk − 1 và |B| = k − 1 > k − 2 nên áp dụng định lý không điểm tổ hợp, tồn tại x ∈ A, y ∈ B sao cho f(x, y) ̸= 0. Mâu thuẫn vì theo định nghĩa của f, f(a, b)=0 với mọi a ∈ A, b ∈ B. Kết luận Sự phong phú và phức tạp của các ví dụ đã trình bày nói trên nói lên tính hiệu quả của phương pháp Combinatorial Nullstenllensatz, đã được thừa nhận là mang tính cách mạng trong lý thuyết số cộng tính. Phép chứng minh đẹp đẽ “như sách giáo khoa” của một kết quả nổi tiếng, như bài toán 6, đã minh chứng cho sự kiện là cách tiếp cận đa thức là một cách tiếp cận tự nhiên trong các bài toán dạng như vậy. Để so sánh, chứng minh ban đầu của Da Silva và Hamidon năm 1994 dài hơn một cách đáng kể và sử dụng các kết quả của đại số tuyến tính và lý thuyết biểu diễn các nhóm đối xứng. Bài tập Bài toán 7. Cho p là số nguyên tố và G là đồ thị trong đó có không ít hơn 2p − 1 đỉnh. Chứng minh rằng tồn tại một tập con U khác rỗng các đỉnh của G, sao cho số các cạnh của G, có ít nhất một đầu mút thuộc U, chia hết cho p. Bài toán 8 (Alon). Cho H1,...,Hm là họ các siêu mặt phẳng trong Rn, phủ kín tất cả các đỉnh của hình lập phương đơn vị {0, 1}n ngoại trừ 1 điểm. Chứng minh rằng m ≥ n. (Siêu mặt phẳng trong Rn là tập hợp các điểm (x1,...,xn) ∈ Rn, thỏa mãn phương trình dạng a1x1 + ... + anxn = b.) Bài toán 9 (Giả thuyết Sneville, được chứng minh bởi Alon). Cho p ≥ 3 là số nguyên tố và giả sử A = {a1, ..., ak} ⊆ Zp và B = {b1, ..., bk} ⊆ Zp là các tập con k phần tử của tập hợp các số dư khi chia cho p (các phần tử trong mỗi tập hợp A và B phân biệt). Chứng minh rằng tồn tại 22 Tạp chí Epsilon, Số 14, 12/2018 hoán vị (s1,...,sk) của tập hợp (1,...,k) sao cho các phần tử ai + bsi ∈ Zp, 1 ≤ i ≤ k, đôi một khác nhau. 23 Tạp chí Epsilon, Số 14, 12/2018 TOÁN HỌC VÀ ẢO THUẬT Nguyễn Hùng Sơn GIỚI THIỆU Hãy tưởng tượng chúng ta đang có mặt tại buổi biểu diễn của một nhà ảo thuật gia nổi tiếng. Ông rút trong túi ra một bộ bài gồm 32 quân và mời 5 người bất kỳ lên tham gia buổi biểu diễn và ngồi ở 5 cái ghế đã được chuẩn bị sẵn. Trước tiên ông mời 1 khán giả lên tráo bài (bốc một phần trên của bộ bài và cho xuống dưới bộ bài). Sau đó ông chia cho 5 người chơi 5 quân bài liên tiếp và đề nghị họ phải nhìn kỹ quân bài của mình nhưng giấu kín không cho ai biết. Nhà ảo thuật nói với 5 người chơi: - Đề nghị các bạn chỉ dùng thần giao cách cảm để gửi cho tôi thông tin về con bài của các bạn. Một lúc sau nhà ảo thuật lại nói: - Ở đây có quá nhiều người nên tôi không thể phân biệt được các thông tin mà các bạn gửi cho tôi. Bây giờ tôi đề nghị những bạn đang cầm các quân bài màu đỏ hãy đứng lên và thử truyền tin cho tôi một lần nữa. Lúc này có 2 người ngồi đầu dãy đứng lên và chỉ sau vài giây nhà ảo thuật gia đã đọc các lá bài của 2 người đó là ♥8 và ♦A. Ông còn đọc được chính xác các lá bài (màu đen) của 3 người đang ngồi. Trò ảo thuật này hoàn toàn không cần đánh dấu bài, không cần cài người quen vào các người chơi hoặc các khả năng phi thường nào hết. Chúng ta hãy tìm hiểu lý thuyết toán học ẩn đằng sau trò ảo thuật này. 1. Dãy số De Bruijn Một số bạn có thể đã đoán ra rằng chìa khóa là các quân bài được sắp xếp theo thứ tự đặc biệt. Tuy nhiên, điều này không giải thích làm thế nào các nhà ảo thuật có thể đọc tên chính xác năm quân bài trên cơ sở thông tin về màu sắc của chúng. Để giải quyết điều đó, chúng ta phải tìm hiểu một khái niệm "huyền bí", được gọi là các dãy số De Bruijn. Trước hết chúng ta hãy nhắc lại một số định nghĩa cơ bản sau đây. Dãy số nhị phân (dãy nhị phân) là các dãy chỉ chứa 2 ký tự duy nhất là 0 và 1. Mỗi phần tử của dãy nhị phân ta sẽ gọi là bít. Một dãy số gồm n bít liên tiếp của một dãy A = (an) cho trước sẽ được gọi là dãy con có độ dài n của dãy A. Dãy số hữu hạn A = (a1, ..., ak) được gọi là dãy vòng nếu các bít của nó được đặt lên một đường tròn như hình sau đây 24 Tạp chí Epsilon, Số 14, 12/2018 Hình 1: Dãy vòng A = (1010000110010111) Định nghĩa 1 (dãy số De Bruijn bậc n) Dãy số nhị phân B(n) được gọi là dãy số De Bruijn bậc n nếu mọi dãy nhị phân độ dài n đều xuất hiện đúng một lần trong dãy vòng B(n) (ở dạng một dãy con có độ dài n). Ví dụ B(2) = (0011) vì mọi dãy nhị phân có độ dài 2 đều xuất hiện trong dãy vòng B(2). Ta cũng dễ dàng kiểm tra rằng B(3) = 01000111. Chúng ta dễ dàng nhận ra rằng dãy số De Bruijn bậc n chứa đúng 2n số hạng. Bạn đọc có thể tham khảo thêm các thông tin về dãy Bruijn cũng như thuật toán xây dựng dãy B(n) với mọi giá trị của n tại trang web [1] 2. Cơ sở toán học Các nhà ảo thuật thực sự không bao tiết lộ bí quyết của họ. May mắn thay cho bạn đọc, tôi không phải là một nhà ảo thuật mà là một nhà toán học, vì vậy nhiệm vụ của tôi là phải tiết lộ bí mật cho bạn. Trong trò ảo thuật này, chúng ta sẽ sử dụng dãy số Bruijn B(5). Dãy số đó như sau: B(5) = 00001001011001111100011011101010. Có một cách đơn giản để tạo ra dãy này như sau: chúng ta bắt đầu với 5 bít 00001 và bít tiếp theo được tính bằng cách cộng bít thứ nhất và bít thứ ba (modulo 2): 0+0, ta được 0. Chúng ta thêm kết quả vào cuối dãy, và nhận được 000010. Sau đó chúng ta di chuyển sang phải, nghĩa là chúng ta quên bít ở đầu và lặp lại bước này (tức là ta xét dãy con gồm 5 bít cuối cùng 00010). Tương tự như trước, chúng ta cộng bít thứ nhất và bít thứ ba (modulo 2): (0 + 0) mod 2 = 0, sau đó thêm bít này vào cuối dãy. Sau bước này, dãy số có dạng 0000100. Chúng ta chuyển sang bên phải một lần nữa (tức là chúng ta bỏ qua hai bít đầu tiên), vì vậy chúng ta xét dãy 5 bít cuối 00100. Lần này, tính tổng modulo 2 của bít đầu tiên và thứ ba chúng ta nhận được 1. Sau khi thêm bít này vào cuối dãy ta được 00001001. Tiến hành theo cách này, chúng ta sẽ có được chuỗi nhị phân De Bruijn bậc 5 như đã viết ở trên. Chính xác hơn, dãy B(5) = a1a2 ...a32 thỏa mãn tính chất truy hồi như sau: a1a2a3a4a5 = 00001 ak = ak−5 + ak−3 mod 2 với k = 6, ..., 32 25 ♠ ♣ ♦ ♥ 00 01 10 11 ab = cde = Tạp chí Epsilon, Số 14, 12/2018 A 2 3 4 5 6 7 8 001 010 011 100 101 110 111 000 Hình 2: Cách mã hóa chuỗi nhị phân gồm 5 bít abcde bằng các lá bài. Để sử dụng dãy B(5) cho trò ảo thuật, mỗi quân bài trong bộ bài 32 quân sẽ được mã hóa bằng một dãy nhị phân abcde có chiều dài bằng 5 theo nguyên tắc: dùng 2 bít đầu tiên (ab) để mã hoá bốn chất ♥, ♦, ♣, ♠ (cơ, rô, nhép, bích) và dùng 3 bít tiếp theo (cde) để mã hóa giá trị của quân bài : át (một), 2, 3,4,5,6,7,8 như trong Hình 2. Bạn đọc có thể nhận thấy rằng (cde) chính là 3 chữ số cuối của các số từ 1 đến 8 khi chúng được viết ở dạng nhị phân. Ta cũng biết rằng nếu ký hiệu B(5) = a1a2 ...a32 thì các dãy con dạng akak+1ak+2ak+3ak+4 của B(5) (với k = 1, 2, ..32, và a33 = a1, a34 = a2...) tạo thành tập hợp tất cả 32 dãy nhị phân có chiều dài bằng 5. Chúng ta để ý rằng nếu tương ứng với bít ak tại vị trí thứ k của dãy B(5) ta đặt quân bài có mã số là akak+1ak+2ak+3ak+4 thì ta sẽ được một cách sắp xếp các quân bài như ở Hình 3. Cách sắp xếp bài như thế này thỏa mãn đồng thời 2 điều kiện: 1. Các quân bài màu đỏ luôn ở vị trí tương ứng với bít có giá trị bằng 1, các quân đen ở các bít có giá trị bằng 0. 2. chỉ cần biết 5 bít là chúng ta biết chính xác vị trí của chúng ở trong dãy de Bruijn. 0 0 0 0 1 0 0 1 0 1 1 0 0 1 1 1 1100... ♠A ♠2 ♠4 ♣A ♦2 ♠5 ♣3 ♦6 ♣4 ♥A ♦3 ♠7 ♣7 ♥7 ♥6 ♥4 1 1 0 0 0 1 1 0 1 1 1 0 1 0 1 0 0000... ♥8 ♦A ♠3 ♠6 ♣5 ♥3 ♦7 ♣6 ♥5 ♥2 ♦5 ♣2 ♦4 ♣8 ♦8 ♠8 Hình 3: Cách biểu diễn dãy B(5) bằng 32 quân bài 3. Vén màn bí mật Đến đây thì có lẽ bạn đọc đã biết toàn bộ nền tảng của điều kỳ diệu là gì. Chúng ta hãy phân tích lại một lần nữa trò ảo thuật được giới thệu ở đầu bài báo: có 5 người chơi, hai người đầu tiên có quân bài màu đỏ, ba người còn lại có quân bài màu đen. Chúng ta sẽ ghi lại màu đỏ là 1, màu đen là 0, và vì vậy, chúng ta nhận được dãy nhị phân 11000. Dãy này chính là mã số nhị phân của quân bài trong tay người đầu tiên: 11 có nghĩa là đó là con cơ, còn 3 bít 000 ở cuối có nghĩa là số 8. Quân bài trong tay người đâu tiên chính là ♥8. Để tìm ra con bài trong tay người thứ hai là gì, chúng ta cộng các bít đầu tiên và thứ ba theo modulo 2 (1 + 0 = 1), và ghi vào cuối dãy. Dãy số gồm 5 bít cuối cùng của chúng ta hiện nay là 10001. Mã 10 là rô, trong khi 001 có nghĩa là A. Vì vậy, người chơi ở vị trí thứ hai đang giữ trong tay con ♦A. 26 Tạp chí Epsilon, Số 14, 12/2018 Tiếp tục cách này, chúng ta sẽ thấy rằng ba con bài của 3 người tiếp theo sẽ lần lượt là: ♠3, ♠6, ♣5. Bạn hãy thử tập làm và biểu diễn trò ảo thuật này cho bạn bè và người thân của mình! 4. Các ứng dụng khác Ảo thuật không phải là lĩnh vực duy nhất trong các ứng dụng của dãy số de Bruijn. Một số tính chất quan trọng của các dãy số de Bruijn như: "không có dãy con nào xuất hiện nhiều lần", khiến cho chúng trở nên cực kỳ hữu ích trong việc phát triển mã hóa. Lưu ý rằng mọi dãy con có độ dài k đều tương ứng với một ký tự khác nhau của thông điệp được mã hóa! Đây chính là cơ hội để những người làm bảo mật có thể sử dụng trong việc mã hóa văn bản. Ma trận de Bruijn là một cách cách mở rộng của dãy de Bruijn cho bảng hai chiều và đã được sử dụng rộng rãi trong ngành Computer Vision (thị giác máy tính). Hãy tưởng tượng một robot công nghiệp di chuyển trên hành lang theo các hướng khác nhau. Tuy nhiên, chúng ta muốn robot có thể xác định chính xác vị trí của nó. Một nhà toán học nhận thấy rằng có thể coi mặt phẳng dưới robot như là ma trận de Bruijn. Nhờ tính chất: mỗi ma trận con chỉ xuất hiện duy nhất một lần, robot có thể xác định chính xác vị trí của nó. Cũng dựa trên nguyên tắc này, công ty Anoto đã thiết kế và đưa ra thị trường một cây bút kỹ thuật số. Để có thể sử dụng nó, bạn cần một tờ giấy đặc biệt được in với các chấm sao cho chúng tạo thành ma trận de Bruijn. Máy ảnh được đặt ở đầu cây bút sẽ đọc cửa sổ của ma trận xung quanh đầu bút và trên cơ sở này nó có thể xác định vị trí của nó trên một trang giấy. Chính vì các ứng dụng thực tế kể trên và do tính chất độc đáo của chúng, các bài toán liên quan đến dãy và ma trận de Bruin cũng là chủ đề của nhiều nghiên cứu lý thuyết. Tài liệu [1] https://en.wikipedia.org/wiki/De_Bruijn_sequence Nicolaas Govert de Bruijn Ông Nicolaas Govert (Dick) de Bruijn ( sinh ngày 9 tháng 7 năm 1918 mất ngày 17 tháng 2 năm 2012) là một nhà toán học người Hà Lan, được biết đến bởi nhiều đóng góp trong các lĩnh vực giải tích, lý thuyết số, tổ hợp và logic. Ông sinh ra ở The Hague, nhận bằng thạc sĩ toán học tại Đại học Leiden vào năm 1941. Ông bảo vệ tiến sĩ năm 1943 ở Vrije Universiteit Amsterdam (Đại học tổng hợp tự do Amsterdam) với luận án nhan đề "Over modulaire vormen van meer veranderlijken" (tiếng Hà Lan, tạm dịch: Về các dạng modular nhiều biến). 27 Tạp chí Epsilon, Số 14, 12/2018 De Bruijn bắt đầu sự nghiệp nghiên cứu giảng dạy của mình tại Đại học Amsterdam, nơi ông là giáo sư toán học từ năm 1952 đến năm 1960. Năm 1960 ông chuyển đến trường Đại học Kỹ thuật Eindhoven và làm giáo sư toán học ở đó cho đến lúc nghỉ hưu vào năm 1984. Vào năm 1957 ông được bầu làm thành viên của Viện hàn lâm Khoa học và Nghệ thuật Hoàng gia Hà Lan. Ông được phong danh hiệu "Hiệp sĩ Sư tử" của Hà Lan. 28 Tạp chí Epsilon, Số 14, 12/2018 THẾ NÀO LÀ TƯ DUY LOGIC - HÀNH TRÌNH ĐI TÌM MÁY BAY MẤT TÍCH MIG-21U Ban Biên tập Tạp chí Epsilon GIỚI THIỆU Ngày 14 tháng 3 năm 2018, đài truyền hình Việt Nam (VTV) phát sóng một phóng sự về cuộc hành trình tìm kiếm máy bay MiG-21U thuộc trung đoàn Không quân 921 mất tích từ tháng 4 năm 1971. Đến đầu tháng 10 năm 2018, nhiều vật dụng cá nhân thuộc về liệt sĩ phi công Công Phương Thảo và huấn luyện viên, đại úy không quân Liên Xô Poyarkov Yuri Nikolaevich đã được tìm thấy và xác nhận bởi các cơ quan có thẩm quyền. Một kỳ tích đã được tạo nên nhờ vào một chuỗi những suy luận và dự đoán logic, vào kinh nghiệm, trình độ và tấm lòng của những con người nhân hậu kết hợp với sự giúp đỡ từ cộng đồng và Internet. Một câu chuyện cổ tích có thật, và có hậu. Chi tiết của cuộc tìm kiếm này bạn đọc quan tâm có thể đọc và xem lại ở VTV cũng như các báo trong thời gian đó. Trong Epsilon số 14 này, Ban Biên tập Epsilon muốn đem lại với bạn đọc một góc nhìn khác, những thông tin khác từ câu chuyện kỳ diệu này: Epsilon muốn kể lại chuỗi những suy luận logic của quá trình làm việc trung thực và suy diễn rất hệ thống để giải quyết được một vấn đề "không thể tin nổi". Cá nhân chúng tôi vẫn luôn tin rằng, học toán là học cách suy luận, học văn là học làm người. Vì thế chúng tôi hi vọng câu chuyện này sẽ góp thêm một phần vào chủ đề chung "Học toán để làm gì"của tạp chí số này. Để viết bài này, chúng tôi vinh dự được sự đồng ý của thầy Nguyễn Lê Anh, một nhà toán học, một cựu giảng viên, và là người thúc đẩy cho vụ tìm kiếm này như ông vẫn luôn khiêm tốn nói về vai trò của mình như vậy. Chúng tôi đăng bài này dựa theo nguyên bản bài viết của ông trên Facebook: "Thế nào là tư duy Logich"đăng ngày 5 tháng 3 năm 2018. Chúng tôi có kết hợp thêm một số thông tin tìm được qua báo chí, qua cuộc gọi điện ngắn gọn nhưng rất chân tình với ông để bổ sung và giải thích vào những chỗ vốn khó có thể diễn giải đầy đủ chỉ với một bài ngắn trên Facebook. Xin được mở đầu cho câu chuyện bằng chính lời viết của ông: "Khi còn nhỏ tôi có được đọc về một nhà chiêm tinh tướng số nổi tiếng về dự báo các sự kiện. Ông ta bị hỏi dự đoán chính xác ngày chết của mình, để rồi đến ngày ấy ông ta đành phải "tự tử"để lưu danh tài năng. Điều này thì chẳng hay ho gì, và vì thế tôi không thấy vui mừng về việc tìm thấy chiếc MiG-21U mất tích, bởi không có sự việc này thì vui hơn. Cái vui nhất là sự trung thực của quá trình tư duy và làm việc. Chỉ những ai muốn biết và muốn có được phép lạ biến "từ không thành có"ấy thì nên đọc tiếp." 29 Tạp chí Epsilon, Số 14, 12/2018 Những suy luận ban đầu Ngày 25 tháng 9 năm 2017, trên Facebook (FB) của một người lấy tên là Nam Nguyen có một bài viết như sau: KHÔNG AI BỊ QUÊN LÃNG, KHÔNG GÌ ĐƯỢC LÃNG QUÊN Cô gái Nga Anna Poyarkova - cháu gái của một sỹ quan Xô viết đã mất tích tại Việt Nam năm 1971 đang nỗ lực tìm dấu vết người ông của mình. Câu chuyện như sau: Poyarkov Yuri Nikolaevich là đại úy không quân của Liên Xô sinh năm 1933, đảng viên từ 1961, trung đoàn phó không quân của đơn vị với mã số в\ч06858 làm nhiệm vụ ở Việt Nam với vai trò phi công huấn luyện. 30/04/1971 trong một chuyến bay tập máy bay của ông đã bị rơi vào rừng rậm. Cả máy bay, cả thi thể người phi công đều không được tìm thấy, và từ đó đại úy Poyarkov được coi là mất tích. Tình hình còn phức tạp hơn bởi sự hiện diện của những chuyên gia Liên Xô thời bấy giờ không được loan báo nhiều, ngay cả trong Bộ Quốc phòng CCCP, kể cả nhiều năm sau. Và thế là gia đình nay chỉ còn có được những kỷ vật sau: - những tấm ảnh của ông Yuri Poyarkov - bằng khen, huân huy chương của Thủ tướng, Bộ Quốc phòng nước Việt Nam Dân Chủ Cộng Hòa. Riêng huân chương “Đoàn Kết” được thứ trưởng Bộ QP ký tặng 3 tháng sau khi ông hy sinh. - thẻ chấm công của đại úy Poyarkov có ghi rằng ông giữ chức vụ “phi công huấn luyện để đào tạo phi công cho không quân Việt Nam bay ban ngày và bay đêm trong các điều kiện khí tượng đơn giản và phức tạp” – và “hy sinh trong khi làm nhiệm vụ bởi một tai nạn bay” Xin ghi nhớ là ở thẻ này ghi rõ “hy sinh” chứ không phải “mất tích”. Thế nhưng thi hài ông không thấy được đưa về nước, còn thân nhân thì được báo tin rằng “mất tích”! Vậy điều gì đã xảy ra với đại úy Poyarkov vậy? Gia đình có mấy câu hỏi: Ông mất tích trong hoàn cảnh nào? Đó là buổi bay tập hay trận không chiến? Đoàn bay của ông đóng ở đâu? Máy bay của ông rơi ở khu vực nào? Tại sao không thể tìm ra chiếc máy bay rơi, và thi thể của phi công? Đại úy được chôn cất cẩn thận ở Việt Nam hay bây giờ xác ông và chiếc máy bay vẫn còn trong rừng sâu? Con cháu ông vẫn còn một hy vọng, dù là rất mong manh, rằng một điều kỳ diệu nào đó đã xảy ra, và ngày nay ông Poyarkov với tuổi 84 vẫn còn sống đâu đó trong một làng bản ở Việt Nam. Hoặc không thì họ cũng muốn biết – và hoàn toàn có quyền được biết – cha ông của họ đã ngã xuống như thế nào, không lẽ đã 46 năm trôi qua mà tại đây vẫn chưa tìm ra chiếc máy bay? Ông Poyarkov đã chiến đấu cho tất cả chúng ta, vậy nên có lẽ Việt Nam nợ gia đình ông một câu trả lời thỏa đáng. Bởi không ai phải bị quên lãng và không có điều gì có thể lãng quên... Bên dưới bài viết là các hình ảnh còn lưu giữ của gia đình Pyyarkov và hai ghi chú của Nam Nguyen, ghi chú thứ nhất là lời kêu gọi bạn bè và cộng đồng trên FB và "Ghi chú 2: trên trang 30 Tạp chí Epsilon, Số 14, 12/2018 của Sergey đã xác định được, có lẽ ông Poyarkov cùng bay huấn luyện và hy sinh cùng với phi công Công Phương Thảo tại vùng trời Tam Đảo... Vẫn cần thêm thông tin!" Nói theo ngôn ngữ của khoa học, thì "bài toán"có thể tóm tắt như sau: thông tin đầu vào: máy bay bay tập của phi công Công Phương Thảo và thầy dạy Poyarkov bị mất tích vào ngày 30/4/1971 ở vùng Tam Đảo, thông tin cần tìm: máy bay rơi ở khu vực nào, liệu họ có còn sống, nếu không, thi thể ở đâu ... Ngay khi đọc được bài trên, ông Nguyễn Lê Anh viết: "Tôi cảm thấy rất nhục khi một chiếc máy bay mất tích không tìm thấy ở một nơi cách trung tâm Hà Nội không quá 70km. Những suy nghĩ ấy kích hoạt quá trình tư duy của tôi. Tôi đã viết mấy còm như sau: 1. Cần phải tra vào hồ sơ liệt sĩ "Cống Phương Thảo"1 để tìm thân nhân. Chắc sẽ có thông tin tốt. 2. Cần phải hỏi thân nhân để tìm ra nơi có mộ của "Cống Phương Thảo". Nếu có mộ thật tức là người nhà biết được bối cảnh hy sinh. 3. Trong vùng núi Tam Đảo người dân có nói về sự kiện máy bay Mỹ rơi vào khu vực núi. Người Mỹ đã tới kiểm tra và đã xác minh đúng. Không thấy người dân nói về MiG bị rơi vào núi trong dãy Tam Đảo. 4. Vào thời điểm ấy ở vùng núi Tam Đảo chỉ có người dân tộc sinh sống. Nên hỏi thì may ra mới có thông tin. 5. Cách duy nhất là nhờ chính quyền xã toàn miền núi phía Bắc hỏi người già về khả năng máy bay rơi. Đi tìm trực tiếp không dễ vì mưa gió đã làm mất hết dấu vết. Đấy là khởi đầu của quá trình suy diễn dựa trên tư duy logich khi biết thông tin "ông Poyarkov cùng bay huấn luyện và hy sinh cùng với phi công Công Phương Thảo."Lẽ dĩ nhiên không thể có thành công dễ như vậy. Chắc chắn không thể có thông tin gì về mộ của anh Công Phương Thảo, nhưng không thể bỏ qua mà không kiểm tra. Tôi đã tìm cách hỏi thông tin về gia đình Công Phương Thảo cũng như hỏi người dân khu vực Tam Đảo về tung tích các máy bay bị rơi." Trao đổi thêm với Epsilon, ông cho biết nhiều người thắc mắc vì sao không tra cứu lịch trình bay, sao không kiểm tra lại hệ thống định vị hay giáo trình tập huấn bay để có thêm thông tin. Đơn giản vì hoàn toàn không có ai lưu giữ lại các thông tin đó. Chúng tôi tạm nhắc lại bối cảnh tai nạn xảy ra vào tháng 4 năm 1971, và đó là những thời điểm khốc liệt của chiến tranh (tháng 2/1971 với chiến trường đường 9 Nam Lào, tháng 8 với trận lụt lịch sử ở sông Hồng, tháng 12 khi miền Bắc liên tục bị đánh bom ... ), gần như mọi thông tin và hoạt động đều rất hạn chế. Tính theo sức người, sức của và kể cả tiềm lực đều khó có thể tiến hành dễ dàng vào thời điểm đó. Mặc dù khó khăn như vậy, có ít nhất 2 cuộc tìm kiếm đã được thực hiện ngay sau tai nạn, nhưng bất thành. Chúng tôi cũng muốn nói thêm rằng với sức lan toả của Internet, thông tin đến rất nhanh từ mọi người, nhưng với một sự kiện đã xảy ra gần một nửa thế kỷ, độ tin cậy của các 1Ở thời điểm này, họ của phi công vẫn bị ghi là Cống, cho đến khi có một người khác ghi chú là: "Ở Phú Thượng chỉ nghe có dòng họ Công, không có họ Cống đâu."thì về sau mới xác định được họ là Công. Tất cả các chú thích trong bài đều của Epsilon. 31 Tạp chí Epsilon, Số 14, 12/2018 tin từ FB cũng là một vấn đề không đơn giản để quyết định đâu là thông tin đúng, và đúng đến ... bao nhiêu. Một bài toán quá khó để giải! Ông Nguyễn Lê Anh viết tiếp: "Để ước lượng được vị trí của chiếc MIG21 xấu số, tôi đi xác định các nguồn có thể thu lượm thông tin. Tôi đi đến kết luận nguồn có thể cung cấp các câu trả lời tốt là: về hiện trường chỉ có thể đến từ người dân tộc, và về thân nhân anh Công Phương Thảo chỉ có thể hỏi qua các phi công cùng đơn vị. Tôi quen Trung tướng Phạm Tuân, cựu Phó Tư lệnh Chính trị Quân chủng Không quân, anh hùng phi công vũ trụ. Ngày 26 tháng 9 năm 2017 tôi liên hệ với anh Phạm Tuân. Tôi tập trung vào xác minh những vấn đề khó mà có thể quên sau nhiều năm. - Có hay không sự kiện chiếc MIG 21 rơi như vậy. - Những ai có thể cung cấp thông tin tin cậy về vụ việc (anh Phạm Tuân, anh Quang, anh Khánh Duy đại tá phi công cùng trung đoàn 921 với anh Công Phương Thảo.) - Điều kiện thời tiết bay hôm ấy thế nào? ("thời tiết đơn giản- tức độ nhìn xa trên 10km, trời trong không mây, không mưa, không gió mạnh) - Quỹ đạo bay và thời gian bay của chiếc MIG 21 ngày 30-4-1971 thế nào? Tôi hỏi độc lập các anh Phạm Tuân, anh Quang, anh Nguyễn Khánh Duy để so sánh kiểm chứng và đưa ra kết luận." Trong mỗi cuộc phỏng vấn ông luôn ghi âm lại và sau khi được sự đồng ý của các cá nhân được phỏng vấn, các cuộc phỏng vấn này cũng đã được đăng lên trên FB của ông. Ông cũng có trò chuyện thêm với Epsilon vì sao mình lại quyết tâm như vậy. Ông kể trung tướng Phạm Tuân có nói với ông: "Chú mày làm gì mà hỏi những cái chi tiết ấy làm gì?" "Không tìm được! Tao đã bảo rồi, chả nhẽ toàn quân không đi tìm lại để cho một người đi tìm. Sao mà tìm được!" Và ông trả lời: "Thế hệ các anh lớn rồi, không còn đi nổi. Còn thế hệ bọn nhỏ, sẽ không ai quan tâm nữa. Nếu không phải tụi em làm thì ai sẽ làm đây!" Ông viết tiếp: Sau khi làm việc nhiều lần với các anh phi công: anh Phạm Tuân, anh Quang, anh Nguyễn Khánh Duy (phi công cùng đơn vị bay với anh Công Phương Thảo) và anh Công Văn Mão (người thờ cúng anh Công Phương Thảo), tôi tạm đưa ra nhận định sau. Các thông tin tin cậy (từ ghi âm cuộc nói chuyện): 1. Anh Phạm Tuân khẳng định lượng xăng của máy bay chỉ đủ bay trong 30 phút, sự cố xảy ra vào khoảng từ 10 giờ tới 12 giờ sáng ngày 30/4/1971. Thời tiết tốt. 2. Anh Nguyễn Khánh Duy (phi công cùng đơn vị bay với anh Công Phương Thảo) khẳng định "Công Phương Thảo xin phép bay về". 3. Máy bay thực hiện bài bay độ cao trung bình (từ 2000m tới 6000m) cất cánh theo hướng Đông Nam bay tới khu vực Đại Từ, như trong hình vẽ. Khi bay về phải tới được điểm phía nam Phúc Yên nơi có đài chỉ huy, cách sân bay chừng 10km rồi hạ dần độ cao xuống đường băng. 32 Tạp chí Epsilon, Số 14, 12/2018 4. Tất cả các anh Phạm Tuân và Nguyễn Khánh Duy đều khẳng định bộ tư lệnh Không Quân và chính quyền các cấp đã tổ chức tìm kiếm ngay nhưng không thấy. 5. Mặc dù các anh Phạm Tuân và anh Khánh Duy không khẳng định nhưng nhiều lần nói tới khả năng chiếc MIG 21 rơi ở phía Tam Đảo Bắc. Như vậy sau các cuộc phỏng vấn, thông tin đầu vào đã có nhiều hơn, nhưng vẫn còn rất mơ hồ. Chúng tôi tạm tóm tắt lại qua hình 1. Hình 1: Bản đồ tóm tắt thông tin có được sau các cuộc phỏng vấn với các phi công: máy bay xuất phát từ sân bay Đa Phúc (nay là sân bay Nội Bài) ở điểm số 1, máy bay sẽ bay đến vùng Đại Từ (vùng màu đỏ số 2), ở độ cao trung bình từ 2000m tới 6000m. Máy bay sau đó bay thẳng về đài chỉ huy ở Nam Phúc Yên (điểm số 3) và sau đó hạ dần độ cao xuống đường băng Nội Bài (điểm số 1). Tai nạn xảy ra trong đoạn từ 2 đến 3, vào khoảng 10 tới 12 giờ sáng ngày 30/4/1971. Ông suy luận tiếp: 33 Tạp chí Epsilon, Số 14, 12/2018 Tổng hợp các thông tin tin cậy kết hợp với tọa độ và thời gian: 1. Tất cả các phi công đều khẳng định sự cố xảy ra vào khoảng từ 10 giờ tới 12 giờ sáng ngày 30/4/1971. Ngày hôm ấy trời quang mây tầm nhìn xa "ban ngày khí tượng đơn giản". Ngay sau khi xảy ra sự cố, quân binh chủng và chính quyền các cấp đã tổ chức tìm kiếm nhưng không thấy. Cho tới tận ngày 29/9/2017 vẫn chưa tìm thấy máy bay cũng như xác phi công. 2. Tất cả các phi công đều khẳng định sự cố xảy ra trong quá trình phi công Công Phương Thảo bay, nhắc lại cùng với thày là Yuri Poyarkov. Yuri Poyarkov là một phi công rất giỏi. Máy bay cất cánh từ sân bay Đa Phúc, nay gọi là sân bay Nội Bài, ở tọa độ (21.219086, 105.800507). Sau khi bay tập sẽ quay về hạ cánh cũng xuống sân bay này. Đường băng theo hướng Tây Bắc - Đông Nam. Máy bay cất cánh theo hướng Đông Nam, và hạ cánh từ hướng Tây Bắc. 3. Sự cố xảy ra do mất tín hiệu liên lạc, máy bay không về được căn cứ. Thời gian cất cánh là khoảng 10 giờ sáng. Anh Phạm Tuân khẳng định "lượng xăng của máy bay chỉ đủ bay trong 30 phút". Với lượng xăng như vậy máy bay chỉ có thể rơi trong phạm vi lãnh thổ Việt Nam. 4. Thời gian bay tới không vực tập bay là 5 phút. Thời gian bay tập dự định là 10 phút. Anh Nguyễn Khánh Duy (phi công cùng đơn vị bay với anh Công Phương Thảo) khẳng định "Công Phương Thảo xin phép bay về". Như vậy sự cố xảy ra sau khi bài bay tập đã hoàn thành và sự cố xảy ra với máy bay không thể bị coi là vì "trục trặc kỹ thuật". Sự cố xảy ra rất nhanh và đột ngột đến mức cả 2 phi công không kịp báo về sở chỉ huy bay. Như vậy sự cố xảy ra có lẽ là do yếu tố chủ quan của phi công khi hạ độ cao và có thể rơi vào vùng không khí nhiễu loạn (dòng đối lưu không khí) rồi đâm vào lưng chừng núi. Khoảng từ 10 giờ tới 12 giờ, ở nơi này mây thường xuyên bốc lên cao và bao phủ đỉnh núi. 5. Anh Nguyễn Khánh Duy khẳng định quỹ đạo bay về sân bay là bay theo đường thẳng từ không vực tới điểm có tọa độ (21.259882, 105.688555) (cách sân bay 10km) ở phía Nam Phúc Yên để bắt đầu hạ cánh xuống sân bay (21.219086, 105.800507). Đường bay về hướng thẳng vào dãy núi Tam Đảo. Tùy theo thời điểm bắt đầu quay về mà ước tính vị trí xảy ra sự cố. Thời tiết quang, tầm nhìn xa và khả năng nghe tốt, loại trừ các hướng bay mà người dân có thể phát hiện ra sự cố. 6. Động cơ của chiếc MIG 2. Khối lượng 1 tấn, dài hơn 4.5 mét, đường kính hơn 1.5m. Thông tin tiếng Nga: На МиГ-21С устанавливался один турбореактивный двигатель Р 11-Ф2С-300 (Сухая масса: 1040 кг; Максимальный диаметр: 0,825 м; Длина: 4,61 м) Từ tổng kết về thông tin này, ông tính toán và đưa ra kết quả suy diễn như sau (tóm tắt trong hình 2). Vòng tròn tâm B (21.698588, 105.659969) bán kính từ 10km đến 15km là không vực bay tập (chi tiết tỷ lệ có thể tìm thấy trên bản đồ Google). Thời gian bay từ sân bay tới không vực là 5 phút. Thời gian bay tập trong không vực là 10 phút. Độ cao bay từ 2000m tới 3000m cách mặt đất. Kết thúc bài bay tập anh Công Phương Thảo thông báo và xin phép quay về. Như vậy sự cố xảy ra trên đường máy bay bay về và máy bay ở trong tình trạng kỹ thuật tốt. Từ đây suy ra sự cố xảy ra là do chủ quan khi hạ độ cao và máy bay đâm vào núi ở độ cao từ 700m trở lên. 34 Tạp chí Epsilon, Số 14, 12/2018 Hình 2: Các khả năng có thể xảy ra dựa trên thông tin sau phỏng vấn, tính toán và suy diễn logic. Nguồn: NLA. Dựa vào hướng của sân bay chúng ta thấy kịch bản hạ cánh như sau. Để hạ cánh được máy bay phải bay đến điểm C (21.259882, 105.688555) ở độ cao 600m thấp dần trong khoảng 10km để tới đường băng (đường màu xanh). Như vậy máy bay phải hạ độ cao từ khoảng 2000m đến 3000m xuống độ cao 600m trong quãng đường 30km (từ không vực bay tập tới điểm 10km cách sân bay hạ cánh). Nếu bán kính vòng bay là 15km thì điểm máy bay sẽ bắt đầu quay về sân bay là từ điểm có tọa độ (21.704967, 105.474574) với độ cao khoảng 2000m tới 3000m. Địa hình đồi núi dọc quỹ đạo bay về thấp hơn 400m, và sau đó máy bay bay dọc theo lưu vực "Sông Phó Đáy"2. Như vậy khó có thể xảy ra sự cố với máy bay, và nếu như sự cố có xảy ra thì người dân sẽ nhìn thấy. Vậy bán kính vòng bay là 10km. Điểm bắt đầu hạ độ cao từ 3000m đến 4000m (so với mặt nước biển) để bay về có thể là điểm có tọa độ (21.736703, 105.553796). Từ nhận định này chúng ta có thể tìm thấy góc phương vị bay về (góc mầu đỏ) là góc ACB, tạo bởi 2 tia AC và BC . 2Đường màu vàng. 35 Tạp chí Epsilon, Số 14, 12/2018 Khả năng 1: Đường bay về AC. MIG 21 bay về qua đỉnh 3 của dãy Tam Đảo (nơi có độ cao 1400m ở tọa độ (21.493228, 105.634756)3. Trong 10km đầu tiên độ cao phải hạ được từ 1500m đến 2000m xuống còn 1500m. Như vậy khả năng đâm vào đỉnh 3 của dãy Tam Đảo Nam ở độ cao từ 1300 trở lên trong bán kính 10km, ở tọa độ (21.493228, 105.634756) từ phía Tây- Bắc. Khả năng 2: Đường bay về BC. MIG 21 bay về qua hướng hồ Đại Lải (nơi có đỉnh núi cao 1250m ở tọa độ (21.442866, 105.687328). Sau 20km bay phải hạ được 2000m độ cao xuống còn 1000m để bay tiếp 10km nữa phải hạ được 700m độ cao xuống còn 600m tại điểm (21.259882, 105.688555). Trên quỹ đạo bay có đỉnh núi cao 1250m. Vậy nếu đâm vào núi thì khả năng sẽ đâm ở độ cao khoảng 700m trở lên, trong bán kính 5km quanh tọa độ (21.442866, 105.687328), từ hướng Bắc - Tây Bắc. Như vậy trong cả 2 phương án, khả năng cao là máy bay đều xảy ra tai nạn ở vùng Nam Tam Đảo, nếu theo phương án về AC thì tai nạn sẽ xảy ra ở mặt phía Tây Bắc của đỉnh Tam Đảo 3, còn nếu theo phương án về BC thì tai nạn sẽ xảy ra ở phía Bắc - Tây Bắc so với đỉnh 1250m ở toạ độ (21.442866, 105.687328). Hai khả năng này được tóm tắt ở hình 3. Với kết quả suy luận này, không gian tìm kiếm đã được thu hẹp đáng kể so với khả năng "vô vọng"như ban đầu. Hình 3: Hai khả năng có thể khi máy bay bay về. Ngày 29 tháng 9 năm 2017, trong cuối một bài viết về vật lý, ông (Nguyễn Lê Anh) ghi một dòng ngắn: "Phải đi vụ Poliarkov", và như vậy cuộc hành trình tìm kiếm chính thức bắt đầu. 3Ở phía Nam của Tam Đảo có 3 đỉnh núi cao: đỉnh 1 là đỉnh Phù Nghĩa (1250m), đỉnh 2 là đỉnh Thiên Thị (1591m) và đỉnh 3 là đỉnh Thạch Bàn (1420m). Đỉnh đang nói đến trong suy luận là đỉnh Thạch Bàn. 36 Tạp chí Epsilon, Số 14, 12/2018 Lên đường Mặc dù phạm vi tìm kiếm đã được thu hẹp, nhưng vẫn còn rất rộng, và dẫu sao đó vẫn chỉ đang ở dạng các khả năng có thể xảy ra nhờ vào quá trình suy luận logic. Cần phải có những khảo sát chi tiết hơn cũng như nhiều thông tin hơn, tốt nhất là từ chính những người dân địa phương. Để có được thông tin, ông Nguyễn Lê Anh tiến hành thăm dò trực tiếp bằng cách cứ cuối tuần thì ông đi vào rừng thuộc dãy Tam Đảo Nam để leo núi, vốn cũng là một hoạt động quen thuộc của ông trong nhiều năm. Ông làm quen với những người dân tộc đi lấy nấm trong rừng và nhờ họ hỏi mọi người già thông tin về các máy bay rơi trong khu vực. Đôi khi đó chỉ là những câu hỏi bâng quơ, đôi khi thông tin được đổi lại từ việc hỗ trợ họ một ít chi phí cho việc hái nấm. Việc xác định được thông tin thoạt nghe có vẻ đơn giản nhưng trong thực tế không phải như vậy. Có những người dù thật lòng chia sẻ nhưng với tuổi tác và một sự kiện đã xảy ra hơn bốn thập kỷ thì độ chính xác của thông tin không còn cao nữa, cũng có những người sẵn sàng đưa ra những thông tin với độ tin cậy gần như không có, vốn chỉ để đổi lấy một chút tài lộc. Chúng tôi cũng nhắc lại là sự kiện xảy ra vào thời điểm chiến tranh khốc liệt, có rất nhiều máy bay bị bắn rơi hoặc bị tai nạn chứ không chỉ có duy nhất chiếc máy bay cần tìm. Nói thêm một chút về thông tin của chiếc MiG-21, theo Wikipedia, tên đầy đủ của nó là Mikoyan-Gurevich MiG-21 (tiếng Nga: Микоян и Гуревич МиГ-21 ) (tên ký hiệu của NATO: Fishbed) là một máy bay tiêm kích phản lực, được thiết kế bởi phòng thiết kế Mikoyan, Liên bang Xô viết. Ở Nga Mikoyan-Gurevich MiG-21 được gọi là Cây đàn Balalaika của bầu trời, vì nó có hình dáng cánh tam giác giống cây đàn dân tộc Nga, với quân đội Việt Nam, MiG-21 được gọi là thanh gương máu, huyền thoại của bầu trời. MiG-21 có nhiều phiên bản khác nhau, đa số là để chiến đấu, chỉ có một chỗ ngồi dành cho phi công, nhưng chiếc đang được tìm kiếm là MiG-21U, phiên bản huấn luyện có 2 chỗ ngồi với vị trí phía sau dành cho huấn luyện viên. Đây là một chi tiết quan trọng giúp cho việc xác định sau này. Sau một thời gian khảo sát, từ nói chuyện trực tiếp đến phỏng vấn qua điện thoại, Nguyễn Lê Anh hiểu rằng chiếc MiG-21U không bị rơi trong khu vực Tam Đảo Nam, hay nói cách khác, cả 2 khả năng từ suy luận ban đầu đều có dấu hiệu phải bị loại trừ. Vậy phải tìm ở đâu? Ông viết: "Việc dự đoán nó [máy bay MiG21-U] đâm phải phía bắc đỉnh Tam Đảo 3 trong dãy Tam Đảo Nam là dựa trên giả định chiếc MiG-21U giảm độ cao từ từ. Tôi tin chắc chắn chiếc MiG-21U sẽ bay theo đường thẳng AC, vậy nếu loại bỏ khả năng chiếc MiG-21U đâm vào khu vực phía Bắc của đỉnh Tam Đảo 3, thì MiG-21U phải có khả năng giảm độ cao đột ngột. Tôi lại tìm cách hỏi anh Khánh Duy và được anh cho biết MiG-21U có khả năng bổ nhào và giảm tốc bằng cánh. Vậy là rõ chiếc MiG-21U đã đột ngột giảm độ cao ở điểm A (trên hình 2) ngay sau khi xin phép bay trở về và đã va vào gờ núi Tam Đảo Bắc, suy đoán cách đỉnh Tam Đảo 3 khoảng 13km theo đường chim bay. Chúng tôi có một nhóm leo núi thể thao, chuyên leo vào cuối tuần ở các đỉnh núi Tam Đảo 1, 2, và 3; và cũng đã bàn về việc sẽ tổ chức đi tìm. Tuy nhiên công việc hàng ngày đã choán hết cả tâm trí." Ông viết tiếp: Câu chuyện được bắt đầu lại từ ngày 18/2/2018 khi có một bạn trẻ năm nay 30 tuổi tên là Đặng Tuấn. Đặng Tuấn có kể lại là mấy ngày Tết về quê nghe mọi người nói về chiếc máy bay rơi từ 37 Tạp chí Epsilon, Số 14, 12/2018 Hình 4: Suy đoán: "Ngay sau khi xin phép bay trở về và đã va vào gờ núi Tam Đảo Bắc, suy đoán cách đỉnh Tam Đảo 3 khoảng 13km theo đường chim bay." năm 1971. Đặng Tuấn đã truy tìm thông tin trên mạng Internet và thấy được bài viết của tôi cùng các tính toán gần giống với vị trí chiếc máy bay rơi mà bạn ấy biết từ khi còn nhỏ. Bạn ấy đã chủ động liên hệ với tôi. Đặng Tuấn cho biết: "Bác của cháu kể lại rằng năm 1971 có thấy một chiếc trực thăng của quân đội Việt Nam hạ cánh ở chân núi nói là tìm máy bay rơi nhưng không ai biết thông tin gì. Sau đó một thời gian thì mọi người kể lại rằng những người thợ săn họ tìm được một chiếc máy bay rơi và cháy ở trên núi. Những người đầu tiên biết về chiếc máy bay đó thì cũng nhặt được nhôm vụn đem bán, nhưng họ đã già và mất lâu rồi. Những người thợ săn trẻ sau này có người đã quay lại vị trí đó nhưng cũng chỉ thấy một cục sắt lớn nghi là động cơ và có kích thước bằng tủ lạnh." "Vì cháu cũng đi làm lâu rồi nên ngày tết cũng không dám hỏi thêm thông tin, nên cháu dùng Google và thấy được thông tin của bác, cháu hy vọng rằng thư mà cháu biết có thể giúp được gì đó ạ!" Ông nhận được thông tin này lúc đang ở Sài Gòn ăn tết, ngay lập tức ông mua vé đi Hà Nội và 8 giờ sáng ngày 19 tháng 2 năm 2018 ông cùng Đặng Tuấn đi về Yên Mỹ. Ông tiếp: Đặng Tuấn báo với mẹ làm cơm cho 3 người đi cùng ăn, và chúng tôi đi thẳng tới nhà chú của Đặng Tuấn là anh Hiệu. Anh Hiệu năm nay 49 tuổi. Anh Hiệu cho biết đúng là anh đã lăn cái lốp máy bay từ trên đỉnh núi cao xuống vực. Tuy nhiên việc này đã xảy ra từ hơn 20 năm về trước và anh đã không còn nhớ vị trí mà anh đã lăn chiếc lốp xuống. 38 Tạp chí Epsilon, Số 14, 12/2018 Như vậy là thông tin về việc có chiếc máy bay bị rơi trên định núi đã được một người xác nhận. Tôi không có thói quen đưa ra khẳng định khi không đủ chứng cứ khách quan. Thông báo của Hiệu là một thành tố rất có trọng lượng tuy nhiên nó đã có từ hơn 20 năm. Nó cần được kiểm chứng trực tiếp bằng cách tìm lại chiếc lốp ấy. Tôi cũng được nghe bà của Đặng Tuấn nói về người đầu tiên phát hiện ra chiếc máy bay đã lấy nhôm về bán và nhà ông ấy giầu lên đột ngột. Tôi chưa xác minh được chính xác thời điểm người dân ấy phát hiện ra chiếc máy bay, nhưng qua hỏi sơ bộ thời điểm chiếc máy bay được phát hiện ra ít nhất cũng trên 40 năm. Về sau người ta đã nung chảy tại chỗ chiếc máy bay để mang về bán. Vị trí nung chảy chiếc máy bay được gọi là Bãi Nhôm. Hiệu cho biết anh cũng không còn nhớ vị trí của Bãi Nhôm ấy. Về chiếc động cơ thì Hiệu chỉ nghe nói mà chưa bao giờ nhìn thấy. Ngoài ra Hiệu nói có tin đã có người đưa máy khò lên để cắt nhỏ chiếc động cơ mang đi bán. Như vậy các thông tin đưa tôi đến suy nghĩ, trường hợp xấu nhất, là sẽ không còn có khả năng tìm thấy dấu vết của chiếc máy bay. Cùng lắm chỉ có thể tìm thấy chiếc lốp bị lăn xuống vực. Tôi đề nghị Hiệu tổ chức một nhóm thám hiểm. Trao đổi thêm với Epsilon, ông cho biết ông đã phải suy nghĩ rất nhiều, và rất nhanh trong trường hợp này. Làm sao có thể biết liệu những "người bạn mới"này có đưa ra thông tin chính xác hay không? Có chắc là họ còn nhớ đúng? Giả sử nếu như họ nhớ đúng thì chắc gì máy bay nói đến đã là chiếc MiG-21U cần tìm? Tuy nhiên vì vị trí được mô tả trùng khớp với kết quả suy luận của ông, nên nó là một động lực thôi thúc ông, vì ông tin vào sức mạnh của suy luận logic. Hơn nữa, cần phải đi nhanh vì nếu chậm trễ có thể do nôn nóng những người khác sẽ làm cho cuộc tìm kiếm vốn đã khó sẽ trở nên khó khăn hơn. Do vậy, ông quyết định đi tìm ngay vào cuối tuần đó, vào hai ngày 24/2 và 25/2 (thứ Bảy và chủ nhật). Chúng tôi tiếp tục đăng lại trọn vẹn lời kêu gọi của ông cho nhóm tình nguyện tìm kiếm của ông, đăng ngày 22/2/2018 để tôn vinh tầm quan trọng của tư duy logic và cách làm việc khoa học: Đội tìm kiếm lưu ý: Việc tìm kiếm có thể sẽ gặp khó khăn do trời mưa và sẽ rất nhiều vắt rừng, ngoài ra dấu vết chiếc MiG-21U cũng không còn nhiều. Trời tuy có mưa nhưng là mưa phùn nên không ngại, để lâu hơn nữa sẽ có mưa rào là rất khổ. Ngại nhất là rét. Về ban đêm trên đỉnh núi trời có thể rét có thể xuống tới 2 độ C. Chúng ta sẽ quyết đi tìm vào ngày 24/2/2018, tìm trong ít nhất 2 ngày thứ 7 và Chủ Nhật. Dù thế nào cũng đi, không tìm thấy quyết không về. Vì quá trình tìm kiếm khó khăn nên đội đi tìm sẽ chỉ gồm những người thực sự chịu đựng được gian khổ, không nên đi vì háo hức. Có lẽ chỉ nên lính đi tìm lính thì hơn. Vậy cần danh sách người đi. Chúng ta nên đi cùng nhau trên 1 chiếc ô tô đến nơi thì leo. Những ai cùng đi thì inbox cho tôi để chúng ta lấy điện thoại của nhau. A - Nhiệm vụ: Nếu toàn bộ số kim loại của máy bay đã bị lấy đi mất chúng ta chỉ còn hy vọng vào 2 chiếc lốp máy bay đã vì nghịch mà cho lăn xuống vực. Lốp trước đường kính khoảng 0.5m, lốp sau đương kính khoảng 1m. Phía bên trong của chiếc lốp sẽ có chữ in nơi sản xuất ra chúng. Từ đây chúng ta sẽ đưa ra được quyết định có phải đó là chiếc MiG-21U hay không. Như vậy chúng ta phải tìm bằng được 2 chiếc lốp ấy. Dự tính hai chiếc lốp ấy đã bị lăn xuống vực sâu dốc đá thẳng đứng. Rất có thể phải dùng dây để leo xuống. Theo ý tôi chúng ta không nên mạo hiểm mà nên thuê thợ săn bản xứ đi tìm cùng. Chúng ta cần một đội hậu cần mang đồ lên cho chúng ta ăn ở tại chỗ. Chúng ta cần 1 lều ngủ cho 4 người, lương thực thực phẩm và nước uống. Những thứ này sẽ thuê người địa phương 39 Tạp chí Epsilon, Số 14, 12/2018 mang lên. B - Kỹ thuật leo núi: Vào mùa này trong rừng có rất nhiều vắt. Lên trên cao do đang mùa lạnh vắt sẽ ít hơn. Trời cũng đã ấm lên các loại rắn bò ra kiếm ăn. Khi leo núi cần phải để ý quan sát. + Mỗi người tự in ra bản đồ (lấy trên Google). Tôi sẽ chỉ rõ cho các bạn vị trí bắt đầu leo cũng như điểm phải leo đến. Khi di chuyển trong rừng các bạn phải hình dung ngay ra vị trí của các bạn trên bản đồ. Ngay cả khi không có sóng điện thoại thì tín hiệu vệ tinh vẫn có, chính vì vậy các bạn cần nhanh chóng cập nhật bản đồ vào điện thoại để biết được mình đang ở đâu. + Trường hợp bị lạc các bạn phải nhanh chóng leo lên vị trí cao nhất trong khu vực, nơi ấy sẽ có sóng điện thoại để gọi. Các bạn phải rất bình tĩnh đợi, vì sóng có thể lúc có lúc không. Trong mọi trường hợp phải cố di chuyển về vị trí lúc xuất phát. + Không nên mạo hiểm đi một mình. Vách đá có thể rất trơn và có thể tuột xuống vực ngay dưới chân mình mà không hay. C - Về trang bị: - Cả đoàn: 1 lều ngủ và tấm trải cách nhiệt cho 4 người. Lương thực thực phẩm và nước uống. - Cá nhân: Mỗi người tự lo cho mình, bao gồm 1 balo, tất cả mọi thứ trong balo đều được gói trong túi nilon đề phòng mưa ướt sẽ gầm vào đồ dùng khiến chúng trở nên rất nặng. Đồ dùng cá nhân phải có: + Về đồ dùng gồm: đèn pin + võng + 1 tấm vải mưa 1.5mx2m (mua ở Hà Trung) + túi ngủ cá nhân + găng tay, giầy leo núi (cỡ to hơn 1 số so với giày thường đi) và tất + quần áo mặc để leo (2 bộ) và quần áo ấm đề phòng ngủ lại trong rừng. Mỗi người mang theo 1 đôi giầy dự phòng để phòng khi giày hỏng (giày thể thao nhẹ) và khoảng 40m dây dù nhẹ mà bền. + Về đồ ăn thì phải mang theo nước uống 3 lit + đồ ăn nhẹ đủ sống được 2 ngày + cồn khô và bật lửa đề phòng tối phải ngủ lại thì đốt đống lửa + điện thoại & sạc dự phòng. Đường màu xanh là đường dự kiến sẽ leo để lên đỉnh. Vùng màu đỏ dự kiến là vùng máy bay rơi. Chắc chắn có máy bay rơi nhưng không rõ là máy bay gì. Khuya ngày 23 rạng sáng ngày 24 tháng 2 năm 2018, khoảng nửa ngày trước khi lên đường ông viết tiếp cho đoàn: Hầu hết tất cả các núi kể cả Everest chiều cao từ chân núi lên tới đỉnh cũng chỉ vào khoảng 1500m. Như vậy vị trí mà chúng ta cần phải tới được trong ngày hôm nay là rất cao. Thông thường chúng ta phải đi 12km để lên được cao 1000m, như vậy tổng đường đi bộ để lên độ cao 1500m là khoảng 18km. Thời gian đi khoảng 5 giờ. Chúng ta đi theo con đường tắt, nhiều chỗ dốc thẳng đứng, vì thế chúng ta cần ít nhất 4 tiếng đê lên tới nơi. Theo kinh nghiệm thì nhanh nhất cũng khoảng 2 giờ chiều thì đoàn chúng ta mới tới được độ cao 1500m. Đến nơi chúng ta phải theo người dân bản địa đi tìm và xác định tọa độ bãi nhôm, nơi đã nấu chảy chiếc máy bay. Và tìm kiếm một vài mảnh vụn. Việc tiếp theo là xác định xem có phải nơi 40 Tạp chí Epsilon, Số 14, 12/2018 Hình 5: Suy đoán: "Đường màu xanh là đường dự kiến sẽ leo để lên đỉnh. Vùng màu đỏ dự kiến là vùng máy bay rơi." Nguồn: NLA. đấy đã từng có một chiếc máy bay thì chúng ta phải mang máy dò kim koại và tổ chức đoàn tìm kiếm có kinh nghiệm hơn. Nếu mọi việc tốt đẹp thì chúng ta sẽ xuống núi vào lúc 5 giờ chiều và vào khoảng 10 giờ đêm sẽ xuống tới chân núi. Lưu ý. 1- Đặt chế độ cho điện thoại ghi nhớ đường đi. 2- Đường đi thường là men theo vực Có một chi tiết nhỏ cũng cần phải nói thêm về độ tin cậy của thông tin nhận được. Bằng suy luận, ông có nghĩ tới "thuyết âm mưu", nhưng vì mọi thông tin đều rất trùng khớp với tính toán ban đầu, nên ông vẫn quyết đi tìm và tạm gạt bỏ suy nghĩ này. Cụ thể vào trước ngày đi, ông viết: Như vậy chúng ta không có cơ hội tìm thấy chiếc máy bay vì nó đã bị biến thành sắt vụn. Nếu chúng ta tìm thấy chiếc lốp và nếu chúng ta đọc được chữ trên chiếc lốp ấy thì có thể xác định được nó có phải là MiG-21U hay không. Chỉ có điều nếu như toàn bộ sự việc diễn ra đúng như lời kể thì chiếc máy bay đã không bị cháy nổ. Bởi nếu cháy nổ thì chả dễ gì có thể gom được mảnh để nấu. Nếu cháy nổ thì chiếc lốp sẽ bị cháy không còn để có mà lăn xuống vực. Vậy từ đây theo thuyết âm mưu có thể có nhiều điều để suy diễn. 41 Ngày tìm kiếm thứ nhất Tạp chí Epsilon, Số 14, 12/2018 Hà Nội, 5 giờ sáng ngày 24 tháng 2 năm 2018, những con người dũng cảm lên đường. Toàn bộ thành viên tham gia gồm có 8 người: 4 người địa phương và 4 người tình nguyện viên, trong đó có Nguyễn Lê Anh, cũng là người lớn tuổi nhất. Ngoài 8 người họ ra, còn có các hỗ trợ khác ở bên dưới mặt đất và đông đảo mọi người hỗ trợ tinh thần qua Internet. Nguyễn Lê Anh viết lại về ngày đầu tiên này như sau: Nhóm dẫn đường gồm 4 người Hiệu, Trung, Nam, và Phú. Không một ai trong số họ còn nhớ vị trí Bãi Nhôm và có lẽ họ chưa từng đến đấy. Trong số 4 người thì duy nhất có Phú cho biết anh đã từng sờ tay vào chiếc động cơ máy bay và Hiệu thì nói là đã từng lăn một chiếc lốp từ trên cao xuống vực. Nhóm tình nguyện viên gồm 4 người, trong đó có tôi. Trừ tôi ra các thành viên đều còn trẻ và rất hưng phấn. Khi nhìn thấy các thành viên không mang theo nước uống tôi biết họ không thể theo được. Vì thế tôi đã không kiểm tra tư trang của các thành viên trước khi lên đường. Họ chưa đủ kinh nghiệm cho một cuộc đi như vậy. Khoảng 10 giờ sáng ngày 24/2/2018 chúng tôi bắt đầu leo núi. Theo quy định tôi chỉ bật điện thoại vào các phút chẵn 30. Hai thành viên sớm bỏ cuộc sau khi leo lên một cái đồi không cao lắm. Tôi đưa cho Hiệu 1 chai nước to và phân công Hiệu đi cuối. Tôi và 3 người địa phương còn lại đến được vị trí cắm trại vào lúc 14:00 chiều, đúng như dự kiến. Nhờ có nước uống, thành viên thứ 3 cũng tới được, vị ấy tới vào lúc 19:00 tối, nhưng anh đề nghị quay lui vào sáng hôm sau. Sáng hôm sau tôi cử Hiệu đưa anh ấy quay trở về và dặn là phải trở lại tìm tiếp4. Ngày 24/2/2018 diễn ra như vậy. Pin điện thoại và sạc chỉ được dự tính cho 2 ngày thám hiểm cũng đã không còn nhiều. Và như vậy, sau khi ngày đầu tiên kết thúc, cả đoàn chỉ còn lại mỗi "người thúc đẩy"Nguyễn Lê Anh, một nhà toán học đã trên 60 tuổi, và 4 người dân địa phương. 4Trên trang longnguyen48.blogspot.com có nói về ngày đầu tiên và một phần của ngày thứ hai này với góc nhìn từ các thành viên tình nguyện còn lại như sau: Chẳng hiểu mọi lần leo núi (là nghề) của GS Lê Anh như thế nào nhưng lần này đi, mưa xuân rả rích nặng hạt, trời mù. . . Nói chung, thời tiết khá phức tạp cho cả việc leo núi lẫn bay huấn luyện khi xưa. Thậm chí tiến sĩ Doãn Hà Thắng còn không thể cho cái Drone (là một thiết bị giống flycam) hoạt động được vì cây cối quá rậm rạp. Ngay ngày đầu tiên, bác sĩ Phúc đã phải bỏ cuộc vì ong rừng đốt đến mức tê dại hai chân, khiến chân không có cảm giác để đứng thăng bằng và chuyên gia an ninh mạng Ngô Việt Khôi thì vì sức khỏe của dân phượt, không thể đua được với chuyên gia leo núi cũng bỏ cuộc. Như vậy là hai nhân tố trẻ của đoàn đã phải bỏ cuộc tìm kiếm ngay từ ngày đầu tiên. Đây cũng là nguyên nhân mà GS Lê Anh cảm thấy sợ hãi sau này (sẽ kể ở đoạn sau). Cuộc tìm kiếm kéo dài đến tối mịt, chúng tôi ở nhà hồi hộp chờ đợi kết quả. Nhưng cái chính là lo cho tính mạng của các thành viên đoàn. Cho tới lúc này chỉ còn anh và Ts Doãn Hà Thắng là người tìm kiếm cùng ba người bản xứ vốn là người Kinh, cách đây hai mươi năm đã từng trèo lên khu vực này tìm kiếm xác máy bay để. . . lấy nhôm bán sắt vụn. Sang ngày thứ hai, tiến sĩ Doãn Hà Thắng cũng phải bỏ cuộc vì tim đập nhanh quá, nghẹt thở, có lúc bị ngất vì những cú trèo đá núi cheo leo. Đến lúc này chỉ còn GS Lê Anh và “đồng đội tìm sắt vụn”. Các tính toán bằng xác xuất toán học của Lê Anh cho đến giờ này vẫn khá chính xác hướng tìm kiếm. Viện trưởng Mai Hương liên tục động viên bằng cách phi xe đến tận chân núi gần nhất có thể để . . . hóng tin. 42 Tạp chí Epsilon, Số 14, 12/2018 Ngày tìm kiếm thứ hai Nguyễn Lê Anh viết tiếp: Vị trí cắm trại phải là một nơi có nước. Nó nằm cách nơi chúng tôi dự định tìm khoảng 1 giờ đi bộ. Thời tiết lạnh và mưa phùn cũng như ngày hôm trước. Đường rất trơn. Như đã nói ngoại trừ anh Phú, còn hai anh Nam và Trung chỉ nghe đồn thổi về vụ máy bay rơi mà chưa hề nhìn thấy chiếc động cơ cũng như Bãi Nhôm. Do trời mù mà phải tới 10 giờ sáng ngày 25/2 mới bắt đầu đi tìm chiếc động cơ. Tôi không can dự vào quyết định mà để họ tự đi tìm theo ý của mình, với hy vọng sớm tìm thấy nó. [Nếu căn cứ theo lời kể của họ thì] nhiều bộ phận của chiếc động cơ đã bị tháo vì thế kích thước mỗi chiều của nó chỉ khoảng nửa mét. Dự tính phạm vi tìm kiếm diễn ra trong phạm vi khoảng 1km2. Tôi không thật sự tin là họ sẽ tìm được một vật như vậy, một khi không biết chắc chắn vị trí từ trước. Đến 14:00 thì cả ba anh về trại. Sau khi nghe thông báo đã tìm nát hết các vị trí có thể tôi trầm ngâm suy nghĩ. Từ 10:00 đến 14:00 là 4 tiếng. Đi lại nhanh hết 1 tiếng vậy chỉ có 3 tiếng đi tìm. Đi trong rừng rậm, vừa đi vừa phạt cây để lấy lối đi, tốc độ chỉ khoảng 2km/giờ. Ba người đi được cùng lắm 18km. Nếu tầm mắt bao quát nhìn sáng 2 bên được 10m thì phải đi 50km mới quét hết diện tích 1km2... Như vậy có nghĩa là họ chưa tìm được một phần ba khu vực cần tìm kiếm, làm sao có thể kết luận là "tìm nát hết"các vị trí. Quay lại giả thuyết về "thuyết âm mưu", liệu chăng có khả năng toàn bộ thông tin về Bãi Nhôm lẫn chi tiết lăn lốt bánh xe xuống vực là không chính xác. Liệu rằng các sự kiện đó có thật sự xảy ra. Nếu giả như tất cả đều không xảy ra ... Ông kể lại nổi lo lắng của mình vào buổi chiều của ngày thứ hai: Chiều muộn tôi nhận được tin nhắn của Nam [Nam Nguyen, người kêu gọi đầu tiên trên FB vào ngày 25/9/2017] thông báo quay về vì tất cả những người địa phương đang đi tìm với tôi đều không biết tí gì, và vị trí chiếc máy bay rơi ở chỗ khác. Tôi giữ im lặng đề phòng. Các bạn người địa phương tranh luận với nhau rất nhiều và gọi điện thoại để hỏi. Họ bảo tôi là chúng ta không thể tìm được gì vì đã có người mang máy dò kim loại lên để dò hết các mảnh vỡ. Thậm chí tên người khò và tên người bán cái động cơ cũng được nói ra và họ tìm cách gọi điện hỏi. Chúng tôi ăn tối. Tôi hôm ấy tôi không ngủ và cố hình dung ra vị trí chiếc máy bay. Tôi cho rằng chiếc máy bay nếu chỉ bay cao hơn một vài chục mét là nó đã thoát, và vì vậy vị trí va chạm của chiếc máy bay phải ở ngần ngay trên gờ của núi. Tôi đánh dấu trên bản đồ vị trí của cái gờ nhô lên. Và ông tiếp tục cố gắng thêm một ngày nữa, tiếp tục tìm kiếm vào ngày Thứ Hai, 26/2/2018. 43 Ngày tìm kiếm thứ ba Nguyễn Lê Anh kể tiếp: Tạp chí Epsilon, Số 14, 12/2018 Sáng thứ 2 trời vẫn mưa và rét. Hiệu gọi điện thoại với giọng ngập ngừng. Tôi hiểu nguyên nhân của sự thay đổi ấy. Hiệu cho biết tối hôm trước Nam, Thắng và Hiệu đã đến nhà một số người dân tộc và nhận được những thông tin nào đó. Cái này Nam đã nhắn tin cho tôi. Tôi làm như không biết gì và nói Hiệu phải lên để gặp gỡ chúng tôi vì tinh thần đồng đội. Trên thực tế tôi cũng đã hơi lo, và muốn phải có Hiệu. Tôi cần phải đưa ra quyết định trong bối cảnh không biết ai nói đúng, ai đáng tin và thông tin nào đáng tin cậy. Tôi nói với mọi người rằng thép của chiếc động cơ rất dày không thể bị khò mang đi và chúng ta nên tìm ngược theo các con suối cạn ngược lên đỉnh. Vào khoảng 9:00 chúng tôi nhổ trại vừa đi tìm vừa về. Khoảng 10:00 chúng tôi đến chỗ tìm. Ngay lúc ấy chúng tôi gặp 3 cậu thanh niên dân tộc. Ngoảnh trước ngoảnh sau họ đã rút và uống hết chai nước của tôi. Chỗ này không có một con suối nào có nước. Không có nước uống là sẽ chết khát. Tôi bắt đầu thấy sợ vì có thể đấy là một lời cảnh cáo. Tôi có cảm giác hình như mọi thứ đều chống lại việc tôi tiếp tục đi tìm. Tuy nhiên tôi hiểu là việc phải giữ vẻ bề ngoài bình tĩnh không thay đổi là quan trọng. Quá trình tìm kiếm rất chậm. Sau khoảng 30 phút tôi hiểu là không thể tìm được cái động cơ ấy giữa rừng rậm. Tôi nói mọi người chúng ta lên đỉnh cao nhất và tìm cách xác định vị trí để đi tìm xuống. Trên thực tế tôi chỉ muốn xác định được vị trí nơi chiếc máy bay đã lao vào núi. Khoảng 13:00 giờ chúng tôi trèo đến đỉnh. Trên đó rất lạnh, chúng tôi đốt lửa chờ Hiệu. Vào khoảng 14:00 chúng tôi quyết định đi tìm tiếp. Lần này tôi đi trước và cố tìm thấy cái gờ mà tôi đã thấy trên bản đồ Google. Tôi leo lên một đỉnh núi và tôi biết khi đi xuống là sẽ đến cái gờ tôi muốn. Khoảng 14:30 chúng tôi đến chỗ ấy. Mọi người cũng đồng ý là tìm xuống từ chỗ ấy. Chúng tôi chia ra thành các nhóm đi tìm xuống. Tôi đi cùng với Phú. Phú có dao phạt cây để đi, tôi khảo sát một chỗ đỡ dốc mà tôi cho là nơi máy bay đâm vào. Không tìm thấy gì, tôi thất vọng đi vào phía khe suối cạn toàn đá tảng lổn nhổn. Lòng suối rất khó đi, tôi bị ngã mấy lần. Và tôi bắt đầu thấy sợ, bởi chỉ cần bị ngã thụt chân vào khe đá gãy chân thì sẽ không thể ra khỏi rừng được. Tôi bắt đầu suy nghĩ. Nếu cứ thế này xuống được tới con đường bên dưới mất 2 giờ. Sau đó phải mất 3 giờ mới ra được khỏi rừng. Như vậy là mất 5 giờ. Lúc ấy đã là khoảng 3 giờ chiều, vậy chúng tôi chỉ có thể ra khỏi rừng lúc 8 giờ đêm. Đi trong rừng trời tối rất nhanh và khi trời tối thì đi rất chậm. Nếu vậy thì sẽ phải 10 giờ đêm chưa ra được khỏi rừng. Hiểu ra sự nguy hiểm, tôi nói với Phú chúng ta thất bại rồi, phát lệnh "Lui Quân". Tôi lệnh cho Phú vẫn phải phát cây, vẫn đi tìm cái động cơ, nhưng tìm ở con đường ít dốc nhất, dễ đi nhất để xuống. Xét thâm tâm là do tôi sợ, tôi cần phải xuống thật nhanh và ra khỏi rừng trước khi trời tối. Phú đi trước phát đường vào chỗ ít dốc nhất có thể, tôi đi ngay sau. Và thật kỳ diệu, chỉ phát đường được khoảng 5m thì chúng tôi nhìn thấy mảnh máy bay. Kỳ tích đã thật sự xảy ra! 44 Tạp chí Epsilon, Số 14, 12/2018 Hình 6: Vào lúc 14:50 ngày 26 tháng 2 năm 2018, đoàn tìm kiếm đã tìm thấy mảnh máy bay ở vị trí (21◦34’46.9"N 105◦33’04.9"E) cách xa không quá 100m so với vị trí được tính toán dựa vào suy luận logic. Nguồn: NLA. Hình 7: Chiều ngày 25/2/2018, một ngày trước khi tìm ra mảnh máy bay, ông Nguyễn Lê Anh viết: "Mặc dù chưa tìm được dấu vết chiếc MiG-21U, nhưng tất cả đều tin đây chính là nơi chiếc máy bay "hạ cánh". Nó tiếp đất an toàn ở độ cao khoảng 1400m. Theo phong tục Việt Nam chúng tôi thắp nén hương kính viếng linh hồn các anh." Nguồn: NLA. 45 Ông kết thúc ký sự của mình như sau: Tạp chí Epsilon, Số 14, 12/2018 Xét về bản chất sau 47 năm mưa dầm mảnh máy bay ấy sẽ chỉ có thể đọng lại nơi mà ít dốc nhất có thể. Chúng tôi ở trong rừng 3 ngày 2 đêm. Không kể thời gian đi đường, tổng thời gian tìm là khoảng 3 giờ và 30 phút. Rất ít người biết được cái tư duy cuối cùng đưa lại thành công bắt nguồn không phải từ tư duy logich mà từ nỗi sợ hãi của tôi. Chúng tôi nhanh chóng ra về 5 giờ chiều đã ra khỏi rừng. Sau một kết cục thật có hậu cho một chuyến đi vất vả, anh em chúng tôi, Nam, Trung, Hiệu và Phú thấy tin yêu nhau hơn. Nam Nguyen lên đón tôi và gửi chút quà nhỏ biếu các bạn cùng đi tìm. Kết thúc hành trình Sau khi tìm được mảnh vỡ máy bay bị nghi là của máy bay MiG-21U mất tích, đoàn tìm kiếm buộc phải quay về và không thể tìm tiếp vì lý do họ đã kiệt sức, cũng như không đủ nước, lương thực và cả pin cho các thiết bị. Họ sau đó đã bàn giao mảnh máy bay cho các đơn vị có thẩm quyền. Với mảnh máy bay này, hàng loạt câu hỏi cần được giải đáp: liệu nó có phải đúng là mảnh của máy bay cần tìm hay không? Máy bay bị tai nạn như thế nào? Nếu đúng, liệu có thể tìm thấy thi hài của những người đã mất hay không? Trong lúc đi tìm thông tin, có người kể rằng "thày Poyarkov cõng anh Thảo mặt đầy máu đi trong rừng và có ai đó đã bắn chết họ", liệu đây có phải là sự thật? Như đã giới thiệu với bạn đọc, MiG-21U là máy bay duy nhất trong dòng MiG-21 có 2 chỗ ngồi, và may mắn là mảnh vỡ được tìm thấy là mẫu ở giữa 2 vị trí đó. Với một chuỗi logic và so sánh, đối chiếu với hình ảnh và thông tin, chuyên gia Nhật Đình, một người bạn của ông Nguyễn Lê Anh đã chứng minh được đó chắc chắn phải là mảnh vỡ của MiG-21U. Về sau các cơ quan có thẩm quyền cũng đã xác nhận tính chính xác, kỳ tích được xác nhận! Về các câu hỏi còn lại, ông Nguyễn Lê Anh viết lại việc này vào ngày 23/3/2018 như sau: Ngày 28/2/2018, chúng tôi đã bàn giao mảnh ID cùng tọa độ nơi tìm ra nó và ảnh chụp vị trí mảnh ghép cho Quân chủng Không Quân. Lẽ dĩ nhiên vị trí rơi của chiếc MiG-21U nằm không xa vị trí mảnh máy bay tìm được và nhiều người dân biết rất rõ vị trí ấy. Tuy vậy vẫn cần phải xác minh thực hư về câu chuyện lan đồn "thày Poyarkov cõng anh Thảo mặt đầy máu đi trong rừng và có ai đó đã bắn chết họ."Lời nguyền độc địa này cần phải được giải thoát để trả lại sự vô can cho linh hồn những người thợ săn đã khuất. Tôi đã leo lên núi lần thứ hai. Lần này là nhằm mục đích xác định "liệu phi công có khả năng còn sống hay không?" Thông tin chính xác: - Tọa độ (21◦34’50.1"N 105◦33’12.0"E ) nơi tìm được dù (có lẽ là dù máy bay) cùng rất nhiều mảnh nhỏ và vết đâm sâu vào sườn núi nghi là do máy bay đâm vào. - Tọa độ (21◦34’46.9"N 105◦33’04.9"E) nơi tìm được mảnh ID MiG-21U 46 Tạp chí Epsilon, Số 14, 12/2018 Suy diễn: Mảnh ID MiG-21U nằm ngay dưới vị trí máy bay rơi, cao độ chênh 140m, ở khoảng cách xa 183m. Trong bán kính 200m phía trên mảnh ID MiG-21U không còn có một vị trí nào có thể nghi là nơi bị một máy bay đâm vào. Như vậy chúng ta có thể khẳng định được vị trí chiếc MiG-21U của anh Thảo và Poyarkov đâm vào sường núi là tọa độ nói trên. Rất nhiều người dân khẳng định tìm được một chiếc vỏ màu vàng không ruột của chiếc đồng hồ Poljot tại nơi nghi là máy bay đâm vào sường núi. Nếu chắc chắn đó là chiếc đồng hồ đeo tay của Đại úy phi công Poyarkov thì cả hai phi công đã hy sinh nơi máy bay đâm vào núi. Nhật Đình đã xác minh mảnh vỡ ID MiG-21U là ở phía sườn trái cabin của MiG-21U. Dựa vào tọa độ mảnh vỡ ID MiG-21U tìm được, và tọa độ nơi có hố vết đâm vào núi, chúng ta tính ra được mảnh vỡ đã văng ra sớm hơn 70m so với vị trí của hố. Như vậy chiếc máy bay đã cà lườn phải vào vách núi một khoảng 50m trước khi nó húc và khoét ra một hố mỗi chiều 2m. Mảnh ID MiG-21U bị văng ra ngay khi máy bay cà lườn phải vào núi. Mảnh ID không bị biến dạng theo chiều dọc, nó bị văng ra theo phương vuông góc. Vận tốc mảnh ID MiG-21U bắn ra khỏi máy bay là khoảng 100km/giờ. Tính toán này phù hợp với vết sạt dài khoảng 30m để lại hiện trường. Những giây cuối cùng. Ngay sau khi xin phép bay về, chiếc MIG đã bổ nhào từ 4000m xuống 2000m để bay về Nam Phúc Yên. Lúc này vận tốc chiếc MIG lên đến trên 1200km/giờ. Chiếc MIG chắc đã kịp dùng bụng cản gió để triệt tiêu động năng. Vận tốc của nó giảm xuống còn khoảng 600km/giờ. Cuối tháng tư, mùa hè nóng. Từ 10 giờ trở đi mặt trời lên cao làm cho dòng khí đối lưu mạnh. Nơi sát đỉnh núi không khí bốc lên rất mạnh, nơi thung lũng thì dòng khí đi xuống. Thung lũng Hoàng Nông có đường kính 4km, và như vậy 12 giây cuối chiếc MIG21U mới phát hiện ra nó đã bị hạ thấp độ cao quá mức. Chiếc MiG-21U đã ngoặt gấp sang trái 90 độ với hy vọng vượt qua gờ núi Hoàng Nông - Yên Mỹ, phía thấp hơn. Nó có thể đã thoát nạn nếu bay cao hơn được 10m (bay theo đúng giáo trình không thể phạm sai lầm nhiều). Khi phi công rời khỏi máy bay, chiếc máy bay sẽ bay thẳng theo quán tính. Như vậy trong thời gian bay vòng phi công vẫn ở trong máy bay. Độ dài đoạn bay thẳng cuối cùng không quá 0.5km. Nếu bay với tốc độ 600km/giờ thì mất khoảng 3 giây. Như vậy giây thứ 3 các phi công vẫn còn trong máy bay. Thời gian để phóng ghế ra khỏi máy bay không ít hơn 3 giây. Do đó cho dù các phi công có khởi động ghế phóng hay không, họ cũng đã lao vào sườn núi và chết ngay tại vị trí máy bay rơi. Những suy diễn nói trên phù hợp với sự thật là người dân cho biết vào thời điểm ấy máy bay trực thăng bay quần đảo nhiều ngày trong phạm vi núi Tam Đảo Bắc. Đó chính là máy bay trực thăng của Quân chủng Không Quân đi tìm chiếc MiG-21U. Nếu phi công còn sống họ đã phát tín hiệu cho trực thăng. Như vậy câu chuyện lan đồn về hình ảnh người thày Poyarkov cõng anh Thảo đi trong rừng chỉ như một minh chứng cho tình cảm và sự giúp đỡ của Liên Xô với Việt Nam. Hình ảnh rất đẹp nhưng chỉ là sự tưởng tượng. Các phi công đã chết ngay khi máy bay sạt vào núi. Và đây cũng là khẳng định giải thoát lời nguyền độc địa rằng có ai đó đã bắn họ. Xin được mặc niệm và tỏ lòng biết ơn các anh hùng liệt sĩ đã hy sinh vì tổ quốc Việt Nam. Đến cuối tháng 9, đầu tháng 10 năm 2018 như chúng tôi có nói ở đầu bài viết, các cuộc khai quật đã được tiến hành, và dù, vật dụng ... của những người đã khuất đã được tìm thấy. Một câu chuyện cổ tích đã có kết thúc có hậu sau đúng một năm tìm kiếm, tính từ ngày bài đăng trên FB Nam Nguyen. 47 Tạp chí Epsilon, Số 14, 12/2018 Lời kết Vào một ngày đẹp trời mùa hè 2018, tổng biên tập Trần Nam Dũng có ý muốn "hồi sinh"Epsilon sau khi số cuối cùng (số 13) ra mắt bạn đọc hơn 1 năm về trước. Chúng tôi vẫn luôn tự hào về Epsilon và luôn hài lòng với quyết định để Epsilon dừng lại khi đang được ủng hộ rất nhiệt thành từ cộng đồng (để nhường chỗ cho tạp chí Pi). Tôi luôn suy nghĩ nếu để Epsilon quay trở lại, thì tôi phải làm một cái gì đó mới hơn, và hay hơn. Tôi tự hỏi đâu là thế mạnh lớn nhất của Epsilon và tạm trả lời đó chính là sự tự do. Epsilon không gò bó về nội dung, miễn nó có liên quan đến toán, cũng không áp đặt về hình thức. Epsilon không phân biệt đó là bài viết từ học sinh, hay từ giáo sư, tiến sĩ danh giá như Ngô Bảo Châu, tất cả đều bình đẳng với nhau. Vậy thì tại sao không có một số bài viết với một hình thức mới, không phải chỉ là đề bài và lời giải? BM2E là một phong trào khác, cũng được khởi xướng bởi cùng một người, Trần Nam Dũng. BM2E có mục tiêu chính là đem toán học tới mọi người và chủ đề gần đây nhất của BM2E là "học toán để làm gì". Khi đọc tiêu đề đó, tôi nói với ông: "Em không thấy có ai làm tốt hơn được huyền thoại NLA. Nếu thầy mời được, sẽ là vinh dự và là đột phá lớn cho BM2E", và ông đã làm được! Khi ý tưởng bên trên đã thông, mọi thứ còn lại nó đến một cách hết sức tự nhiên. Vốn kiến thức có được từ những bài viết trên FB của Nguyễn Lê Anh, có lẽ chúng tôi có dùng thêm 10 năm nữa với Epsilon mới có thể đăng hết! Nhưng tôi không muốn chọn ngay những bài viết quá thuần toán học của ông, tôi muốn chọn một vấn đề "đời"hơn, và hấp dẫn hơn. Hành trình tìm ra chiếc MiG-21U có lẽ rất nhiều người biết vì truyền thông đã làm tốt điều này, nhưng làm thế nào họ tìm ra nó, chuỗi suy luận và tính toán đó, nếu không ai ghi lại cho hậu thế một cách rõ ràng và chi tiết, thì thật đáng tiếc. Và nếu chưa ai làm, thì chúng tôi sẽ làm. Tuy chúng tôi đứng tên cho bài viết này, nhưng phải khẳng định toàn bộ bài viết gần như đều là trích dẫn từ chính lời viết của tiến sĩ Nguyễn Lê Anh. Chúng tôi không phải và không đủ khả năng làm tác giả của bài viết này, chúng tôi chỉ làm công việc thuật lại câu chuyện cổ tích có thật này mà thôi. 48 Tạp chí Epsilon, Số 14, 12/2018 GIỚI THIỆU BÀI TOÁN TỐI ƯU HAI LỚP (BI-LEVEL OPTIMISATION PROBLEM) Võ Nhật Vinh (Đại học Caen Normandie, Caen, Pháp) GIỚI THIỆU Tại giải Penalty Challenge 2019, một nhóm các cầu thủ sẽ thách thức một nhóm các thủ môn đang thi đấu tại V-League 2019 với màn "đấu súng" từ chấm 11m: duy nhất một cú đá với một thủ môn và một cầu thủ mà mỗi nhóm tự chọn. Mỗi thủ môn tại thời điểm hiện tại có một phong độ (xác suất chặn được bóng) khác nhau. Ứng với mỗi cầu thủ, mỗi thủ môn có một xác suất chặn được cú đá 11m của cầu thủ đó. Câu hỏi được đặt ra là: thủ môn nào sẽ được chọn, để xác suất chắc chắn chặn được bóng là cao nhất. 1. Giới thiệu chung Trong câu chuyện vừa kể phía trên, với mỗi thủ môn, xác suất chắc chắn chặn được bóng ứng với xác suất chặn được bóng thấp nhất khi đối đầu với các cầu thủ. Sư so sánh giữa các xác suất thấp nhất này cho phép chọn lựa thủ môn phù hợp tiêu chí: là người có xác suất thấp nhất cao nhất. Nói cách khác, đây là bài toán "lớn nhất - nhỏ nhất" (Max-Min) bao gồm hai bài toán tối ưu, trong đó lời giải của một bài toán này (xác suất thấp nhất) được dùng như ràng buộc phục vụ cho bài toán còn lại (xác suất thấp nhất cao nhất). Đây là một ví dụ đơn giản của bài toán tối ưu hai lớp (bi-level optimisation problem). Bài toán tối ưu hai lớp khởi nguồn từ trò chơi Stackelberg [1, 2] nhằm nghiên cứu sự phụ thuộc của các quyết định trong nền kinh tế thị trường [3]. Trong các nghiên cứu đó, quá trình lên kế hoạch kinh tế liên quan đến sự tương tác giữa các tác nhân ở hai cấp độ riêng biệt: vài cá nhân (được gọi chung là lãnh đạo - leader) ra chỉ thị cho những tác nhân còn lại (gọi là người đi theo - followers) [4]. Các bài toán này bắt đầu được nghiên cứu trong loạt bài của hai tác giả Bracken và McGill [5, 6, 7], nhưng thuật ngữ "hai lớp" (bi-level) bắt đầu xuất hiện trong một báo cáo của Candler và Norton [8]. Trong thực tế, bài toán này xuất hiện rộng rãi trong nhiều vấn đề thuộc các lĩnh vực đa ngành. Phần 3 sẽ giới thiệu thêm về bài toán này trong các lĩnh vực thực tế. Bài toán tối ưu hai lớp xem xét hai biến quyết định x và y cho hai mục tiêu tối ưu phụ thuộc nhau, trong đó x(y) là lời giải tối ưu cho bài toán thứ hai ứng với mỗi y cho trước. Chương trình 49 Tạp chí Epsilon, Số 14, 12/2018 hai lớp là một bài toán tối ưu mà trong đó, một phần của các ràng buộc được định nghĩa bởi một bài toán tối ưu thứ hai. Theo [3], bài toán tối ưu thứ hai có thể được định nghĩa bởi ( 1): minx {f(x, y) : g(x, y) ≤ 0} (1) Bài toán này được gọi là bài toán theo (follower’s problem) hay bài toán dưới (lower problem). Gọi Φ(y) là tập nghiệm của bài toán ( 1) và x(y) là một phần tử nào đó của tập Φ(y). Nhiệm vụ của bài toán ( 2), bài toán dẫn (leader’s problem) hay bài toán trên (upper problem) là xác định lời giải tốt nhất y∗ và x(y∗) thỏa mãn ràng buộc G(x(y), y) ≤ 0 và đem đến giá trị tốt nhất cho F(x(y), y): miny{F(x(y), y) : G(x(y), y) ≤ 0, x(y) ∈ Φ(y)} (2) Bài toán tối ưu hai lớp có thể được giải bởi nhiều phương pháp khác nhau bằng cách biển đổi nó thành bài toán tối ưu thông thường (một lớp) ( [3], trang 3). Nếu bài toán dưới cho lời giải duy nhất, bài toán hai lớp là ổn định (stable). Tuy nhiên, khi bài toán hai lớp được biến đổi thành bài toán một lớp, nó vẫn có thể được giải trong trường hợp nghiệm của bài toán dưới không là duy nhất. Trở lại bài toán ở giải Penalty Challenge 2019, biến x và biến y lần lượt biểu diễn cho cầu thủ và thủ môn. Gọi P(x, y) là xác suất chặn được bóng của thủ môn y khi người sút bóng là cầu thủ x. Gọi Q(y) là phong độ hiện tại của thủ môn y. Bài toán dưới (lower problem) ( 1) được viết lại như sau: minx {f(x, y) : g(x, y) ≤ 0} f(x, y) = P(x, y) (3) g(x, y)=0 (4) Bài toán trên (upper problem) ( 2) được viết lại như sau: miny{F(x(y), y) : G(x(y), y) ≤ 0, x(y) ∈ Φ(y)} F(x(y), y) = −P(x(y), y) × Q(y) (5) G(x(y), y)=0 (6) 2. Kỹ thuật giải Bài toán tối ưu hai lớp, trong trường hợp đơn giản nhất, bao gồm hai bài toán tuyến tính cũng là một bài toán với độ khó NP − Hard [9]. Vì vậy, có thể nói rằng bài toán tối ưu hai lớp là một bài toán khó và để giải được nó, chúng ta cần thuật toán có độ phức tạp thuộc lớp NP. Trong bài viết của mình [4], các tác giả đề cập đến khá nhiều kỹ thuật giải cho bài toán tối ưu hai lớp. Tuy nhiên, trong phạm vi bài viết với mục đích chính là giới thiệu bài toán này, kỹ thuật biến đổi bài toán tối ưu hai lớp sang bài toán tối ưu thông thường (một lớp) dựa vào điều kiện Karush-Kuhn-Tucker (KKT) sẽ được trình bày. Ngoài ra, một công cụ giải bài toán tối ưu hai lớp trong Matlab [10] cũng được giới thiệu. 50 Tạp chí Epsilon, Số 14, 12/2018 2.1. Kỹ thuật sử dụng điều kiện KKT Điều kiện KKT [11, 12] (KKT conditions) được dùng để biến đổi bài toán theo thành một điều kiện ràng buộc trong bài toán dẫn, qua đó biến bài toán tối ưu hai lớp thành bài toán tối ưu một lớp. Bài toán tối ưu hai lớp được viết lại thành: min x,yF(x, y) (7) Thỏa mãn hệ điều kiện: G(x, y) ≤ 0 (8) g(x, y) ≤ 0 (9) λ ≤ 0 (10) λT × g(x, y)=0 (11) ∂L(x, y, λ)=0 (12) Trong đó L(x, y, λ) = f(x, y) + λT × g(x, y). Nếu L(x, y, λ) là một hàm lồi (convex) thì ∂L(x, y, λ) = ∇L(x, y, λ). Trong trường hợp tổng quát, điều kiện KKT là điều kiện đủ nên bảo đảm lời giải là nghiệm của bài toán (nếu có, nhưng không đảm bảo được tính đầy đủ của tập nghiệm). Ngoài ra, khi bài toán theo không có nghiệm duy nhất, bài toán tối ưu hai lớp trở nên phức tạp hơn với trường hợp lạc quan (optimistic position) và trường hợp bi quan (pessimistic position) cũng như tính toàn cục hay cục bộ của lời giải tối ưu. 2.2. Công cụ YALMIP Bộ công cụ (toolbox) YALMIP dành cho Matlab [13, 14] đóng vai trò của một giao diện phổ quát (generic interface) nhằm giúp người dùng giải quyết các bài toán tối ưu khác nhau, trong đó có bài toán tối ưu hai lớp một cách dễ dàng hơn. YALMIP cho phép người dùng chỉ đích danh công cụ giải (solver) kèm theo các tùy chọn. Nói cách khác, YALMIP đơn giản hóa việc thực thi (implementation) giải các bài toán tối ưu với optimize kèm theo sự chỉ định công cụ giải như linprog hay quadprog. Các bài toán tối ưu hai lớp đơn giản có thể được giải trực tiếp bằng cách sử dụng solvebilevel của YALMIP. 3. Một số áp dụng Như đề cập ở phía trên, bài toán tối ưu hai lớp xuất hiện rộng rãi trong thực tế ở nhiều lĩnh vực đa ngành. Phần dưới đây giới thiệu hai bài toán thực tế mà bài toán tối ưu hai lớp đã được sử dụng để mô hình hóa chúng. 51 3.1. Bài toán thu phí cầu đường Tạp chí Epsilon, Số 14, 12/2018 Ở nhiều quốc gia, các tuyến giao thông huyết mạch được xây dựng theo hai nhánh song song: nhánh miễn phí và nhánh có thu phí (chất lượng tốt hơn, tốc độ cho phép cao hơn). Người điều khiển phương tiện giao thông có thể chủ động chọn nhánh đường tùy theo nhu cầu của mình. Ở góc độ đơn vị quản lý cầu đường, họ muốn thu thật nhiều tiền. Số tiền họ thu được phụ thuộc vào giá tiền đưa ra và số lượng xe chấp nhận trả số tiền ấy. Ngược lại, ở góc độ người điều khiển xe, mỗi người sẽ cân nhắc lựa chọn đường đi với việc chấp nhận trả mức phí đó hay không: nhanh hơn thuận tiện hơn thế nào, yêu cầu thời gian chuyến đi ra sao, nhu cầu đi nhanh hay chậm của những người ngồi trên xe ... Ứng với mỗi mức phí được ấn định, bài toán theo liên quan đến chi phí tối thiểu cho mỗi xe, tức là chi phí tốn kém khi sử dụng tuyến đường có thu phí (bao gồm phí cầu đường) hoặc chi phí sử dụng tuyến đường miễn phí. Nghiệm của bài toán theo là câu trả lời cho số lượng xe sử dụng tuyến đường có thu phí ứng với mức phí đã ấn định. Bài toán dẫn sẽ quyết định mức phí được ấn định để tích của mức phí và số lượng xe trả phí là lớn nhất. 3.2. Bài toán giá trong lưới điện thông minh Khác với lưới điện truyền thống, người sử dụng điện trong lưới điện thông minh (Smart-Grids) có khả năng tùy chỉnh mức sử dụng điện của mình cũng như dễ dàng thay đổi nhà cung cấp. Trong một dự án liên quan đến việc sử dụng bình ắc-quy trong lưới điện thông minh tại Đại học Kỹ thuật Vienna (TU Wien - Áo), mô hình đơn vị kinh doanh điện (trung gian giữa nhà cung cấp điện và người tiêu dùng) được áp dụng với vai trò thúc đẩy đảm bảo sản lượng tiêu thụ điện theo cam kết. Trong dự án này, chính sách giá linh hoạt theo giờ được ấn định bởi các đơn vị kinh doanh đã được nghiên cứu nhằm định hướng hành vi sử dụng điện của khách hàng. Tương tự bài toán thu phí cầu đường, sự ảnh hưởng qua lại của chính sách giá và hành vi của khách hàng đã được nghiên cứu trong bài toán tối ưu hai lớp [15]. Ở góc độ chi tiết hơn, việc sử dụng năng lượng tái tạo đòi hỏi sự tuân thủ chặt chẽ về công suất tiêu thụ điện (tải) nhằm tránh sự quá tải. Trong khi đó, các hộ gia đình ở Đức đã sử dụng đến 79 TWh điện để đun nước nóng vào năm 2015 [16], bao gồm xấp xỉ 12 TWh cho máy nước nóng gia đình [17]. Mức tiêu thụ điện này chiếm 9.3% lượng điện tiêu thụ của các hộ gia đình và 2.0% tộng lượng tiêu thụ điện của toàn nước Đức. Các con số này cho thấy bài toán sử dụng điện hiệu quả cho máy nước nóng gia đình rất quan trọng. Trường Đại học Kỹ thuật Hamburg (TUHH - Đức) đã thực hiện một nghiên cứu về bài toán này với việc áp dụng bài toán tối ưu hai lớp. Ứng với mỗi mức giá điện cho trước, các hộ gia đình sẽ có chính sách sử dụng máy nước nóng gia đình sao cho hóa đơn điện phải trả là thấp nhất (bài toán theo). Chính sách sử dụng máy nước nóng gia đình của các hộ dân ứng với mỗi mức giá điện sẽ cho ra đường tiêu thụ điện tương ứng - thứ mà người ta mong muốn nó giống với đường tiêu thụ điện định mức (căn cứ vào công suất điện được cung cấp vào các thời điểm). Vì vậy, trong bài toán dẫn, người ta mong muốn tìm ra mức giá điện sao cho sự khác biệt giữa đường tiêu thụ điện (ứng với mức giá) và đường tiêu thụ điện định mức là nhỏ nhất [18]. 52 Tạp chí Epsilon, Số 14, 12/2018 4. Kết luận Bài toán tối ưu hai lớp (bi-level optimisation problem) là một bài toán tối ưu trong đó có sự phụ thuộc kiểu dẫn-theo của hai biến quyết định. Bài toán này là một loại bài toán khó yêu cầu thuật toán với độ phức tạp thuộc lớp NP để giải. Cách thường sử dụng để giải bài toán này là cố gắng biến đổi bài toán gốc thành bài toán tối ưu thông thường một lớp. Có nhiều kỹ thuật để giải nhưng trong bài viết này, kỹ thuật sử dụng điều kiện Karush-Kuhn-Tucker (KKT) đã được giới thiệu. Ngoài ra, công cụ YALMIP trên môi trường Matlab cũng được giới thiệu để giải các bài toán tối ưu, trong đó có bài toán tối ưu hai lớp. Bài toán tối ưu hai lớp xuất hiện rộng rãi trong thực tế, đặc biệt trong các lĩnh vực liên ngành. Trong bài viết này, các vị dụ liên quan tới bóng đá hay tính phí cầu đường cũng như điều chỉnh giá trong lưới điện thông minh (Smart-Grids) đã được giới thiệu và mô hình hóa bằng cách sử dụng bài toán tối ưu hai lớp. Bài toán này vẫn đang thu hút được nhiều nhà nghiên cứu nhằm sử dụng chúng để mô hình hóa các bài toán thực tế, cũng như nhằm tìm cách để giải chúng một cách hiệu quả hơn. Tài liệu [1] H. Von Stackelberg, The Theory of the Market Economy. London: Oxford University Press, 1952. [2] H. Von Stackelberg, Market Structure and Equilibrium. Berlin, Heidelberg: Springer Berlin Hei delberg, 2011. [3] S. Dempe, Nonconvex Optimization and Its Applications - Foundations of Bilevel Programming. Dordrecht: Kluwer Academic Publisher, 2002. [4] B. Colson, P. Marcotte, and G. Savard, “An overview of bilevel optimization,” Annals of Operations Research, vol. 153, no. 1, pp. 235–256, 2007. [5] J. Bracken and J. T. McGill, “Mathematical Programs with Optimization Problems in the Con straints,” Operations Research, vol. 21, pp. 37–44, feb 1973. [6] J. Bracken and J. T. McGill, “Defense Applications of Mathematical Programs with Optimization Problems in the Constraints,” Operations Research, vol. 22, pp. 1086–1096, oct 1974. [7] J. Bracken and J. T. McGill, “Production and marketing decisions with multiple objectives in a competitive environment,” Journal of Optimization Theory and Applications, vol. 24, pp. 449–458, mar 1978. [8] W. Candler and R. Norton, “Multilevel programming,” tech. rep., World Bank Development Re search Center, Washington D.C, 1977. [9] R. G. Jeroslow, “The polynomial hierarchy and a simple model for competitive analysis,” Mathe matical Programming, vol. 32, pp. 146–164, jun 1985. [10] MathWorks, “Matlab,” 2016. [11] W. Karush, Minima of Functions of Several Variables with Inequalities as Side Constraints. Msc thesis, University of Chicago, 1939. 53 Tạp chí Epsilon, Số 14, 12/2018 [12] H. W. Kuhn and A. W. Tucker, “Nonlinear programming,” in Proceedings of 2nd Berkeley Sympo sium, (Berkeley, CA), pp. 481–492, University of California Press, 1951. [13] J. Lofberg, “YALMIP : A Toolbox for Modeling and Optimization in MATLAB,” in ¨ Proceedings of the CACSD Conference, (Taipei, Taiwan), 2004. [14] J. Lofberg, “YALMIP,” 2017. ¨ [15] R. M. Kovacevic, N. V. Vo, and J. Haunschmied, “Bilevel approaches for distributed DSM using internal individualized prices,” in 2017 IEEE International Conference on Smart Grid Communi cations (SmartGridComm), pp. 521–526, IEEE, oct 2017. [16] Destatis, “Destatis—statistisches bundesamt,” 2017. [17] G. Stryi-Hipp, “Grosol—studie zu großen solarwarmean- lagen,” tech. rep., Bundesverband- ¨ Solarwirtschaft e.V, Berlin, 2007. [18] T. Lubkert, M. Venzke, N. V. Vo, and V. Turau, “Understanding price functions to control domes- ¨ tic electric water heaters for demand response,” Computer Science - Research and Development, vol. 33, pp. 81–92, feb 2018. 54 Tạp chí Epsilon, Số 14, 12/2018 TÍNH CHẤT PHI ARCHIMEDEAN CỦA ĐỊNH GIÁ P-ADIC Nguyễn Song Minh GIỚI THIỆU Khái niệm về định giá p-adic (hay còn gọi là số mũ đúng), là một khái niệm quan trọng trong Số Học vì nó giúp chuyển hóa các vấn đề Số học sang ngôn ngữ Giải Tích để tìm cách giải quyết. Bài viết này, có nội dung chính bàn đến một tính chất căn bản của định giá p-adic, đó là tính phi Archimedean. Phần đầu bài viết, trình bày lại những khái niệm, quy tắc và tính chất căn bản. Phần còn lại, là các bài toán áp dụng. Các quy ước và ký hiệu Trong bài viết này, chúng ta sử dụng các ký hiệu với ý nghĩa được quy ước thống nhất như sau: ! gcd.a; b/ W Ước số chung lớn nhất của a; b 2 Z. ! m 6 j a W Số nguyên a không chia hết cho số nguyên m ¤ 0: ! bxc W Số nguyên lớn nhất không vượt quá số thực x (phần nguyên của x). ! '.m/ W Phi hàm Euler. ! vp.x/ W Hàm định giá p-adic. ! P W Tập hợp chứa tất cả các số nguyên tố. 1. Mở đầu Với hai số nguyên dương a và m, theo định lý cơ bản của Số học, ta luôn có biểu diễn sau đây a D Y p2P pvp.a/; m D Y p2P pvp.m/: Ở đó, các bộ số tự nhiên !vp.a/"p2P và !vp.m/"p2P xác định duy nhất, theo nghĩa nếu a D m thì điều kiện cần và đủ là vp.a/ D vp.m/; 8p 2 P: 55 Tạp chí Epsilon, Số 14, 12/2018 Việc kiểm tra quan hệ m j a, giờ đây quy về xem xét tính đúng sai của đánh giá vp.a/ ! vp.m/; 8p 2 P: Việc xem xét quan hệ a " b .mod m/ với a>b và a; b 2 N!, quy về kiểm tra bất đẳng thức vp.a # b/ ! vp.m/; 8p 2 P: Như vậy, nhiều định tính cơ bản trong Số học sẽ được kiểm soát qua các đánh giá định lượng với vp. Lý do đó, cho ta thấy là rất cần quan tâm đến định nghĩa sau. Định nghĩa 1 (Định giá p-adic trên N!). Nếu số nguyên dương m có phân tích ra thừa số nguyên tố ở dạng m D Y p2P pvp.m/; trong đó vp.m/ 2 N, thì với mỗi một số nguyên tố p cố định vp.m/ được gọi là định giá p-adic của m: Ví dụ v2.2016/ D 5; v3.2016/ D 2 và v7.2016/ D 1 còn v11.2016/ D v17.2016/ D v2017.2016/ D 0: Với số nguyên tố p và số nguyên dương m cho trước, nếu ta đặt Y q2Pnfpg thì p 6 j m0 và có được m D pkm0: qvq.m/ D m0; vp.m/ D k; Như vậy, về bản chất thì k D vp.m/ là số lớn nhất trong các số tự nhiên t thỏa mãn pt j m. Vì pt j 0 với t lớn thỏa ý, nên ta mở rộng được khái niệm định giá p-adic lên N với quy ước vp.0/ D C1; 8p 2 P: Nếu quy ước thêm rằng vp.a/ D vp.#a/ với a 2 Z n N, ta mở rộng được khái niệm định giá p-adic trên Z, vẫn với bản chất vp.m/ D k, nếu m D pkm0 với k 2 N; m0 2 Z và p 6 j m0: Bây giờ, với một số hữu tỷ r ¤ 0 bất kỳ, ta thấy luôn tồn tại duy nhất số nguyên k và a; b 2 Z! thỏa p 6 j ab và r D pk ab : Nhờ vậy, ta mở rộng được khái niệm lên Q, như sau đây. Định nghĩa 2 (Định giá p-adic trên Q). Cho trước một số nguyên tố p và hữu tỷ r. i. Nếu r D 0, thì định giá p # ad ic của r là vp.0/ D C1. ii. Nếu r ¤ 0 thì định giá p # ad ic của r ký hiệu là vp.r/ và ta có vp.r/ D k khi và chỉ khi r D pk ab ; trong đó, k 2 Z và a; b 2 Z! với p 6 j ab. Ở một góc nhìn khác, nếu số hữu tỷ r viết được dưới dạng phân số là r D mn với m; n 2 Z thì ta sẽ có được vp .r/ D vp .m/ # vp .n/: 56 Tạp chí Epsilon, Số 14, 12/2018 2. Các tính chất cơ bản Với p là một số nguyên tố, x; y là các số hữu tỷ bất kỳ và a; b; m là các số nguyên với m ¤ 0. Khi đó, chúng ta có các tính chất và quy tắc cơ bản như sau. 1. vp.xy/ D vp.x/ C vp.y/ và vp .xn/ D nvp.x/ với n 2 N!. # 2. vp $ x D vp .x/ # vp .y/; vp.1/ D 0: y 3. vp .gcd.a; b// D min ˚vp.a/; vp.b/%: 4. vp .lcm.a; b// D max ˚vp.a/; vp.b/%: 5. x 2 Z khi và chỉ khi vp.x/ ! 0; 8p 2 P: 6. a j b khi và chỉ khi vp.a/ " vp.b/; 8p 2 P. 7. a " b .mod m/ khi và chỉ khi vp.a # b/ ! vp.m/; 8p 2 P. 8. vp.x C y/ ! min ˚vp.x/; vp.y/%và khi vp.x/ < vp.y/ thì vp.x C y/ D vp.x/: Các quy tắc và tính chất này đều chứng minh rất đơn giản, qua việc trực tiếp sử dụng định nghĩa về định giá p-adic trên Q. Trong các tính chất đã nêu, bốn tính chất đầu tiên cho ta các quy tắc rất tiện lợi để tính toán định giá. Ba tính chất kế tiếp, thực chất là các điều kiện tương đương, chúng giúp ta chuyển hóa các định tính Số học căn bản thành định lượng theo vp. Riêng tính chất cuối cùng, còn được gọi là tính chất phi Archimedean, nhờ tính chất này ta hình thành được chuẩn p-adic trên Q bởi công thức sau đây kxkp D p"vp.x/: Khi ta có chuẩn p-adic, ta có được các không gian siêu metric với metric d.x; y/ D kx # ykp. Các không gian siêu metric, có một tính chất thú vị là một hình cầu mở (hoặc hình cầu đóng) vừa là một tập mở vừa là một tập đóng và hai hình cầu cứ có điểm chung thì sẽ có một hình cầu chứa hình cầu còn lại. Những tư tưởng hiện đại nhưng rất căn bản này, là nền tảng để chúng ta soi xét nhiều bài toán Olympiad. 3. Một số bài toán áp dụng Bài toán 1. Cho trước số nguyên tố p, và n (với n>1) phân số tối giản a1 b1 , a2 b2 ; :::; an bn . Biết rằng tổng n phân số đó là một số nguyên, đồng thời tồn tại số nguyên dương m và chỉ số j thỏa pm j bj . Chứng minh rằng, tồn tại chỉ số k ¤ j sao cho pm j bk: bi D ri và giả sử không tồn tại chỉ số k ¤ j sao cho pm j bk, khi đó Lời giải. Đặt ai vp!rj"" #mc D v2.P .3k//, thế thì theo bất đẳng thức Bernoulli ta có 1 C n " 2n và kết hợp giả thiết mà suy ra c<1 C n<1 C n C k<2n C k " v2.P .32nCk//: Theo tính chất phi Archimedean và đánh giá (1), ta có mâu thuẫn là n " v2.P .32nCk/ # P .3k// D v2.P .3k// D c: Tổng kết các suy luận trên cho ta thấy P !3k"D 0 với mọi số nguyên dương k, từ đó đa thức P .x/ có vô số nghiệm nên nó phải là đa thức 0. Bài toán 4. Xét các số hữu tỉ dương x1; x2; :::; xn thỏa mãn xi C 1pilà các số nguyên dương với pi D x1x2###xn xi ; 8i D 1; n: i. Chứng minh rằng x1x2 $$$ xn D 1: ii. Có bao nhiêu bộ số .x1; x2; :::; xn/ thỏa mãn đề bài. Lời giải. Đặt p1p2 $$$ pn D a, từ giả thiết ta có đánh giá sau với mọi số nguyên tố p & 0 " vp xi C1pi'D vp#xi Cxia$D vp .xi/ C vp .1 C a/ # vp .a/: (2) Lấy tổng lại với lưu ý là vp .a/ D P 1"i"n vp .xi/, ta có 0 " nvp .1 C a/ # .n # 1/ vp .a/: (3) i. Nếu vp.a/ > 0, theo tính chất phi Archimedean ta có vp .1 C a/ D vp .1/ D 0: Vậy nên từ (3), có mâu thuẫn với tình huống đang xét là 0 " # .n # 1/ vp .a/: Còn nếu vp.a/ < 0, theo tính chất phi Archimedean lại có vp .1 C a/ D vp .a/: Và lại từ (3), có mâu thuẫn với tình huống đang xét là 0 " nvp.1 C a/ # .n # 1/ vp .a/ D vp.a/: Tóm lại là luôn phải có vp.a/ D 0 với mọi p 2 P, kết hợp a>0 ta có được a D 1. ii. Từ a D 1, với mỗi số nguyên tố p lẻ và mọi chỉ số i theo (2) có 0 " vp .xi/; 0 " 1 C vp .xi/: Vậy, các số hữu tỷ đó có dạng xi D 2"1Cki với ki 2 N và từ a D 1 ta có ràng buộc k1 C k2 C $$$ C kn D n: Mỗi bộ .k/ D .k1; k2; :::; kn/ 2 Nn sẽ tương ứng với một bộ số hữu tỷ dương thỏa yêu cầu, vì thế theo bài toán chia kẹo Euler ta có số bộ cần tìm là N D !2n"1 ": n"1 Lời giải hoàn tất. 59 Tạp chí Epsilon, Số 14, 12/2018 Bài toán 5. Cho a1; a2; ::: là một dãy vô hạn các số nguyên dương. Giả sử tồn tại số nguyên dương N sao choa1 a2Ca2 a3C $$$ Can"1 anCan a12 Z; 8n ! N: Chứng minh rằng tồn tại số nguyên dương M sao cho amC1 D am; 8 m ! M. anC1 C anC1"an Lời giải. Từ giả thiết, ta có ngay an p và số nguyên dương n ! N ta có & an a1 2 Z; 8n ! N. Vì thế, với mọi số nguyên tố ' vp anC1CanC1 # an a1 ! 0; 8n ! N: (4) Giả sử tồn tại số nguyên tố p sao cho vp .an/ < vp .anC1/, với n ! N. Do vp từ tính chất của định giá phi Archimedean và (4) ta có # an anC1 $ < 0 nên ' vp < 0: Từ đó có & an anC1 ' D vp &anC1 # an a1 0>vp .an/ # vp .anC1/ D vp .anC1 # an/ # vp .a1/ D vp .an/ # vp .a1/: Tức là vp .an/ < vp .anC1/ D vp .a1/: Nếu p là số nguyên tố thỏa mãn vp .an/ > vp .anC1/, với n ! N. Từ (4) và tính chất phi Archimedean có 0 " vp &anC1 # an a1 ' D vp .anC1 # an/ # vp .a1/ D vp .anC1/ # vp .a1/: Như vậy lại có đánh giá vp .a1/ " vp .anC1/ < vp .an/: Tóm lại là với n ! N thì nếu vp .an/ < vp .anC1/ sẽ có vp .an/ < vp .anC1/ D vp .a1/: (5) Còn nếu vp .an/ > vp .anC1/ và n ! N, thì sẽ có vp .a1/ " vp .anC1/ < vp .an/: (6) Với mỗi số nguyên tố p, ta xét hai trường hợp 1. Nếu tồn tại k ! N sao cho vp .ak/ < vp .akC1/ thế thì theo (5) có vp .akC1/ D vp .a1/. Từ đây vp .akC2/ D vp .a1/, bởi nếu vp .akC2/ > vp .a1/ D vp .akC1/ sẽ mâu thuẫn với (5) còn nếu vp .akC2/ < vp .a1/ D vp .akC1/ lại mâu thuẫn với (6). Truy toán sẽ cho ta vp .an/ D vp .a1/; 8n ! k C 1: 60 Tạp chí Epsilon, Số 14, 12/2018 2. Nếu không tồn tại k ! N sao cho vp .ak/ < vp .akC1/ thì có nghĩa là vp .an/ ! vp .anC1/; 8n ! N: Những suy luận trên cho thấy với số nguyên tố p bất kỳ, sẽ tồn tại Np để có vp .anC1/ " vp .an/; 8n ! Np: Nhưng do giá trị của vp .an/ là các số tự nhiên, nên việc không xảy ra dấu bằng chỉ ở hữu hạn chỉ số. Từ đó, với mỗi số nguyên tố p, đều tồn tại Mp đủ lớn sao cho vp .anC1/ D vp .an/; 8n ! Mp: Cũng từ (5) và (6) ta lại thấy ngay là với mọi số nguyên tố p thì vp .anC1/ " max ˚vp .an/; vp .a1/%" max ˚vp .aN /; vp .a1/%; 8n ! N: Từ đó thấy dãy fangn2N! chỉ có hữu hạn ước nguyên tố. Chọn M D max ˚Mp%với p chạy khắp tập các ước nguyên tố của fangn2N! , ta có điều cần chứng minh. Bài toán 6. Chứng minh rằng, tồn tại vô số các bộ ba số hữu tỷ .a; b; c/ thỏa mãn a C b C c D abc D 6: Lời giải. Giả sử bộ ba số hữu tỷ .a; b; c/ thỏa yêu cầu, khi đó có c ¤ 0 đặt c D # 6m và có a C b D 6 C6m; ab D #m ¤ 0: Vậy nên theo Viettè, a; b là nghiệm của đa thức P .x/ D mx2 # .6m C 6/ x # m2, đa thức này cần có hai nghiệm hữu tỷ cho nên phải tồn tại r 2 Q sao cho "0P D 9.m C 1/2 C m3 D r2: Đến đây thế m C 3 D x, ta quy bài toán về đi chứng minh có vô số .x; y/ 2 Q2 thỏa x3 # 9x C 9 D y2: Xét đường cong elliptic có phương trình .E/ W y2 D x3 # 9x C 9, ta cần chứng tỏ trên .E/ có vô số điểm hữu tỷ. Trước tiên, ta thấy M0.0; 3/ là một điểm như thế. 61 y Mn (xn, yn) O Tạp chí Epsilon, Số 14, 12/2018 x Mn+1 (xn+1, yn+1) Giả sử Mn .xn; yn/ là một điểm hữu tỷ trên .E/, khi đó tiếp tuyến tại Mn có phương trình là .Tn/ W y D &3x2n # 9 2yn ' .x # xn/ C yn: Hoành độ giao điểm của tiếp tuyến đó với .E/ là nghiệm phương trình (&3x2n # 9 2yn ' .x # xn/ C yn )2 D x3 # 9x C 9: Sau biến đổi, phương trình trở thành .x # xn/2 " x # &3x2n # 9 2yn '2 C 2xn # D 0: Vì thế, tiếp tuyến và đường cong sẽ còn có một điểm chung là MnC1 .xnC1; ynC1/ D !3x2n # 9"2 4y2n# 2xn; &3x2n # 9 2yn ' .xnC1 # xn/ C yn ! : Do y2n D x3n # 9xn C 9, nên có xnC1 D f .xn/ với !3x2 # 9"2 f .x/ D 4 .x3 # 9x C 9/# 2x D x4 C 18x2 # 72x C 81 4 .x3 # 9x C 9/ : 62 Tạp chí Epsilon, Số 14, 12/2018 Ta có x1 D f .0/ D 94 , nên v2 .x1/ D #2. Bây giờ giả sử x 2 Q và v2.x/ " #2 khi đó rõ ràng có các đánh giá sau 4v2 .x/ D v2!x4"< 1 C 2v2 .x/ D v2!18x2"; 4v2 .x/ D v2!x4"< 3 C v2 .x/ D v2 .72x/; 4v2 .x/ D v2!x4"< 0 D v2 .81/; 3v2 .x/ D v2!x3"< v2 .x/ D v2 .9x/; 3v2 .x/ D v2!x3"< 0 D v2 .9/: Từ đó theo tính chất phi Archimedean ta có v2 .f .x// D v2 &x4 C 18x2 # 72x C 81 ' 4 .x3 # 9x C 9/ D v2!x4 C 18x2 # 72x C 81"# v2!x3 # 9x C 9"# 2 D v2!x4"# v2!x3"# 2 D v2.x/ # 2: Như vậy, với x1 D 94 và xnC1 D f .xn/ với n 2 N, ta có được #2 D v2 .x1/ > v2 .x2/ > $$$ > v2 .xn/ > $$$ Vậy nên các điểm Mn là đôi một phân biệt, và ta có điều phải chứng minh. 4. Bài tập Sau đây là các bài toán, để tự luyện tập thêm. Bài tập 1. Cho các số nguyên dương a; b; c thỏa mãn gcd.a; b; c/ D 1 và a j bc; b j ca; c j ab: Chứng minh rằng bca là một số chính phương. Bài tập 2. Cho các số nguyên dương a; b; m lớn hơn 1 và số nguyên tố p thỏa mãn pm D ab C 1: Chứng minh rằng p.a # 1/.b C 1/ không là số chính phương. Bài tập 3. Cho số nguyên dương a lớn hơn 1, tìm tất cả các số nguyên tố p và q thỏa mãn pq j .ap"q C 1/: Bài tập 4. Cho các số hữu tỷ a và b phân biệt, giả sử tồn tại vô số số nguyên dương n thỏa mãn an # bn 2 Z: Chứng minh rằng a và b đều là các số nguyên. 63 Tạp chí Epsilon, Số 14, 12/2018 Bài tập 5. Tìm các số nguyên dương a; b; c sao cho ab # c; bc # a và ca # b đều là các số nguyên dương không có ước nguyên tố lẻ. Bài tập 6. Cho f W Z ! Z là một hàm số khác hàm hằng thỏa mãn điều kiện là với các số nguyên phân biệt a và b bất kỳ thì .a # b/ j .f .a/ # f .b//: Chứng minh rằng tồn tại một tập vô hạn S chứa các số nguyên tố, sao cho với mỗi p 2 S đều tồn tại m 2 Z sao cho p j f .m/. Bài tập 7. Với mỗi số nguyên dương n, ký hiệu f .n/ D &2v2.n/ n 'v2. n2 / : Chứng minh rằng nếu m là một số nguyên dương, và a là số nguyên dương lẻ không vượt quá m bất kỳ thì a j Y 1"k"m f .k/: Bài tập 8. Cho trước các số nguyên dương a; b nguyên tố cùng nhau, dãy fxngn2ZC xác định bởi x1 D a; x2 D b và xnC2 D x2nC1 C x2n xnC1 C xn; 8n 2 N!: Chứng minh rằng với xn … Z; 8n ! 3. Bài tập 9. Với mỗi số thực x, ta ký hiệu kxk là khoảng cách từ x đến số nguyên gần x nhất. Chứng minh rằng với các số nguyên dương a; b luôn tồn tại số nguyên tố p và số nguyên dương k thoả mãn****apk****C****bpk****C****a C b pk **** D 1: Bài tập 10. Chứng minh rằng với n là số nguyên dương thì lcm n 0 ! ; n 1 ! ; :::; n n !! D lcm .1; 2; : : : ; n; n C 1/ n C 1 : Bài tập 11. Cho số nguyên dương n ! 2; với mỗi s % f1; 2; : : : ; ng và s ¤ ; ta ký hiệu #.s/ D Qe2s e. Chứng minh rằng với k là số nguyên dương nhỏ hơn n thì Yn jDk lcm & 1; 2; : : : ; +n j ,' D gcd .#.s/ W jsj D n # k/: 64 Tạp chí Epsilon, Số 14, 12/2018 Tài liệu [1] www.mathscope.org [2] www.artofproblemsolving.com [3] www.mathoverflow.net [4] www.math.stackexchange.com [5] J. Dieudonne, Cơ sở giải tích hiện đại. 65 Tạp chí Epsilon, Số 14, 12/2018 PHƯƠNG PHÁP THÊM BIẾN TRONG GIẢI PHƯƠNG TRÌNH HÀM Võ Quốc Bá Cẩn (Archimedes Academy) GIỚI THIỆU Đôi khi, trong quá trình xử lý các bài toán phương trình hàm, ta có thể thêm một vài biến phụ vào để phép thế trở nên linh hoạt hơn, từ đó phát hiện được nhiều tính chất thú vị của hàm giúp ích cho việc giải toán. Bài viết này, chúng tôi xin giới thiệu cùng bạn đọc một số bài toán được xử lý bằng phương pháp này. Bài toán 1. Tìm tất cả các hàm số f : R → R thỏa mãn f!y+ f(x)"= f(x)f(y) + f!f(x)"+ f(y)−xy, ∀x, y ∈ R. Lời giải. Thay y bởi y+ f(z) vào phương trình hàm đã cho và khai triển vế phải, ta được f!y+ f(x) + f(z)"= #f(x) +1$f!y+ f(z)"+ f!f(x)"−x#y+ f(z)$ = #f(x) +1$%# f(z) +1$f(y) + f!f(z)"−yz&+ f!f(x)"−xy−x f(z) = A+ f(x)f!f(z)"−yz f(x)−x f(z), trong đó A = #f(x) + 1$# f(z) + 1$f(y) + f!f(x)"+ f!f(z)"− y(x + z). Đảo vị trí của x và z trong phương trình trên với chú ý giá trị của các biểu thức f!y+ f(x) + f(z)"và A vẫn không đổi, ta được f(x)f!f(z)"−yz f(x)−x f(z) = f(z)!f(x)"−yx f(z)−z f(x). Xem hai vế ở phương trình là hai đa thức ẩn y. Hai đa thức này có giá trị bằng nhau với mọi y nên đồng nhất với nhau, từ đó ta có z f(x) = x f(z), ∀x, z ∈ R. Suy ra f(x) = kx (k là hằng số thực nào đó) với mọi x ∈ R. Tiếp theo, ta sẽ tìm giá trị của k. Thay f(x) = kx trở lại phương trình hàm đã cho, ta được k(y+kx) = k2xy+k2x+ky−xy, ∀x, y ∈ R, hay xy(k2 −1) = 0, ∀x, y ∈ R. Do đó k = ±1. Vậy có hai hàm số thỏa mãn yêu cầu là f(x) = x và f(x) = −x. 66 Tạp chí Epsilon, Số 14, 12/2018 Bài toán 2 (IMO Shortlist, 2007). Tìm tất cả các hàm số f : R+ → R+ thỏa mãn f!x+ f(y)"= f(x+y) + f(y), ∀x, y > 0. Lời giải. Thay y bởi y+ f(z) vào phương trình đã cho rồi khai triển hai vế, ta được f!x+ f!y+ f(z)"" = f!x+y+ f(z)"+ f!y+ f(z)" = f(x+y+z) + f(y+z) +2 f(z). Mặt khác, ta lại có f!x+ f!y+ f(z)"" = f!x+ f(y+z) + f(z)" = f!x+z+ f(y+z)"+ f(z) = f!x+y+2z) + f(y+z) + f(z). Đối chiếu hai phép khai triển trên, ta được f(x+y+2z) = f(x+y+z) + f(z), ∀x, y, z > 0. Một cách tương đương, ta có f(x+y) = f(x) + f(y), ∀x > y > 0. Từ đây, với mọi x, y > 0 và z > x+y, ta đều có f(z+x+y) = f(z) + f(x+y), và Do đó f(z+x+y) = f!(z+x) +y"= f(z+x) + f(y) = f(z) + f(x) + f(y). f(x+y) = f(x) + f(y), ∀x, y > 0. Như thế, f là hàm cộng tính từ R+ vào R+. Suy ra f(x) = kx (k là hằng số dương nào đó) với mọi x > 0. Thay trở lại phương trình đã cho, ta được k!x+ky) = k(x+y) +ky, ∀x, y > 0, hay k(k −2)y = 0, ∀y > 0. Do đó k = 2. Vậy có duy nhất một hàm số thỏa mãn yêu cầu là f(x) = 2x. Bài toán 3 (IMO Shortlist, 2011). Tìm tất cả các cặp hàm số f, g : R → R thỏa mãn g!f(x+y)"= f(x)+(2x+y)g(y), ∀x, y ∈ R. Lời giải. Đặt f(0) = b và g(0) = a. Từ phương trình hàm đã cho, dễ thấy g!f(x)"= f(x) +2ax, ∀x ∈ R 67 và Từ đó suy ra Tạp chí Epsilon, Số 14, 12/2018 g!f(y)"= b+yg(y), ∀y ∈ R. f(x) = xg(x) +b−2ax, ∀x ∈ R. Bây giờ, thay x bởi x+z vào phương trình hàm đã cho, ta được g!f(x+y+z)"= f(x+z)+(2x+2z+y)g(y) = (x+z)g(x+z) +b−2a(x+z)+(2x+2z+y)g(y). Thay y bởi y+z vào phương trình hàm đã cho, ta cũng có g!f(x+y+z)"= f(x)+(2x+y+z)g(y+z) = xg(x) +b−2ax+ (2x+y+z)g(y+z). Đối chiếu hai kết quả trên, ta được xg(x) +b−2ax+ (2x+y+z)g(y+z)=(x+z)g(x+z) +b−2a(x+z)+(2x+2z+y)g(y). Trong phương trình này, cho x = y và rút gọn thành xg(x+z)=(x+z)g(x)−az, ∀x, z ∈ R. Trong phương trình này, thay x = 1 và z = x−1, ta được g(x) = xg(1)−a(x−1) = kx+a, ∀x ∈ R, trong đó k = g(1)−a. Suy ra f(x) = xg(x)+b−2ax = kx2 −ax+b với mọi x ∈ R. Thay trở lại phương trình đã cho, ta được k#k(x+y)2 −a(x+y) +b$+a = kx2 −ax+b+ (2x+y)(ky+b), ∀x, y ∈ R. So sánh hệ số của x2 ở hai vế, ta được k2 = k. Suy ra k = 0 hoặc k = 1. • Với k = 0, ta có a = −ax+b+ (2x+y)b với mọi x, y ∈ R. So sánh hệ số của y ở hai vế, ta được b = 0. Từ đó, so sánh hệ số của x ở hai vế, ta cũng có a = 0. Vậy trong trường hợp này, ta có f(x) ≡ 0 và g(x) ≡ 0. Thử lại thỏa mãn. • Với k = 1, ta có (x+y)2 −a(x+y) +b+a = x2 −ax+b+ (2x+y)(y+b), ∀x, y ∈ R, hay −ay+a = 2xb+yb, ∀x, y ∈ R. So sánh hệ số của x ở hai vế, ta được b = 0. Từ đó, so sánh hệ số của y ở hai vế, ta được a = 0. Vậy trong trường hợp này, ta có f(x) = x2 và g(x) = x. Thử lại thỏa mãn. Tóm lại, có hai cặp hàm số!f(x), g(x)"thỏa mãn yêu cầu đề bài là (0, 0) và (x2, x). 68 Tạp chí Epsilon, Số 14, 12/2018 Bài toán 4. Tìm tất cả các hàm số f : R+ → R+ thỏa mãn f ' x x−y ( = f!x f(x)"− f!x f(y)", ∀x > y > 0. Lời giải. Nếu có a > b > 0 sao cho f(a) = f(b) thì ta có f ' a a−b ( = f!a f(a)"− f!a f(b)"= 0, mâu thuẫn. Do đó f đơn ánh. Từ giả thiết, ta suy ra f ' x x−y ( + f!x f(y)"= f' x x−z ( + f!x f(z)"= f!x f(x)" với mọi x > max{y, z} > 0. Không mất tính tổng quát, ta chỉ cần xét y ! z > 0. Trong phương trình trên, chọn x = z+ 1 f(y) thì ta có x f(y) = x x−z. Suy ra = f!x f(z)". f Do f đơn ánh nên ta có x ' x x−y ( x−y = x f(z), hay x = y+ 1 f(z). Từ đó suy ra z+1 f(y) = y+1 f(z), ∀z ! y > 0. Như thế, ta có f(x) = 1 x+c (c là hằng số thực nào đó) với mọi x > 0. Vì f(x) > 0 với mọi x > 0 nên c ! 0. Thay trở lại phương trình hàm ban đầu, ta được x+c(x−y) = x+c x−y Cho x = y+1, ta được x+c(x+c)− y+c x+c(y+c), ∀x > y > 0. y+1+c = y+1+c 1 y+1+c(y+1+c)− y+c y+1+c(y+c), ∀y > 0. Trong phương trình trên, cho y → 0+, ta được 1+c+c2 − c 1+c = 1+c 1 1+c2 . Giải phương trình này, ta được c = 0. Từ đó suy ra f(x) = 1x với mọi x > 0. Thử lại ta thấy thỏa mãn. Vậy có duy nhất một hàm số thỏa mãn yêu cầu là f(x) = 1x . Bài toán 5 (IMC, 1999). Tìm tất cả các hàm số f : R+ → R+ thỏa mãn f(x+y) = f(x)f!y f(x)", ∀x, y > 0. 69 Tạp chí Epsilon, Số 14, 12/2018 Lời giải. Thay y bởi y+z vào phương trình đã cho, ta được f(x+y+z) = f(x)f!(y+z)f(x)", ∀x, y, z > 0. Thay x bởi x+z vào phương trình đã cho, ta được f(x+y+z) = f(x+z)f!y f(x+z)", ∀x, y, z > 0. Từ hai kết quả trên, ta suy ra f(x+z)f!y f(x+z)"= f(x)f!(y+z)f(x)", ∀x, y, z > 0. Giả sử tồn tại x0, z0 > 0 sao cho f(x0 + z0) > f(x0). Trong phương trình trên, ta thay x = x0, z = z0 và y = z0 f(x0) f(x0+z0)−f(x0) thì có y f(x0 +z0)=(y+z0)f(x0). Suy ra f(x0 +z0) = f(x0), mâu thuẫn. Do đó f(x+z) " f(x) với mọi x, z > 0, tức f không tăng. Xét các trường hợp sau: • Trường hợp 1: f giảm ngặt. Thay y bởi y f(x) vào phương trình đã cho, ta được ' f x+ y f(x) ( = f(x)f(y), ∀x, y > 0. Đảo vị trí của x và y trong phương trình trên với chú ý f giảm ngặt, ta được x+ y f(x) = y+x f(y), ∀x, y > 0. Thay y = 1 vào phương trình trên, ta được x + 1 f(x) = 1+ x f(1), hay f(x) = 1 kx+1 với mọi x > 0, trong đó k = 1 f(1) −1. Do f giảm ngặt từ R+ vào R+ nên dễ thấy k > 0. Thử lại, ta thấy hàm số f(x) = 1 kx+1 thỏa mãn các yêu cầu của bài toán. • Trường hợp 2: Tồn tại 0 < a < b sao cho f(a) = f(b). Lần lượt thay x = a và x = b vào phương trình đã cho, ta được f(y+a) = f(a)f!y f(a)"= f(b)!y f(b)"= f(y+b), ∀y > 0. Từ đó suy ra f(y) = f(y + b − a) với mọi y > a. Do f không giảm nên từ đây ta suy ra f(x) = C (C là hằng số dương nào đó) với mọi x > a. Bây giờ, trong phương trình hàm đã ) cho, ta cố định x và cho y > max a, a f(x) * thì có C = f(x+y) = f(x)f!y f(x)"= C f(x), suy ra f(x) = 1 với mọi x > 0. Hàm này thỏa mãn các yêu cầu của bài toán. Tóm lại, các hàm số thỏa mãn yêu cầu có dạng f(x) = 1 kx+1 với k ! 0 là hằng số nào đó. Bài toán 6 (Germany TST, 2007). Tìm tất cả các hàm số f : R+ → R+ thỏa mãn f ' f(x) y f(x) +1 ( = x x f(y) +1, ∀x, y > 0. 70 Tạp chí Epsilon, Số 14, 12/2018 Lời giải. Từ giả thiết, dễ thấy f là một đơn ánh. Thay x bởi f(x) z f(x)+1 vào phương trình đã cho rồi khai triển hai vế, ta được f + x x f(z)+1 xy x f(z)+1 +1 , = f(x) z f(x)+1 z f(x)+1 +1, f(x)f(y) hay f ' x xy+x f(z) +1 ( = f(x) f(x)f(y) +z f(x) +1, ∀x, y, z > 0. Thay y bởi f(y) vào phương trình trên, ta được f + x x#f(y) + f(z)$+1 , = f(x) f(x)#f!f(y)"+z$+1, ∀x, y, z > 0. (1) Đảo vị trí của y và z trong phương trình trên với chú ý giá trị của biểu thức f - x x[ f(y)+f(z)]+1 . vẫn không đổi, ta được f!f(y)"+z = f!f(z)"+y, ∀y, z > 0. Từ đó suy ra f!f(x)"= x+c (c là một hằng số thực nào đó) với mọi x > 0. Từ đó, phương trình (1) có thể được viết lại thành f + x x#f(y) + f(z)$+1 , = f(x) f(x)(y+z+c) +1 = f(x) f(x)f!f(y+z)"+1. Thay x bởi f(x) và y bởi f(y+z) vào phương trình đã cho, ta được f ' x+c (x+c)f(y+z) +1 ( = f(x) f(x)f!f(y+z)"+1, ∀x, y, z > 0. Kết hợp với kết quả trên, ta được + , x ' x+c ( f x#f(y) + f(z)$+1 = f (x+c)f(y+z) +1 , ∀x, y, z > 0. Do f đơn ánh nên ta có x#f(y) + f(z)$+1 = x+c x (x+c)f(y+z) +1, ∀x, y, z > 0, hay x#(x+c)f(y+z) +1$= (x+c)%x#f(y) + f(z)$+1&, ∀x, y, z > 0. Xem hai vế của phương trình trên là các đa thức ẩn x. Hai đa thức này có giá trị bằng nhau với mọi x > 0 nên đồng nhất với nhau, từ đó bằng cách so sánh hệ số của x2 ở hai vế, ta suy ra f(y+z) = f(y) + f(z), ∀y, z > 0. Kết quả này chứng tỏ f là hàm cộng tính từ R+ vào R+. Suy ra f(x) = kx (k là hằng số dương nào đó). Thay trở lại phương trình đã cho, ta được k2x kxy+1 = x kxy+1, ∀x, y > 0. Do đó k = 1. Vậy có duy nhất một hàm số thỏa mãn yêu cầu là f(x) = x. 71 Tạp chí Epsilon, Số 14, 12/2018 Bài toán 7. Tìm tất cả các hàm số f : R+ → R+ thỏa mãn f!x f(x) + f(y)"= y+ f 2(x), ∀x, y > 0. Lời giải. Từ giả thiết, dễ thấy f đơn ánh. Đặt f(1) = a, ta có f!f(y) +a"= y+a2, ∀y > 0. Suy ra hàm f có thể nhận mọi giá trị trên (a2, +∞). Thay y bởi f 2(y) vào phương trình hàm đã cho, ta được f!x f(x) + f!f 2(y)"" = f 2(y) + f 2(x), ∀x, y > 0. Đảo vị trí của x và y trong phương trình trên với chú ý f đơn ánh, ta được x f(x) + f!f 2(y)"= y f(y) + f!f 2(x)", ∀x, y > 0. Từ đó suy ra x f(x) = f!f 2(x)"+ c (c là một hằng số thực nào đó) với mọi x > 0. Từ đây, kết hợp với (1) và giả thiết, ta có f!y+ f 2(x) +a"= f!f!x f(x) + f(y)"+a" = x f(x) + f(y) +a2 = f!f 2(x)"+ f(y) +a2 +c. Do f(x) có thể nhận mọi giá trị trên (a2, +∞) nên ta có f(x+y+a) = f(x) + f(y) +a2 +c, ∀x > a4, y > 0. Từ đây, với mọi x, y > 0 và với mọi z > a4, ta có f(z+x+y+2a) = f!(z+x+a) +y+a" = f(z+x+a) + f(y) +a2 +c = f(z) + f(x) + f(y) +2(a2 +c) vàf(z+x+y+2z) = f!z+ (x+y+a) +a" = f(z) + f(x+y+a) +a2 +c. Kết hợp hai kết quả trên lại, ta được f(x+y+a) = f(x) + f(y) +a2 +c, ∀x, y > 0. Trong phương trình trên, ta lần lượt thay x, y bởi x+y 2 , x+y 2 thì có f(x+y+a) = 2 f 'x+y 2 ( +a2 +c, ∀x, y > 0. Từ đó suy ra 2 f 'x+y 2 ( = f(x) + f(y), ∀x, y > 0. 72 Tạp chí Epsilon, Số 14, 12/2018 Với mọi x, y, z > 0, ta có 4 f 'x+y+z 2 ( = 2#f(z+x) + f(y)$= f(2z) + f(2x) +2 f(y). Đảo vị trí của x và y trong phương trình trên, ta được f(2x) +2 f(y) = f(2y) +2 f(x), ∀x, y > 0. Từ đó suy ra f(2x) = 2 f(x) +m (m là hằng số thực nào đó) với mọi x > 0. Đặt g(x) = f(x) +m thì ta có g(2x) = 2g(x) và g(x) +g(y) = 2 f 'x+y 2 ( +2m = 2g 'x+y 2 ( = g(x+y), ∀x, y > 0. Từ phương trình trên, bằng quy nạp ta chứng minh được g(nx) = ng(x) với mọi n ∈ Z+ và với mọi x > 0. Do g(nx) = f(nx) +m > m nên ta có g(x) >mn , ∀x > 0, n ∈ Z+. Cho n → +∞, ta được g(x) ! 0 với mọi x > 0. Từ đó, ta suy ra g là hàm cộng tính từ R+ vào R!0. Theo tính chất của hàm cộng tính, ta có g(x) = kx (k là hằng số không âm nào đó) với mọi x > 0. Suy ra f(x) = kx−m với mọi x > 0. Thay kết quả này trở lại phương trình đã cho, ta được k(kx2 −mx+ky−m)−m = y+ (kx−m)2, ∀x, y > 0. So sánh hệ số của y ở hai vế, ta được k2 = 1, hay k = 1 (do k ! 0). Từ đó, ta có x2 −mx−2m = (x−m)2, ∀x > 0. So sánh hệ số của x ở hai vế, ta được m = 0. Như vậy, ta có f(x) = x với mọi x > 0. Thử lại, ta thấy thỏa mãn. Vậy có duy nhất một hàm số thỏa mãn yêu cầu là f(x) = x. Bài toán 8. Tìm tất cả các hàm số f : Z+ → Z+ thỏa mãn f(a+b) = f(a) + f(b) + f(c) + f(d) với mọi a, b, c, d nguyên dương sao cho c2 +d2 = 2ab. Ý tưởng chính cho lời giải bài toán trên là tìm các số nguyên dương a, b, c, d, e, g (trong đó {c, d} ̸= {e, f }) thỏa mãn 2ab = c2 +d2 = e2 +g2. Khi đó, ta sẽ có f(c) + f(d) = f(e) + f(g). Ngoài ra, nếu chọn c, d, e, g là các biểu thức có dạng tuyến tính bậc nhất theo n thì ta có thể hoàn tất lời giải bằng phương pháp quy nạp theo n. Ý tưởng biểu diễn một số thành tổng hai bình phương theo hai cách khác nhau gợi ta nghĩ đến đồng nhất thức Lagrange như sau: Với mọi số nguyên m, n, p, q, ta có (m2 +n2)(p2 +q2)=(mp+nq)2 + (mq−np)2 = (mp−nq)2 + (mq+np)2. Chọn m = p = 2 và q = 4, ta được (4n+4)2 + (2n−8)2 = (4n−4)2 + (2n+8)2 = 20(n2 +4). Dựa trên đẳng thức này và một số tính chất riêng của hàm f , ta thu được lời giải như sau. 73 Tạp chí Epsilon, Số 14, 12/2018 Lời giải. Đặt f(1) = k. Thay a = b = c = d = n vào phương trình đã cho, ta được f(2n) = 4 f(n), ∀n ∈ Z+. (1) Thay a = 10, b = n2 +4, c = 4n+4 và d = 2n−8 (n > 4), ta được f(n2 +14) = f(10) + f(n2 +4) + f(4n+4) + f(2n−8) = f(10) + f(n2 +4) +16 f(n+1) +4 f(n−4). Mặt khác, thay a = 10, b = n2 +4, d = 4n−4 và d = 2n+8, ta cũng có f(n2 +14) = f(10) + f(n2 +4) + f(4n−4) + f(2n+8) = f(10) + f(n2 +4) +16 f(n−1) +4 f(n+4). Kết hợp hai kết quả trên, ta được f(n+4) +4 f(n−1) = f(n−4) +4 f(n+1), ∀n > 4. (2) Bây giờ, ta sẽ tính giá trị của f(2), f(3), f(4),..., f(8). Từ (1), dễ thấy f(2) = 4k, f(4) = 16k và f(8) = 64k. Do 2 · 5 · 1 = 32 +12 nên ta có 4 f(3) = f(6) = f(5+1) = f(5) + f(1) + f(3) + f(1) = f(5) + f(3) +2a. Mặt khác, ta cũng có 2 · 4 · 1 = 22 +22 nên f(5) = f(4+1) = f(4) + f(1) + f(2) + f(2) = 25k. Kết hợp với kết quả ở trên, ta được f(3) = 9k. Suy ra f(6) = 4 f(3) = 36k. Ta cũng có 72 +12 = 52 +52 = 2 · 25 · 1 nên f(26) = f(25) + f(1) + f(5) + f(5) = f(25) + f(1) + f(7) + f(1), suy ra f(7) = 2 f(5)− f(1) = 49k. Như vậy, ta chứng minh được f(n) = kn2 với n ∈ {1, 2,..., 8}. Kết hợp với (2), ta dễ dàng quy nạp được f(n) = kn2 với mọi n nguyên dương. Thử lại, ta thấy hàm này thỏa mãn các yêu cầu của bài toán. Vậy có duy nhất một hàm số thỏa mãn yêu cầu là f(n) = kn2 (k là hằng số nguyên dương nào đó). Bài toán 9. Tìm tất cả các cặp hàm số f, g : R → R thỏa mãn f(x3 +2y) + f(x+y) = g(x+2y), ∀x, y ∈ R. (1) Lời giải. Từ giả thiết, ta cũng có f(z3 +2t) + f(z+t) = g(z+2t), ∀z, t ∈ R. (2) Ta sẽ chứng minh rằng, với mọi a, b ∈ R, hệ phương trình sau luôn có nghiệm (x, y, z, t) với x, y, z, t ∈ R :⎧⎪⎪⎪⎪⎨ ⎪⎪⎪⎪⎩ x+2y = a, z+2t = b, x3 +2y = z+t, x+y = z3 +2t. 74 Tạp chí Epsilon, Số 14, 12/2018 Từ các phương trình thứ nhất và thứ hai, ta lần lượt có x = a−2y và z = b−2t. Thay vào phương trình thứ ba, ta được (a−2y)3 +2y = b−t, suy ra t = b−2y−(a−2y)3. Thay x = a−2y, z = b−2t, t = b−2y−(a−2y)3 vào phương trình thứ tư của hệ, ta được a−y = (b−2t)3 +2t = %b−2#b−2y−(a−2y)3$&3+2#b−2y−(a−2y)3$. Đây là phương trình bậc 9 ẩn y nên luôn có ít nhất một nghiệm thực y0. Từ đó suy ra hệ luôn có ít nhất một nghiệm thực (x0, y0, z0, t0) với x0 = a−2y0, z0 = b−2t0 và t0 = b−2y0 −(a−2y0)3. Khẳng định được chứng minh. Từ khẳng định vừa chứng minh và các phương trình (1), (2), ta dễ dàng suy ra g(a) = g(b) với mọi a, b ∈ R. Do đó g(x) ≡ C. Thay trở lại (1), ta được f(x3 +2y) + f(x+y) = C, ∀x, y ∈ R. Thay y = x−x3 vào phương trình trên, ta được f(2x−x3) = C2, ∀x ∈ R. Do 2x−x3 có thể nhận mọi giá trị trên R nên từ đây, ta có f(x) = C2 với mọi x ∈ R. Thử lại, ta thấy f(x) = C2 và g(x) = C thỏa mãn yêu cầu. Vậy có duy nhất một cặp hàm số f, g thỏa mãn yêu cầu là f(x) = C2 và g(x) = C (C là một hằng số thực nào đó). Bài toán 10. Tìm tất cả các hàm số f : R → R thỏa mãn đồng thời các điều kiện sau với mọi bộ số thực (a, b, c) : a) Nếu a+b+c ! 0 thì f(a3) + f(b3) + f(c3) ! 3 f(abc). b) Nếu a+b+c " 0 thì f(a3) + f(b3) + f(c3) " 3 f(abc). Lời giải. Để ý rằng, nếu hàm số f thỏa mãn yêu cầu thì hàm số g với g(x) = f(x)−C (C là hằng số nào đó) cũng thỏa mãn yêu cầu. Do đó, không mất tính tổng quát, ta chỉ cần xét trường hợp f(0) = 0. Từ giả thiết, ta suy ra f(a3) + f(b3) + f(c3) = 3 f(abc) (1) với mọi a, b, c ∈ R mà a+b+c = 0. Thay c = 0 vào (1), ta được f(−a3) = f(b3) = −f(a3), suy ra f là hàm lẻ. Từ đây, thay c = −(a+b) vào (1), ta được f!(a+b)3"= f(a3) + f(b3) +3 f!ab(a+b)", ∀a, b > 0. (2) Thay b bởi b+c vào phương trình trên và khai triển vế phải, ta được f!(a+b+c)3"= f(a3) + f!(b+c)3"+3 f!a(b+c)(a+b+c)" = f(a3) + f(b3) + f(c3) +3 f!bc(b+c)"+3 f!a(b+c)(a+b+c)". 75 Tạp chí Epsilon, Số 14, 12/2018 Đảo vị trí của a và c trong dãy đẳng thức trên, ta được f!bc(b+c)"+ f!a(b+c)(a+b+c)"= f!ab(a+b)"+ f!c(a+b)(a+b+c)". (3) Tiếp theo, ta sẽ chứng minh rằng với mọi 0 < x < y, hệ phương trình sau luôn có nghiệm (a, b, c) với a, b, c > 0:⎧⎪⎨ ⎪⎩ bc(b+c) = x, a(b+c)(a+b+c) = y, ab(a+b) = c(a+b)(a+b+c). (4) Hệ phương trình này tương đương với ⎧⎪⎨ ⎪⎩ bc(b+c) = x, a(b+c)(a+b+c) = y, ab = c(a+b+c). Từ phương trình thứ hai và phương trình thứ ba, ta có a2b(b+c) = cy. Kết hợp với phương trình thứ nhất, ta được a2x = c2y. Suy ra c = ka với k =3xy . Ngoài ra, từ phương trình thứ nhất và phương trình thứ hai, ta cũng có xa(a+b+c) = ybc. Suy ra k(a+b+ka) = b, hay b = ka(1+k) 1−k . Thay c = ka và b = ka(1+k) 3 x(k−1)2 1−k vào phương trình thứ nhất của hệ, ta giải ra được a = 3 2k3(k+1). Như thế, hệ (4) luôn có nghiệm (a, b, c) với a, b, c > 0. Kết hợp kết quả này với (3) và chú ý ab(a+b) +c(a+b)(a+b+c) = bc(b+c) +a(b+c)(a+b+c) = x+y, ta được f(x) + f(y) = 2 f 'x+y 2 ( , ∀x, y > 0, x < y. Dế thấy f(x) + f(x) = 2 f!x+x 2 "nên ta có 'x+y ( f(x) + f(y) = 2 f 2 Từ đây, với mọi x, y, z > 0, ta có , ∀x, y > 0. (5) 4 f 'x+y+z 2 ( = 2#f(x+z) + f(y)$= f(2x) + f(2z) +2 f(y). Đảo vị trí của x và y trong đẳng thức trên, ta được f(2x) + 2 f(y) = f(2y) + 2 f(x) với mọi x, y > 0. Suy ra f(2x) = 2 f(x) +M (M là hằng số thực nào đó) với mọi x > 0. Do đó f(8x) = 2 f(4x) + M = 4 f(2x) +3M = 8 f(x) +7M, ∀x > 0. Mặt khác, từ (2), ta cũng có f(8x) = 2 f(x) +3 f(2x). Suy ra 8 f(x) +7M = 2 f(x) +3#2 f(x) +M$, và do đó M = 0. Như vậy, ta có f(2x) = 2 f(x) với mọi x > 0. Kết hợp với (5), ta được f(x+y) = f(x) + f(y), ∀x, y > 0. 76 Tạp chí Epsilon, Số 14, 12/2018 Mặt khác, từ điều kiện a) của bài toán và f(0) = 0, ta dễ dàng suy ra f(a) ! 0 với mọi a ! 0. Như vậy, f là hàm cộng tính trên R+ và f(x) ! 0 với mọi x > 0 nên ta có f(x) = ℓx (ℓ là hằng số không âm nào đó) với mọi x > 0. Mà f lẻ nên f(x) = ℓx, ∀x ∈ R. Thử lại, ta thấy thỏa mãn. Vậy các hàm số thỏa mãn yêu cầu có dạng f(x) = ℓx+C (ℓ, C là các hằng số thực, ℓ ! 0). Bài toán 11. Tìm tất cả các hàm số f : R → R thỏa mãn f!x+ f(y)"= f(y2 +3) +2x f(y) + f(x)−3, ∀x, y ∈ R. Lời giải. Dễ thấy f(x) ≡ 0 không thỏa mãn phương trình đã cho nên tồn tại y0 ∈ R sao cho f(y0) ̸= 0. Thay y = y0 vào phương trình đã cho, ta được f!x+ f(y0)"− f(x) = 2x f(y0) + f(y20 +3)−3, ∀x ∈ R. Vế phải của phương trình trên là một hàm bậc nhất ẩn x nên nó có thể nhận mọi giá trị trên R. Từ đó suy ra hiệu f(u)− f(v) có thể nhận mọi giá trị trên R. Bây giờ, thay x bởi x− f(y) vào phương trình đã cho, ta được f!x− f(y)"= f(x)− f(y2 +3)−2!x− f(y)"f(y) +3, ∀x, y ∈ R. Trong phương trình trên, ta thay x bởi x+ f(z) thì được f!x+ f(z)− f(y)"= f!x+ f(z)"− f(y2 +3)−2#x+ f(z)− f(y)$f(y) +3 = f(x) + f(z2 +3) +2x f(z)− f(y2 +3)−2#x+ f(z)− f(y)$f(y) = f(x) +2x#f(z)− f(y)$+ f(z2 +3)− f(y2 +3) +2 f(y)#f(y)− f(z)$. Đảo vị trí của y và z trong phương trình trên rồi cộng phương trình thu được và phương trình trên lại theo vế, ta được f!x+ f(z)− f(y)"+ f!x+ f(y)− f(z)"= 2 f(x) +2#f(y)− f(z)$2. Do hiệu f(y)− f(z) có thể nhận mọi giá trị trên R nên từ đây ta có f(x+y) + f(x−y) = 2 f(x) +2y2, ∀x, y ∈ R. Đặt f(0) = a và g(x) = f(x)−x2 −a thì ta có g(0) = 0 và g(x+y) +g(x−y) = 2g(x), ∀x, y ∈ R. Thay x = y vào phương trình này, ta được g(2x) = 2g(x) với mọi x ∈ R nên ta có g(x+y) +g(x−y) = g(2x), ∀x, y ∈ R. Lần lượt thay x, y bởi x+y 2 , x−y 2 vào phương trình trên, ta suy ra g là hàm cộng tính. Tiếp theo, thay x = 0 vào phương trình đã cho, ta được f!f(y)"= f(y2 +3) +a−3, ∀y ∈ R. 77 Tạp chí Epsilon, Số 14, 12/2018 Thay f(y) = g(y) +y2 +a vào và rút gọn với chú ý g cộng tính, ta được g!g(y)"+g2(y) +2(y2 +a)g(y) +2(a−3)(y2 +a+2) +g(a)−g(3) = 0. Thay y = 0 vào hương trình trên, ta được 2(a−3)(a+2) +g(a)−g(3) = 0. Từ đó suy ra g!g(y)"+g2(y) +2(y2 +a)g(y) +2(a−3)y2 = 0, ∀y ∈ R. Thay y bởi ny (n ∈ Z) vào phương trình trên, ta được ng!g(y)"+n2g2(y) +2n(n2y2 +a)g(y) +2n2(a−3)y2 = 0, ∀y ∈ R, n ∈ Z. Ta xem vế trái là một đa thức ẩn n. Đa thức này có giá trị bằng 0 tại vô hạn giá trị của n nên nó phải đồng nhất bằng 0. Từ đó suy ra hệ số của n3 phải bằng 0, hay ta có 2y2g(y) = 0 với mọi y ∈ R. Kết hợp với g(0) = 0, ta suy ra g(y) = 0 với mọi y ∈ R. Từ đây với chú ý 2(a − 3)(a + 2) + g(a) − g(3) = 0, ta tính được a = 3 hoặc a = −2. Suy ra f(x) = x2 + 3 với mọi x ∈ R hoặc f(x) = x2 −2 với mọi x ∈ R. Thử lại, ta thấy chỉ có hàm f(x) = x2 +3 thỏa mãn yêu cầu. Vậy có duy nhất một hàm số thỏa mãn yêu cầu là f(x) = x2 +3. Trên đây, chúng tôi đã giới thiệu cùng bạn đọc một số bài toán hay giải được bằng phương pháp thêm biến. Rất mong sẽ nhận được trao đổi đóng góp xây dựng từ bạn đọc gần xa. Dưới đây là một số bài tập khác cũng có thể giải bằng phương pháp này: Bài tập 1. Tìm tất cả các hàm số f : R → R thỏa mãn f(x+y) = f(x)f(y)f(xy), ∀x, y ∈ R. Bài tập 2 (IMO Shortlist, 2005). Tìm tất cả các hàm số f : R+ → R+ thỏa mãn f(x)f(y) = 2 f!x+y f(x)", ∀x, y > 0. Bài tập 3. Tìm tất cả các hàm số f : R+ → R+ thỏa mãn f!x+ f(y)"= f(x) + f(x+y)−x, ∀x, y > 0. Bài tập 4. Tìm tất cả các hàm số f liên tục, f : R → R thỏa mãn f(x+y) + f(xy) = f(x) + f(y) + f(xy+1), ∀x, y ∈ R. Bài tập 5. Tìm tất cả các hàm số f : Z+ → Z+ thỏa mãn f!f 2(m) +2 f 2(n)"= m2 +2n2, ∀m, n ∈ Z+. Bài tập 6. Cho hàm số f : R → R thỏa mãn f(0) = 0 và f(x) + f(y) = f(x+y−xy) + f(xy), ∀x, y ∈ R. Chứng minh rằng f là hàm cộng tính. 78 Tạp chí Epsilon, Số 14, 12/2018 SÁNG TẠO - LÀM CHẶT Ngô Văn Thái, Thái Bình Một tính chất quan trọng của bất đẳng thức toán học mà học sinh được biết ngay từ khi còn đang học tiểu học, đó là tính chất bắc cầu: “Nếu A ! B; B ! C thì A ! C:” Tính chất này được sử dụng làm phép suy luận logic ở hầu khắp mọi bài toán chứng minh bất đẳng thức. Thế nhưng khi đứng trước một bài toán chứng minh bất đẳng thức A ! C; không phải bài toán nào người ra đề cũng để người giải nhìn ra ngay A ! B; B ! C: Nếu bài toán đề ra mà người giải nhìn thấy ngay A ! B; B ! C thì bài toán đó quá tầm thường. Còn với những bài toán bất đẳng thức hay và khó, người giải phải sử dụng các kiến thức cơ bản phù hợp, cách giải phải tường minh, linh hoạt ngắn gọn để được A ! B và B ! C; thì người giải đã làm tốt được hai việc. Việc thứ nhất là đã hoàn thành chứng minh bài toán đề ra, việc thứ hai là đã sáng tạo ra được một bài toán bất đẳng thức chặt hơn, khó hơn (có thể hay hơn) bài toán đã cho. Để thấy được vẻ đẹp muôn màu của sự sáng tạo bất đẳng thức toán học, tôi xin giới thiệu bài viết: “Sáng tạo bất đẳng thức bằng cách làm chặt một số bất đẳng thức nổi tiếng” dựa vào ý tưởng từ “Tính chất bắc cầu của bất đẳng thức”. Nội dung bài viết gồm năm phần sau đây: ! Làm chặt bất đẳng thức AM-GM. ! Làm chặt bất đẳng thức Cauchy-Schwarz. ! Làm chặt bài toán bất đẳng thức đăng trên báo Toán học và Tuổi trẻ. ! Làm chặt một bài toán bất đẳng thức thi Toán học Quốc Tế. ! Giới thiệu những bài toán mới được làm chặt từ những bài toán đã biết. 1. Làm chặt bất đẳng thức AM-GM Bất đẳng thức AM-GM là một bất đẳng thức cơ bản kinh điển quan trọng nhất của toán học sơ cấp, vì nó đã có khá nhiều cách chứng minh được đưa ra, hàng chục mở rộng, hàng chục kết quả chặt hơn đăng trên các diễn đàn toán học. Phần này tôi xin giới thiệu một kết quả chặt hơn bất đẳng thức AM-GM khác được suy ra từ chính cách chứng minh mới bất đẳng thức AM-GM. Bất đẳng thức AM-GM. Với mọi số thực không âm a1; a2; :::; an .n ! 2/ ta có a1 C a2 C """ C an ! npn a1a2 """ an: (1) 79 Tạp chí Epsilon, Số 14, 12/2018 Lời giải. Nếu a1a2 """ an D 0 thì (1) đúng. Xét a1a2 """ an > 0; dễ thấy với mọi X ! 0 thì .1 # X/;!n # "1 C X C X2 C """ C Xn!1#$ cùng dấu. Ta có thể giả sử .1 # X/!n # "1 C X C X2 C """ C Xn!1#$ ! 0; hay là n .1 # X/ # .1 # Xn/ ! 0; Bây giờ đặt X D nq an .n # 1/ C Xn ! nX: n!p1 a1a2"""an!1; thì .n # 1/ Can r an n!p1 a1a2 """ an!1! n n n!p1 a1a2 """ an!1; Do đó .n # 1/ n!p1 a1a2 """ an!1 C an ! npn a1a2 """ an; (2) .n # 2/ n!p2 a1a2 """ an!2 C an!1 ! .n # 1/ n!p1 a1a2 """ an!1; """""" 2pa1a2 C a3 ! 3p3 a1a2a3; a1 C a2 ! 2pa1a2: Cộng vế với vế của n # 1 ở trên và rút gọn sẽ được a1 C a2 C """ C an ! npn a1a2 """ an: Đẳng thức xảy ra khi và chỉ khi a1 D a2 D """ D an: Ta có điều phải chứng minh. Như vậy từ bất đẳng thức (2) chúng ta đã có thêm một cách chứng minh mới bất đẳng thức AM-GM khá gọn, ngoài ra khi sử dụng bất đẳng thức (2) lại dễ dàng rút ra được kết quả chặt hơn bất đẳng thức AM-GM sau đây. Với các số tự nhiên n ! k ! p ! 2 và n số thực không âm, khi đó a1 C a2 C """ C an ! p pp a1a2 """ ap C apC1 C """ C an ! kpk a1a2 """ ak C akC1 C """ C an ! npn a1a2 """ an: 2. Làm chặt bất đẳng thức Cauchy-Schwarz Trong toán học bất đẳng thức AM-GM và Cauchy-Schwarz được dùng làm định lý cơ bản của lý thuyết bất đẳng thức. Với tư cách là hai hòn đá tảng để nhiều kết luận quan trọng khác của toán học dựa vào, cặp bất đẳng thức AM-GM, Cauchy-Schwarz được sử dụng khá phổ biến ở 80